1. Простые эксперименты - Высшая школа экономики

advertisement
d005
d006
d007
ezka
d008
d009
d010
d011
d012
Задачи по элементарной теории вероятностей и матстатистике
Обращение к читателю
Задачник - в стадии разработки. Смелее спрашивайте и высказывайте свое мнение Борису Демешеву, roah@yandex.ru
Предлагайте свои задачи!
Цель
Есть много сборников задач. Зачем этот:
- Открытость и доступность. www.xion.ru - учеба - 2 курс - теория вероятностей
- Красивые задачи.
- Задачи под курс ГУ-ВШЭ.
Все то, что можно рассказать без теории меры
Об ответах и упрощениях
a
и их очевидно нельзя упростить. Некоa+b
торые ответы простым образом выражаются через биномиальные коэффициенты. Не упрощаются,
Pn 1 Pn k
но встречаются в ответах:
i=1 i ,
i=1 i . Ответы в виде громоздких сумм биномиальных коэффициентов не используются, если это не оговорено в условии. Используется сумма геометрической
x
прогрессии, разложение в ряд Тейлора для e .
Большинство ответов имеют совсем простой вид в духе
1. Простые эксперименты
Задача 1.1.
Подбрасывается два кубика. Какова вероятность выпадения хотя бы одной шестерки? Какова
вероятность того, что шестерка не выпадет ни разу?
Задача 1.2.
Монетка подбрасывается 5 раз. Какова вероятность того, что будет выпадет ровно 1 «орел»? Ровно
2? Ни одного?
Задача 1.3.
Какова вероятность при шести подбрасываниях кубика получить ровно две шестерки?
Задача 1.4.
На отрезке равномерно и независимо выбираются две точки. Верно ли, что длины получающихся
трех отрезков распределены одинаково?
Задача 1.5.
Из 50 деталей 4 бракованных. Выбирается наугад 10 на проверку. Какова вероятность не заметить
брак?
Задача 1.6.
Ω = {a, b, c}, P ({a, b}) = 0, 8, P ({b, c}) = 0, 7.
Найдите
P ({a}), P ({b}), P ({c})
Задача 1.7.
A
и
B
несовместны,
P (A) = 0, 3, P (B) = 0, 4.
Найдите
P (Ac ∩ B c )
Задача 1.8.
P (A) = 0, 3, P (B) = 0, 8.
В каких пределах может лежать
P (A ∩ B)?
Задача 1.9.
Четыре карты одного достоинства. Наугад выбираются две. Вероятность того, что они будут разного
1
d013
d014
d015
d016
z 36
d018
d019
d020
etsi
feti
olov
ulli
2
цвета?
Задача 1.10.
Кубик подбрасывается два раза. Вероятность того, что результат второго броска будет строго
больше, чем результат первого? Вероятность того, что в сумме будет 6? В сумме 9? Максимум равен
5? Минимум равен 3? Разница будет равна 1 или 0?
Задача 1.11.
Как связаны между собой
P (A)
и
E(1A )?
Задача 1.12.
Равной силы команды играют до 3-х побед. Какова вероятность того, что будет ровно 3 партии?
Ровно 4? Ровно 5?
Задача 1.13.
Какова вероятность того, что у 10 человек не будет ни одного совпадения дней рождений? Каков минимальный размер компании, чтобы вероятность одинакового дня рождения была больше половины?
Задача 1.14.
Какова вероятность полностью угадать комбинацию в лотерее 5 из 36?
Задача 1.15.
В мешке 50 орехов из них 5 - пустые. Вы выбираете наугад 10 орехов, какова вероятность того, что
ровно 1 из них будет пустой?
Задача 1.16.
Маша подбрасывает монетку три раза, а Саша - два раза. Какова вероятность того, что у Маши
«герб» выпадет больше раз, чем у Саши?
Задача 1.17.
Подбрасывается кубик, а затем монетка подбрасывается столько раз, сколько очков на выпавшей
грани. Какова вероятность того, что «орел» выпадет ровно 4 раза?
Задача 1.18. Рыцари-близнецы [Mosteller]
Король Артур проводит рыцарский турнир, в котором, так же как и в теннисе, порядок состязания
определяется жребием (см. Выйдет ли второй в финал?). Среди восьми рыцарей, одинаково искусных в ратном деле, два близнеца.
а) Какова вероятность того, что они встретятся в поединке?
n
б) Каков ответ в случае 2 рыцарей?
Задача 1.19.
На подносе лежит 20 шоколадных конфет, одинаковых с виду. В четырех из них есть орех внутри.
Маша съела 5 конфет. Какова вероятность того, что в наугад выбранной оставшейся конфете будет
орех?
Задача 1.20.
Сколько детей должно быть в семье, чтобы вероятность того, что имеется по крайней мере один
ребенок каждого пола была больше 0,95?
Задача 1.21.
Пусть
X
принимает два значения, причем
P (X = 1) = p
и
P (X = 0) = 1 − p.
Найдите
E(X).
umme
tsii
Juan
alos
nnih
enia
3
Задача 1.22.
Кубик подбрасывают до тех пор, пока накопленная сумма очков на гранях не превысит 2. Пусть
- число подбрасываний кубика. Найдите
X
E(X), V ar(X), V ar(36X − 5), E(36X − 17)
Задача 1.23.
Поезда метро идут регулярно с интервалом 3 минуты. Пассажир приходит на платформу в случай-
X - время
P (X < 1), E(X), V ar(X)
ный момент времени. Пусть
Найдите
ожидания поезда в минутах.
Задача 1.24.
Вася посещает 60% лекций по теории вероятностей, Петя - 70%. Они из разных групп и посещают
лекции независимо друг от друга. Какова вероятность, что на следующую лекцию придут оба? Хотя
бы один из них?
Задача 1.25. Судьба Дон-Жуана
n знакомых девушек (их всех зовут по-разному). Он пишет им n писем, но, по рассеянности,
раскладывает их в конверты наугад. С.в. X обозначает количество девушек, получивших письма,
У Васи
написанные лично для них.
а) Найдите
E(X), V ar(X)
б) Какова вероятность того, что хотя бы одна девушка получит письмо, адресованное именно ей?
Каков предел этой вероятности при
n → +∞?
в) Какова вероятность того, что произойдет ровно
k
совпадений?
Задача 1.26. С чего все начиналось...
Париж, Людовик XIV, 1654 год, высшее общество говорит о рождении новой науки - теории вероятностей. Ах, кавалер де Мере, «fort honnete homme sans etre mathematicien»... Тигр: «благородный
человек, хотя и не математик». Старая задача, неправильные решения которой предлагались тысячелетиями (например, одно из неправильных решений предлагал изобретатель двойной записи,
кумир бухгалтеров, Лука Пачоли) наконец решена правильно! Два игрока играют в честную игру
до шести побед. Игрок первым выигравший шесть партий (не обязательно подряд) получает 800
рублей. К текущему моменту первый игрок выиграл 5 партий, а второй - 3 партии. Они вынуждены
прервать игру в данной ситуации.
Как им поделить приз по справедливости?
Задача 1.27.
Из 5-ти деталей 3 бракованных. Сколько потребуется в среднем попыток прежде чем обнаружится
первая дефектная деталь? Какова дисперсия числа попыток?
Задача 1.28. [Mosteller]
Две урны содержат одно и то же количество шаров, несколько черных и несколько белых каждая.
Из них извлекаются
n (n > 3)
шаров с возвращением. Найти число
n
и содержимое обеих урн, если
вероятность того, что все белые шары извлечены из первой урны, равна вероятности того, что из
второй извлечены либо все белые, либо все черные шары.
Задача 1.29. [Mosteller] Осторожный фальшивомонетчик
Дворцовый чеканщик кладет в каждый ящик вместимостью в сто монет одну фальшивую. Король
подозревает чеканщика и подвергает проверке монеты, взятые наудачу по одной в каждом из 100
ящиков.
а) Какова вероятность того, что чеканщик не будет разоблачен?
б) Каков ответ в предыдущей задаче, если 100 заменить на
n?
Задача 1.30. Стратегия удвоения
В казино имеется рулетка, которая с вероятностью по
1/2
выпадает на черное и на красное. Игрок,
ulki
nale
4
поставивший сумму
n и угадавший цвет, получает обратно сумму 2n. Вася решил играть по следую-
щей схеме. Сначала он ставит доллар. Если он выигрывает, то покидает казино, если проигрывает,
то удваивает ставку и ставит два доллара. Если выигрывает, то покидает казино, если проигрывает,
то снова удваивает ставку и ставит четыре доллара. И т.д. пока не выиграет в первый раз или
впервые не хватит денег на новую удвоенную ставку. У Васи имеется 1050 долларов.
а) Какова вероятность того, что Вася покинет казино после выигрыша?
б) Каков ожидаемый выигрыш Васи?
Комментарий: в реальности вероятность меньше
1/2,
т.к. на рулетке есть 0 и (иногда) 00. Их
наличие, естественно, уменьшает и вероятность и ожидаемый выигрыш.
Задача 1.31. Две шкатулки
Васе предлагают две шкатулки. И обещают, что в одной из них денег в два раз больше, чем в
другой. Вася открывает наугад одну из них - в ней
a
рублей. Вася может либо взять деньги, либо
взять оставшуюся шкатулку.
а) Правильно ли Вася считает, что ожидаемое количество денег в неоткрытой шкатулке равно
1 1
( a) + 12 (2a) = 1 41 a, и, поэтому, нужно изменить свой выбор?
2 2
1 k
k
k+1
б)Пусть известно, что в пару шкатулок кладут 3 и 3
рублей с вероятностью pk = ( ) . Стоит ли
2
Васе изменить свой выбор, после того, как он открыл первую шкатулку? Почему?
Задача 1.32. [обработка Mosteller] Выйдет ли второй в финал?
В теннисном турнире участвуют 8 игроков. Есть три тура (четверьтфинал-полуфинал-финал).
Противники в первом туре определяются случайным образом. Предположим, что лучший игрок
всегда побеждает второго по мастерству, а тот в свою очередь побеждает всех остальных. Проигрывающий в финале занимает второе место. Какова вероятность того, что это место займет второй по
мастерству игрок?
Задача 1.33. [Mosteller] Выборы
После выборов, в которых участвуют два кандидата, A и B, за них поступило
a
и
b (a > b)
бюллетеней соответственно, скажем, 3 и 2. Если подсчет голосов производится последовательным
извлечением бюллетеней из урны, то какова вероятность того, что хотя бы один раз число вынутых
бюллетеней, поданных за А и В, было одинаково?
Задача 1.34. [Mosteller] Ничьи при бросании монеты
Игроки А и В в орлянку играют
N
раз. После первого бросания каковы шансы на то, что в течение
всей игры их выигрыши не совпадут?
Задача 1.35.
Бросают два правильных игральных кубика. Пусть
X
- наименьшая из выпавших граней, а
Y
-
наибольшая.
P (X = 3 ∩ Y = 5);
E(X), V ar(X), E(3X − 2Y );
а) Рассчитайте
б) Найдите
from cut-the-knot
Задача 1.36.
A bag contains a counter, known to be either white or black. A white counter is put in, the bag is shaken,
and a counter is drawn out, which proves to be white. What is now the chance of drawing a white counter?
Задача 1.37.
You have a hat in which there are three pancakes: One is golden on both sides, one is brown on both
sides, and one is golden on one side and brown on the other. You withdraw one pancake, look at one side,
and see that it is brown. What is the probability that the other side is brown?
juri
5
Задача 1.38.
The inhabitants of an island tell truth one third of the time. They lie with the probability of 2/3.
On an occasion, after one of them made a statement, another fellow stepped forward and declared the
statement true.
What is the probability that it was indeed true?
Задача 1.39.
When the n’s dice are thrown at the one time, find the probability such that the sum of the cast is
n + 3?
Задача 1.40.
The Wimbledon mens singles tournament has 128 players. The first round pairings are completely
random, subject to the constraint that none of the top 32 players can be paired against each other. Two
competitors, Olivier Rochus, and his brother Christophe are competing, and neither are in the elite group
of 32 players. What is the probability that these brothers play in the first round (as actually occurred)?
Задача 1.41.
Первый автобус отходит от остановки в 5:00. Далее интервалы между автобусами равновероятно
составляют 10 или 15 минут, независимо от прошлых интервалов. Вася приходит на остановку в
5:42.
a) Какова ожидаемая длина интервала, в который он попадает?
б) Какова ожидаемая длина следующего интервала?
в) Пусть
t→∞
???
Задача 1.42.
Светофор 40 секунд горит зеленым светом, 3 секунды - желтым, 30 секунд - красным, затем цикл
повторяется. Петя подъезжает к светофору. На желтый свет Петя предпочитает остановиться.
а) Какова вероятность, что Петя сможет проехать сразу?
б) Какова средняя задержка Пети на светофоре?
а) Вася, стоящий рядом со светофором, смотрит на него в течение 3 секунд. Какова вероятность
того, что он увидит смену цвета?
Задача 1.43.
There are two ants on opposite corners of a cube. On each move, they can travel along an edge to an
adjacent vertex. What is the probability that they both return to their starting point after 4 moves?
Solution:
Answer=(
7 2
)
27
Задача 1.44.
На кубиках написаны числа от 1 до 100. Кубики свалены в кучу. Вася выбирает наугад из кучи по
очереди три кубика.
а) Какова вероятность, что полученные три числа будут идти в возрастающем порядке?
б) Какова вероятность, что полученные три числа будут идти в возрастающем порядке, если известно, что первое меньше последнего?
Задача 1.45.
На факультете
факультет,
b+
n
студентов. Из них наугад выбирают
a
человек. Через год
- приходит на факультет. Из них снова наугад выбирают
a.
b−
студентов покидает
Какова вероятность того,
что хотя бы одного выбирут два раза?
Задача 1.46. Легкомысленный член жюри [Mosteller]
В жюри из трех человек два члена независимо друг от друга принимают правильное решение с
вероятностью
p, а третий для вынесения решения бросает монету (окончательное решение выносится
большинством голосов). Жюри из одного человека выносит справедливое решение с вероятностью
adox
anie
6
p.
Какое из этих жюри выносит справедливое решение с большей вероятностью?
Задача 1.47. Simpson’s paradox
Тренер хочет отправить на соревнование самого сильного из своих спортсменов. Самым сильным
игроком тренер считает того, у кого больше всех шансов получить первое место, если бы соревнование проводилось среди своих. У тренера два спортсмена: А, постоянно набирающий 3 штрафных
очка при выполнении упражнения и Б, набирающий 1 штрафное очко с вероятностью 0,54 и 5
штрафных очков с вероятностью 0,46.
а) Кого отправит тренер на соревнования?
б) Кого отправит тренер на соревнования, если помимо А и Б у него тренируется спортсмен В,
набирающий 2 штрафных очка с вероятностью 0,56; 4 штрафных очка с вероятностью 0,22 и 6
штрафных очков с вероятностью 0,22
в) Мораль?
Задача 1.48.
n монеток, каждая из которых выпадает орлом с вероятностью p. В первом раунде Вася
У Васи есть
подкидывает все монетки, во втором раунде Вася подкидывает только те монетки, которые выпали
орлом в первом раунде. Пусть
i-ом
Ri
- количество монеток, подкидывавшихся и выпавших орлом во
раунде.
а) Каков закон распределения величины
b) Найдите
R2 ?
Corr(R1 , R2 ).
p→0
с) Как ведет себя корреляция при
и
p → 1?
Почему?
Задача 1.49.
ПустьX1 ,
X2 ,..., Xn
1, p
0, (1 − p)
Xi =
Пусть
k
- iid.
такая константа, что
2k ≥ n.
Найдите вероятность того, что самая длинная серия из единиц имеет длину
[?]:
Что делать при
k.
2k < n?
Задача 1.50.
Пусть существует всего два момента времени,
t обозначим Bt
10 phigh = 0, 7
2 plow = 0, 3
момент времени
S1 =
(bond) и
St
t = 0
и
t = 1.
Cтоимости облигаций и акций в
(share). Известно, что
B0 = 1, B1 = 1.1, S0 = 5,
Индивид может покупать акции и облигации по указанным ценам без ограничений. Например,
можно купить минус одну акцию, это означает, что в
t=0
индивид получает 5 рублей, а в
t = 1,
в
зависимости от состояния природы, должен заплатить 10 рублей или 2 рубля.
a) Чему равна безрисковая процентная ставка за период?
б) Найдите дисконтированные математические ожидания будущих цен акций и облигаций. Совпадают ли они с ценами нулевого периода?
в) Найдите такие вероятности
qhigh
и
qlow ,
чтобы дисконтированное математическое ожидание
будущих цен совпало с ценами нулевого периода.
г) Индивиду предлагают купить некий актив, который приносит 8 рублей в состоянии мира
и 11 рублей в состоянии мира
вероятностей
p
ωlow .
ωhigh
Посчитайте ожидание стоимости этого актива с помощью
и с помощью вероятностей
q.
Придумайте такую комбинацию акций и облигаций,
которая в будущем приносит 8 и 11 рублей соответственно, и найдите ее стоимость.
Задача 1.51.
X1 и X 2
P (min {X1 , X2 } = 2)
Игральный кубик подбрасывается два раза. Пусть
вероятности
P (min {X1 , X2 } = 4)
и
- результаты подбрасывания. Найдите
hird
pole
7
Задача 1.52.
17 заключенных, пять камер. Какова вероятность, что Петя и Вася сидят в одной камере?
Задача 1.53.
На десяти карточках написаны числа от 1 до 9. Число 8 фигурирует два раза, остальные числа - по
одному разу. Карточки извлекают в случайном порядке.
Какова вероятность того, что девятка появится позже обеих восьмерок?
Задача 1.54.
Jenny and Alex flip n fair coins each. What is the probability that they get the same number of tails?
2n
n
Answer: C2n /2
Solution: из 2n подбрасываний выберем n. Выбранные в зоне Jenny соответствуют невыбранным в
зоне Alex.
Addition:
Пусть
an =
√
n · pn ,
где
pn
- найденная вероятность. Найдите
Через формулу Стирлинга?
lim an
1/sqrtπ
Задача 1.55.
Кость подбрасывается два раза. Пусть
X
и
Y
- результаты подбрасываний. Найдите
E(|X − Y |).
Задача 1.56.
Suppose you are given a random number generator, which draws samples from an uniform distribution
between 0 and 1.
The question is: how many samples you have to draw, so that you are 95% sure that at least 1 sample lies
between 0.70 and 0.72?
Задача 1.57.
В турнире участвую 8 человек, разных по силе. Более сильный побеждает более слабого. Проигравший выбывает, победитель выходит в следующий тур.
Какова вероятность того, что
i-ый
по силе игрок дойдет до финала?
Задача 1.58.
A bag contains a total of N balls either blue or red. If 5 balls are chosen from the bag the prob all of them
being blue is 1/2. What are the values of N for which this is possible?
Задача 1.59.
Each of two boxes contains both black and white marbles, and the total number of marbles in the two
boxes is 25. One marble is taken out of each box randomly. The probability that both marbles are black
27
is
. What is the probability that both marbles are white?
50
Задача 1.60. [Баврин, Фрибус, Старинные задачи]
Маша с подружкой гуляют в поле. Подружка предлагает погадать на суженого. Она зажимает в
руке шесть травинок так, чтобы концы травинок торчали сверху и снизу. Маша сначала связывает
эти травинки попарно между собой сверху, а затем и снизу (получается три завязывания сверху и
три завязывания снизу). Если при этом все шесть травинок окажутся связанными в одно кольцо,
то это означает, что Маша в текущем году выйдет замуж.
Какие шансы у Маши?
Комментарий: будем считать, что завязывание травинок в «трилистник», «восьмерку» и прочие
нетривиальные узлы также означает замужество.
Задача 1.61. Геометрическое распределение.
N - число подбрасываний.
P (N > 30|N > 20), E(N )
Кубик подбрасывают до первого выпадения шестерки. С.в.
а) Найдите
P (N = 6), P (N = k), P (N > 10)
и
obki
za 6
8
б) Найдите
E( N1 )
Задача 1.62. Биномиальное распределение.
Кубик подбрасывают 5 раз. Пусть
P (N = k)
и
N
- количество выпадений шестерки. Найдите
P (N = 3),
P (N > 4|N > 3), E(N ).
Задача 1.63. Максимальная вероятность для биномиального распределения.
Пусть
X
распределена биномиально. Общее число экспериментов равно
n,
вероятность успеха в
p.
P (X = k)/P (X = k − 1)
При каких k дробь P (X = k)/P (X = k − 1) будет не меньше 1?
Каким должно быть k чтобы P (X = k) была максимальной?
отдельном испытании равна
а) Найдите
б)
в)
Задача 1.64.
Известно, что предварительно зарезервированный билет на автобус дальнего следования выкупается с вероятностью 0,9. В обычном автобусе 18 мест, в микроавтобусе 9 мест. Компания «Микро»,
перевозящая людей в микроавтобусах, допускает резервирование 10 билетов на один микроавтобус.
Компания «Макро», перевозящая людей в обычных автобусах допускает резервирование 20 мест на
один автобус.
У какой компании больше вероятность оказаться в ситуации нехватки мест?
Задача 1.65.
The psychologist Tversky and his colleagues say that about four out of five people will answer (a) to the
following question:
A certain town is served by two hospitals. In the larger hospital about 45 babies are born each day, and
in the smaller hospital 15 babies are born each day. Although the overall proportion of boys is about 50
percent, the actual proportion at either hospital may be more or less than 50 percent on any day. At the
end of a year, which hospital will have the greater number of days on which more than 60 percent of the
babies born were boys?
(a) the large hospital (b) the small hospital (c) neither (about the same).
Дайте верный ответ и попытайтесь объяснить, почему большинство людей ошибается при ответе на
этот вопрос.
Задача 1.66.
Из коробки с 4 синими и 5 зелеными шарами достают 3 шара. Пусть
синих и зеленых шаров. Найдите
B
и
G - количество извлеченных
E(B), E(G), E(B · G), E(B − G).
Задача 1.67.
Вася играет в компьютерную игру - «стрелялку-бродилку». По сюжету ему нужно убить 60 монстров. На один выстрел уходит ровно 1 минута. Вероятность убить монстра с одного выстрела равна
0,25. Количество выстрелов не ограничено.
а) Сколько времени в среднем Вася тратит на одного монстра?
б) Найдите дисперсию этого времени.
в) Какова вероятность того, что Вася закончит игру меньше, чем за 3 часа?
Задача 1.68.
Кость подбрасывается 3 раза. Размер ставки - 1 рубль. Если шестерка не выпадает ни разу, то ставка
проиграна, если шестерка выпадает хотя бы один раз, то ставка возвращается, плюс выплачивается
выигрыш по 1 рублю за каждую шестерку. Найдите стоимость этой лотереи.
Задача 1.69.
Из колоды в 52 карты извлекаются две. Пусть
X
- количество тузов. Найдите закон распределения
uzei
game
9
X , E(X), V ar(X).
Задача 1.70.
Иська пригласил трех друзей навестить его. Каждый из них появится независимо от другого с
вероятностью
0, 9, 0, 7
и
0, 5
а) Рассчитайте вероятности
б) Найдите
E(N )
и
соответственно. Пусть N - количество пришедших
P (N = 0), P (N = 1), P (N = 2) и P (N = 3)
гостей.
V ar(N )
Задача 1.71. [Von Neumann. Что делать, если монетка неправильная?]
Имеется «несправедливая» монетка, выпадающая гербом с вероятностью. Под раундом будем подразумевать двукратное подбрасывание монеты. Проводим первый раунд. Если результат раунда ГР (сначала герб, затем решетка), то считаем, что выиграл первый игрок. Если результат раунда
- РГ, то считаем, что выиграл второй игрок. Если результат раунда - ГГ или РР, то проводим
еще один раунд. И так далее, пока либо не определится победитель, либо количество раундов не
достигнет числа
n.
а) Найдите вероятности «ничьей», выигрыша первого игрока, выигрыша второго игрока в зависимости от
n.
Найдите пределы этих вероятностей при
n → +∞.
б) Как с помощью неправильной монетки сымитировать правильную?
Задача 1.72. Grimmett, [экзамен 1858 года St John’s College, Cambridge]
A large quantity of pebbles lies scattered uniformly over a circular field; compare the labour of collecting
them on by one: (i) at the center O of the field, (ii) at a point A on the circumference.
Задача 1.73. Parrondo’s game
p игру, в которой игрок выигрывает 1
p и проигрывает один рубль с вероятностью (1 − p).
Игра A - это рублевая игра с вероятностью 0,45.
Игра B состоит в следующем: если сумма в твоем кошельке делится на три,
Назовем рублевой игрой с вероятностью
рубль с вероятно-
стью
то ты играешь в
рублевую игру с вероятностью 0,05; если же сумма в твоем кошельке не делится на три, то ты
играешь в рублевую игру с вероятностью 0,7.
Что будет происходить с ожидаемым благосостоянием игрока, если он:
A?
b) Будет постоянно играть в игру B ?
с) Будет постоянно играть A или B с
а) Будет постоянно играть в игру
вероятностью по 0,5?
d) Придумайте «лохотрон» для интеллектуалов
Задача 1.74. Parrondo - var
A much simpler example is dealing cards from a well-shuffled deck. Suppose I get $14 if two cards in a row
match in rank (two 2’s or two Kings for examples), and pay $1 if they don’t. The chance of two cards in
a row matching is 1/17, so I pay $16 for each $14 I win.
Now I play the same game, alternating the deal between two decks. Now the chance of two successive cards
matching is 1/13, so I pay $12 for every $14 I win.
Each game individually loses money, but alternate them and you win money. Eureka! We’re all rich.
source: wilmott
Задача 1.75. Триэль
Три гусара
A, B
и
C
стреляются за прекрасную даму. Стреляют они по очереди (A-B -C -A-B -C -...),
каждый стреляет в противника по своему выбору.
A
попадает с вероятностью 0.1,
Триэль продолжается до тех пор, пока в живых не останется только один.
a) Какой должна быть стратегия ?
b) У кого какие шансы на победу?
B
- 0.5,
C
- 0.9.
10
Задача 1.76. Триэль-2
A, B
Три гусара
C
и
стреляются за прекрасную даму. Стреляют они одновременно, каждый
A
стреляет в противника по своему выбору.
попадает с вероятностью 0.1,
B
- 0.5,
C
- 0.9. Триэль
продолжается до тех пор, пока в живых не останется только один или никого.
a) Какой должна быть стратегия ?
b) У кого какие шансы на прекрасную даму?
Задача 1.77.
В забеге участвует 12 лошадей. Каждый из 10 зрителей пытается составить свой прогноз для трех
призовых мест. Какова вероятность того, что хотя бы один из них окажется прав?
Задача 1.78.
We throw 3 dices one by one. What is the probability that we obtain 3 points in strictly increasing order?
Задача 1.79.
Встретились 6 друзей. Каждый дарит подарок одному из других 5 человек.
Какова вероятность того, что найдется хотя бы одна пара человек, которая вручит подарки друг
другу?
Задача 1.80.
Число
x выбирается равномерно на отрезке [0; 1]. Затем случайно выбираются числа из отрезка [0; 1]
x.
до тех пор, пока не появится число больше
а) Сколько в среднем потребуется попыток?
б) Сколько в среднем потребуется попыток, если
в) Сколько в среднем потребуется попыток,
сти
p(t) = 2(1 − t)
для
x
x
выбирается равномерно на отрезке
[0; r], r < 1?
выбирается не равномерно, а имеет функцию плотно-
t ∈ [0; 1]?
Задача 1.81.
Кубик подбрасывается 3 раза. Какова вероятность того, что сумма первых двух подбрасываний
будет больше третьего?
Задача 1.82.
Случайные величины
X, Y ,
и
Z
независимы и равномерны на
[0; 1].
Какова вероятность того, что
X + Y > Z?
Задача 1.83.
Есть
N
монеток. Каждая из них может быть фальшивой с вероятностью
p.
Известно, сколько весят
настоящие. Известно, что фальшивые весят меньше чем настоящие. Каждая фальшивая может
иметь свое отклонение от правильного веса. Задача - определить, является ли фальшивой каждая
монета. Предлагается следующий способ:
Разбить монеты на группы по
n
монет в каждой группе. Взвесить каждую группу. Если вес группы
совпадает с эталонным, то вся группа признается настоящей. Если вес группы меньше эталонного,
то каждая монеты из группы взвешивается отдельно.
Предположим, что
a) Найдите
N
делится на
n.
Пусть
X
- требуемое число взвешиваний.
E(X)
b) При каком условии на
p
и
n
предложенный способ более эффективен чем взвешивание каждой
монеты?
c) Исследуйте поведение функции
E(X)/N
от
n
(есть ли минимум? максимум? и т.д.)
Задача 1.84.
В коробке лежат три монеты, достоинством в 10, 1 и 5 копеек соответственно. Они извлекаются в
случайном порядке. Пусть
а) Верно ли, что
X1
и
X3
X1 , X2
и
X3
- достоинства монет в порядке их появления из коробки.
одинаково распределены?
11
X1
E(X2 )
б) Верно ли, что
в) Найдите
и
г) Найдите дисперсию
X3
независимы?
X̄2 =
X1 +X2
.
2
Задача 1.85.
2 couples and a single person are to be randomly placed in 5 seats in a row. What is the probability that
no person that belongs to one of the couples sits next to his/her pair?
Задача 1.86. Easy
X
V ar(X).
Пусть
- сумма очков, выпавших в результате двукратного подбрасывания кубика. Найдите
E(X),
Задача 1.87. «Масть» при игре в бридж [Mosteller]
Часто приходится слышать, что некто при игре в бридж получил на руки 13 пик. Какова вероятность, при условии, что карты хорошо перетасованы, получить 13 карт одной масти? (Каждый из
четырех игроков в бридж получает 13 карт из колоды в 52 карты.)
Задача 1.88. Задача Банаха (Banach’s matchbox problem)
У Маши есть две коробки в каждой из которых осталось по
n
конфет. Когда Маша хочет конфету
она выбирает наугад одну из коробок и берет конфету оттуда. Рано или поздно Маша впервые
откроет пустую коробку. В этот момент другая коробка содержит некоторое количество конфет.
ur вероятность того, что в другой коробке ровно r конфет.
a) Найдите ur .
b) Найдите вероятности vr того, что в тот момент, когда из одной коробки возьмут
конфету (она только станет пустой!), в другой будет находится ровно r конфет.
Обозначим
последнюю
c) Найдите вероятность того, что коробка, которая была опустошена раньше, не будет первой
коробкой открытой пустой.
Solutions
а) Последняя попытка взять конфету - из пустой коробки. Назовем эту коробку А. Из предыдущих
n
C2n−r
n + (n − r) конфет n приходятся на коробку А. Вероятности по 21 . Получаем: ur = 2n+(n−r)
Задача 1.89.
У диплоидных организмов наследственные характеристики определяются парой генов. Вспомним
знакомые нам с 9-го класса горошины чешского монаха Менделя. Ген, определяющий форму горошины, имеет две аллели: «А» (гладкая) и «а» (морщинистая). «А» доминирует «а». В популяции
бесконечное количество организмов. Родители каждого потомка определяются случайным образом.
Одна аллель потомка выбирается наугад из аллелей матери, другая - из аллелей отца. Начальное
распределение генотипов имеет вид: «АА» - 30%, «Аа» - 60%, «аа» - 10%.
а) Каким будет распределение генотипов в
n-ом
поколении?
б) Заметив закономерность, сформулируйте и докажите теорему Харди-Вайнберга для произвольного начального распределения генотипов.
Задача 1.90.
У диплоидных организмов наследственные характеристики определяются парой генов. Некий ген,
сцепленный с полом, имеет две аллели: «А» и «а». Т.е. девочка может иметь один из трех генотипов
(АА, Аа, аа), а мальчик - только два (А и а; хромосома, определяющая мужской пол, короче и не
содержит нужного участка). От мамы ребенок наследует одну из двух аллелей (равновероятно),
а от отца либо наследует (тогда получается девочка), либо нет (тогда получается мальчик). «А»
доминирует «а». В популяции бесконечное количество организмов. Родители каждого потомка
определяются случайным образом.
а) Верно ли, что численность генотипов стабилизируется со временем?
б) Известно, что дальтонизм является признаком, сцепленным с полом. Догадавшись, является
ли он рецессивным или доминантным, определите, среди кого (мужчин или женщин) дальтоников
12
больше?
/проверить биологию/
Задача 1.91. (продумать)
Наследование группы крови контролируется аутосомным геном. Локус этого гена обозначают буквой I, а три его аллеля буквами А, В и 0. Аллели А и В доминантны в одинаковой степени, а аллель
0 рецессивен по отношению к ним обоим. Существует четыре группы крови. Им соответствуют следующие генотипы:
Первая (I) 00
Вторая (II) АА ; А0
Третья (III) ВВ ; В0
Четвертая (IV) АВ
Наследование резус-фактора кодируется тремя парами генов и происходит независимо от наследования группы крови. Наиболее значимый ген обозначается латинской буквой D. Он может быть
доминантным - D, либо рецессивным - d. Генотип резус-положительного человека может быть
гомозиготным - DD, либо гетерозиготным - Dd. Генотип резус-отрицательного человека может быть
- dd.
Гемолитическая болезнь плода и новорожденного это состояние, возникающее в результате несовместимости крови матери и плода по некоторым антигенам. Наиболее часто гемолитическая болезнь
новорожденного развивается вследствие резус-конфликта. При этом у беременной женщины резусотрицательная кровь, а у плода резус-положительная. Во время беременности резус-фактор с
эритроцитами резус-положительного плода попадает в кровь резус-отрицательной матери и вызывает в ее крови образование антител к резус-фактору (безвредных для нее, но вызывающих
разрушение эритроцитов плода). Распад эритроцитов приводит к повреждению печени, почек,
головного мозга плода, развитию гемолитической болезни плода и новорожденного. В большинстве
случаев заболевание быстро развивается после рождения, чему способствует поступление большого
количества антител в кровь ребенка при нарушении целостности сосудов плаценты.
Процесс иммунизации беременной женщины начинается с момента образования антигенов в эритроцитах плода. Поскольку антигены системы резус содержаться в крови плода с 9-10й недели
беременности, а групповые антигены - с 5-6й недели, то в некоторых случаях возможна ранняя сенсибилизация организма матери. Проникновению антигенов в материнский кровоток способствуют
инфекционные факторы, повышающие проницаемость плаценты, мелкие травмы, кровоизлияния и
другие повреждения плаценты. Как правило, первая беременность у резус-отрицательной женщины
при отсутствии в прошлом сенсибилизации организма протекает без осложнений. Сенсибилизация организма резус-отрицательной женщины возможна при переливаниях несовместимой крови
(проводимых даже в раннем детском возрасте), при беременностях и родах (если у плода резусположительная кровь), после абортов, выкидышей, операций по поводу внематочной беременности.
По данным литературы после первой беременности иммунизация возникает у 10% женщин. Если
женщина с резус-отрицательной кровью избежала резус-иммунизации после первой беременности,
то при последующей беременности резус-положительным плодом вероятность иммунизации вновь
составляет 10%. Поэтому после любого прерывания беременности у женщины с резус-отрицательной
кровью с профилактической целью необходимо введение антирезус-иммуноглобулина. В течении
беременности у женщины с резус-отрицательной кровью обязательно необходимо определение титра
резус-антител в крови в динамике.
Задача 1.92. [Парадокс голосования]
Пусть
X, Y , Z
- дискретные случайные величины, их значения попарно различны с вероятностью
1. Докажите, что
min {P (X > Y ), P (Y > Z), P (Z > X)} ≤
2
. Приведите пример, при котором эта граница точно
3
olna
chek
auty
blem
13
достигается.
Задача 1.93. [Williams, 4.3]
Пусть
Xi
P (Xi = Xj ) = 0. Обозначим Ek - событие состоящее в том, что Xk оказалась
больше, чем все предыдущие Xi (i < k ). Для определенности будем считать, что
- iid, такие что
«рекордом», т.е.
E1 = Ω.
a) Найдите
P (Ek )
Ek
b) Верно ли, что
независимы?
c) Какова вероятность того, что второй рекорд будет зафиксирован в
n-ый
момент времени?
d) Сколько в среднем времени пройдет до второго рекорда?
Задача 1.94.
На карточках написаны числа от 1 до 100. В левую руку Маша берет одну карточку, в правую -
k
карточек. Какова вероятность того, что карточка в левой руке окажется больше каждой карточки
из правой руки?
Задача 1.95. Спящая красавица
Спящая красавица согласилась принять участие в научном эксперименте. В воскресенье ее специально уколют веретеном. Как только она заснет будет подброшена правильная монетка. Если
монетка выпадет орлом, то спящую красавицу разбудят в понедельник и спросят о том, как выпала
монетка. Если монетка выпадет решкой, то спящую царевну разбудят в понедельник, спросят о
монетке, снова уколют веретеном, разбудят во вторник и снова спросят о монетке. Укол веретена
вызывает легкую амнезию, и красавица не сможет определить, просыпается ли она в первый раз
или во второй.
Красавица только что проснулась.
а) Какова вероятность того, что сегодня понедельник?
б) Как следует отвечать красавице, если за каждый верный ответ ей дарят молодильное яблоко?
в) Как следует отвечать красавице, если за неверный ответ ее тут же превращают в тыкву?
[Осторожно!
Некорректные вопросы!]
Задача 1.96. Monty Hell problem
Сказка: Ежедневно Кощей Бессмертный получает пенсию в размере 10 золотых монет. С начала
пенсионного возраста он аккуратно нумерует каждую полученную монету и кладет ее в Сундук.
Ночью Мышка-Норушка крадет одну золотую монету из Сундука.
а) Какова вероятность того, что
i-ая
монета когда-либо исчезнет из Сундука?
б) Какова вероятность того, что хотя бы одна монета пролежит в Сундуке бесконечно долго?
в) Дни сокращаются в продолжительности (каждый последующий - в два раза короче, чем предыдущий). Сколько монет будет в сундуке в конце времени?
−x
Подсказка: (1 − x) ≤ e
Comment: А где надсказка?
Задача 1.97. Птички на проводе-1
На провод, отрезок
Yn+1
[0; 1],
равномерно и независимо друг от друга садятся
n
птичек. Пусть
- расстояния между птичками.
а) Найдите функцию плотности
б) Верно ли, что все
в) Верно ли, что все
Найдите
Cov(Yi , Yj )
Yi
Yi
Y1 ,
расстояния от левого столба до первой птички
одинаково распределены?
независимы?
(вроде бы ковариации равны?)
г) Как распределена величина
n · Y1
при больших
Solution:
Xi - координата i-ой птички
P (Y1 ≤ t) = 1 − P (min{Xi } > t) = 1 − (1 − t)n
n?
Почему?
Y1 ,...,
olog
rzia
14
lim P (nY1 ≤ t) = lim 1 − (1 − nt )n = 1 − e−t
Экспоненциально, с параметром λ = 1
Задача 1.98. Птички на проводе-2 [Marcin Kuczma]
На провод, отрезок
[0; 1], равномерно и независимо друг от друга садятся n птичек. Мы берем ведро
желтой краски и для каждой птички красим участок провода от нее до ближайшей к ней соседки.
Какая часть провода будет окрашена при больших
n?
Solution 1:
Пусть
n
велико, тогда
Yi
можно считать независимыми и
nYi
- экспоненциально распределенным.
Не красятся только «большие» интервалы, т.е. интервалы, чья длина больше, чем каждого из двух
соседних интервалов. «Больших» интервалов примерно треть.
E(B) = E(max{Y1 , Y2 , Y3 }
7
δ = 1 − E(B) n3 = 18
Находим
Задача 1.99.
В коробке находится четыре внешне одинаковых лампочки. Две из лампочек исправны, две - нет.
Лампочки извлекают из коробки по одной до тех пор, пока не будут извлечены обе исправные.
а) Какова вероятность того, что опыт закончится извлечением трех лампочек?
б) Каково ожидаемое количество извлеченных лампочек?
Задача 1.100. Спелестолог и батарейки
У спелестолога в каменоломнях сели батарейки в налобном фонаре и он оказался в абсолютной
темноте. В рюкзаке у него 8 батареек, 5 новых и 3 старых. Для работы фонаря требуется две новых
батарейки. Спелестолог вытаскивает из рюкзака две батарейки наугад и вставляет их в фонарь.
Если фонарь не начинает работать, то спелестолог откладывает эти две батарейки и пробует следующие и т.д.
а) Сколько попыток в среднем потребуется?
б) Какая попытка скорее всего будет первой удачной?
c) Творческая часть. Поиграйтесь с задачей. Случайна ли прогрессия в ответе? Сравните с 6
новых+4старых и т.д.
Solution:
N
1
2
3
4
5
4
3
2
Prob
14
14
14
14
Solution для 6=4+2
C2
P (N = 1) = C42 = 6/15
6
3·1
P (N = 3) = 4·2
= 4/15
C62 C52
P (N = 2) = 5/15
E(N ) = 28/15
Задача 1.101.
Два ферзя (черный и белый) ставятся наугад на шахматную доску.
а) Какова вероятность того, что они будут «бить» друг друга?
б) К чему стремится эта вероятность для шахматной доски со стороной, стремящейся к бесконечности?
Задача 1.102.
A0 , A1 и A2 - несовместны и вместе покрывают
p0 = P (A1 ∪ A2 ), p1 = P (A0 ∪ A2 ), p2 = P (A0 ∪ A1 ).
Перечислите все условия, которым удовлетворяют p0 , p1 , p2 .
Пусть события
все
Ω.
Обозначим
Задача 1.103.
Найдите вероятность того, что произойдет ровно одно событие из
A и B , если P (A) = 0.3, P (B) = 0.2,
15
P (A ∩ B) = 0.1
Задача 1.104.
На день рождения к Васе пришли две Маши, два Саши, Петя и Коля. Все вместе с Васей сели за
круглый стол. Какова вероятность, что Вася окажется между двумя тезками?
Слева должен сесть тот, у кого есть тезка.
Справа должен сесть его парный.
Итого:
p1 = 4/6
p2 = 1/5
p = p1 · p2 = 2/15
Задача 1.105.
At a bus stop you can take bus #1 and bus #2. Bus #1 passes 10 minutes after bus #2 has passed
whereas bus #2 passes 20 mins after bus #1 has passed.
What is the average waiting time to get on a bus at that bus stop?
Source: wilmott, 26, 55617
25
Solution:
3
Задача 1.106.
Петя подкидывает монетку четыре раза. Если монетка выпадает орлом, то он кладет в мешок
черный шар, если решкой - белый. Петя не знает, как выпадала монетка, и достает два шара из
мешка наугад. Первый шар черного цвета. Какова вероятность того, что второй будет белым?
Задача 1.107.
A wooden cube that measures 3 cm along each edge is painted red. The painted cube is then cut into
1-cm cubes as shown above.
a) If I choose one of the small cubes at random and toss it in the air, what is the probability that it will
land red-painted side up?
b) If I tossed all the small cubes in the air, so that they landed randomly on the table, how many cubes
should I expect to land with a painted face up?
c) If I put all the small cubes in a bag and randomly draw out 3, what is the probability that at least 3
faces on the cubes I choose are painted red?
d) If I put the small cubes in a bag and randomly draw out 3, what is the probability that exactly 3 of
the faces are painted red?
e) Invent a new question!
Answers: a)
1/3,
b)
9
Source: http://letsplaymath.wordpress.com/2007/07/25/puzzle-random-blocks/
Задача 1.108.
Случайным
P (X = n) =
образом
2−n .
выбирается
натуральное
число
X.
Вероятность
выбора
числа
n
равна
а) Какова вероятность того, что будет выбрано четное число? Нечетное число? Число больше 5?
Число от 3 до 11?
б) Пусть независимо друг от друга выбираются
c
чисел. Пусть
Kc
- количество невыбранных чисел
на отрезке от одного до наибольшего выбранного числа. Найдите P (Kc = k)
−(k+1)
Answer: P (Kc = k) = 2
вне зависимости от c. Похоже, что для геометрического распределения
с произвольным
p
появляется зависимость от
c.
Source b: AMM E3061 by T. Ferguson and C. Melolidakis
Задача 1.109.
В уездном городе
N
два родильных дома, в одном ежедневно рождается 50 человек, в другом - 100
человек. В каков роддоме чаще рождается одинаковое количество мальчиков и девочек?
Ответ: в маленьком
16
Задача 1.110.
Равновероятно выбирается три числа от 1 до 20. Какова вероятность того, что третье попадет между
двух первых?
Anwser:
57/200 = 0.285
Задача 1.111.
let you choose a sequence infinite of integer between 1 and 10, what is the possibility that your sequence
doesnt have any «1»?
Answer: zero
Задача 1.112.
Вася наугад выбирает два разных натуральных числа от 1 до 4.
а) Какова вероятность того, что будет выбрано число 3?
б) Какова вероятность того, что сумма выбранных чисел будет четная?
в) Каково математическое ожидание суммы выбранных чисел?
1
1
Ответы: pa = , pb = , E(S) = 5
2
3
Задача 1.113.
Охотник, имеющий 4 патрона, стреляет по дичи до первого попадания или до израсходования всех
патронов. Вероятность попадания при первом выстреле равна 0.6, при каждом последующем уменьшается на 0.1. Найдите:
a) Закон распределения числа патронов, израсходованных охотником
б) Математическое ожидание и дисперсию этой случайной величины
Решение:
xi
1
2
3
4
P (X = xi ) 0, 6 (1 − 0, 6) · 0, 5 (1 − 0, 6) · (1 − 0, 5) · 0, 4 1 − p1 − p2 − p3
xi
1
2
3
4
P (X = xi ) 0, 6 0, 2 0, 08 0, 12
E(X) = 1, 7, V ar(X) ≈ 1, 08
Задача 1.114.
There are three coins in a box. These coins when flipped, will come up heads with respective probabilities
0.3, 0.5, 0.7.
then flipped
а) Найдите
A coin is randomly selected (meaning uniform distribution!) from among these three and
10 times. Let N
Pr[N = 0]
be the number of heads obtained on the first ten flips.
б)If you win $1 each time a head appears and you lose $1 each time a tail appears, is this a fair game?
Explain.
Задача 1.115. Five distinct numbers are randomly distributed to players numbered 1 through 5. Whenever
two players compare their numbers, the one with the higher one is declared the winner. Initially, players
1 and 2 compare their numbers; the winner then compares with player 3. Let
times player 1 is a winner.
Find the distribution of
X
2. Комбинаторика
Задача 2.1.
На столе есть следующие предметы:
4 отличающихся друг от друга чашки;
4 одинаковых граненых стакана;
10 одинаковых кусков сахара;
7 соломинок разных цветов.
Сколькими способами можно разложить:
X
denote the number of
d001
d002
d003
d004
odir
17
а) сахар по чашкам;
б) сахар по стаканам;
в) соломинки по чашкам;
г) соломинки по стаканам;
Как изменятся ответы, если требуется, чтобы пустых емкостей не оставалось?
Задача 2.2.
Сколькими способами можно разложить
k
кусков сахара по
n
различающимся чашкам?
Подсказка: ответ - всего лишь биномиальный коэффициент
n−1
Ответ: Ck+n−1
Задача 2.3. Генуэзская лотерея (задача Леонарда Эйлера)
Из 90 чисел выбираются 5 наугад. Назовем серией последовательность из нескольких чисел, идущих
подряд. Например, если выпали числа 23, 24, 77, 78 и 79 (неважно в каком порядке), то есть две
серии (23-24, 77-78-79).
Определите вероятность того, что будет ровно
k
серий.
Комментарий: сама лотерея возникла в 17 веке
Задача 2.4.
В клубе 25 человек.
a) Сколькими способами можно выбрать комитет из 4-х человек?
б) Сколькими способами можно выбрать руководство, состоящее из директора, зама и кассира?
Задача 2.5.
Сколькими способами можно расставить 5 человек в очередь?
Задача 2.6.
Сколькими способами можно покрасить 12 комнат, если требуется 4 покрасить желтым цветом, 5 голубым и 3 - зеленым?
Задача 2.7.
Шесть студентов, три юноши и три девушки, стоят в очереди за пирожками в случайном порядке.
Какова вероятность того, что юноши и девушки чередуются?
Задача 2.8. Покер
Выбирается 5 карт из колоды (52 карты без джокеров, достоинством от 2 до туза, всего 13 достоинств). Рассчитайте вероятности комбинаций:
Pair (Пара) - Две карты одного достоинства
Two pairs (Две пары) - Две карты одного достоинства и две другого
Three of Kind (Тройка) - Три карты одного достоинства (две другие - разного достоинства)
Straight (Стрит) - Пять последовательных карт не обязательно одной масти
Flush (Масть) - Все карты одной масти
Full House (Фулл) - Три карты одного достоинства и две другого
Four of Kind (Каре) - Четыре карты одного достоинства
Straight Flush (Стрит-Флэш) - Пять последовательных карт одной масти
Royal Flush (Роял-Флэш) - Пять последовательных карт одной масти начиная с туза
[Более
]
слабая комбинация не содержит в себе более сильной
Задача 2.9. «Вилкодыр»
Есть
n
дырочек, расположенных в линию, на расстоянии в 1 см. У каждой вилки два штырька на
расстоянии в 1 см.
а) Сколькими способами можно воткнуть
k
одинаковых вилок?
riad
18
б) Как изменится ответ, если дырочки расположены по окружности?
Задача 2.10.
В ряду
n лампочек. Из них надо зажечь 8, так чтобы было три серии (по 2, 3 и 3 горящих лампочки).
Сколькими способами это можно сделать?
Задача 2.11.
Вася играет в преферанс. Он взял прикуп, снес две карты и выбрал козыря. У Васи на руках четыре
козыря. Какова вероятность, что оставшиеся четыре козыря разделились 4:0, 3:1, 2:2?
Для тех, кто не знает, как играть в преферанс: 32 карты, из которых 8 будущих козырей, раздаются
по 10 между 3 игроками, еще две кладутся в прикуп.
Задача 2.12.
а) Перетасована колода в 52 карты. Какова вероятность того, что какие-нибудь туз и король будут
лежать рядом?
б) Какова вероятность того, что какой-нибудь туз будет лежать за каким-нибудь королем?
Задача 2.13.
Чему равна сумма
Cn0 − Cn1 + Cn2 − ...?
Ее применение к matching problem
Задача 2.14.
В линию выложено
n
предметов друг за другом. Сколькими способами можно выбрать
k
предметов
из линии, так чтобы не были выбраны соседние предметы?
k
Ответ: Cn−k+1
Решение 1: Отдельно рассмотрим два случая: самый правый предмет выбран и самый правый
предмет не выбран. В каждом случае склеиваем предмет и примыкающий к нему справа предмет
«разделитель».
Решение 2: Удалим
k − 1 предмет из линии. Из оставшихся предметов выберем k . Вернем удаленные
как «разделители».
Задача 2.15.
Given eight distinguishable rings, let
n
be the number of possible five-ring arrangements on the four
fingers (not the thumb) of one hand. The order of rings on each finger is significant, but it is not required
that each finger have a ring. source: AIME 2000, 5
Задача 2.16.
5 numbers are randomly picked from 90.
In your bet cards, you get to choose 5 numbers.
How many cards have you got to fill in, to guarantee that at least one of them has 4 right numbers?
Source: wilmott
the answer to the original problem (43,948,843 bet cards) was quoted already several times assuming that
positioning of right numbers is irrelevant:
- there is eactly one bet card choice with 5 right numbers and
- there are 5 x (90-5) = 425 bet card choices with exactly 4 right numbers.
Since the total number of different bet card choices is 90 over 5, we have to fill out (90 over 5) - 425 - 1
+1 = 43,948,843 bet cards to have at least 4 right numbers with prob 1.
Задача 2.17.
Где-то в начале 17 века Галилея попросили объяснить следующее: количество троек натуральных
чисел, дающих в сумме 9, такое же, как количество троек, дающих в сумме 10; но при трехкратном
подбрасывании кубика 9 в сумме выпадает реже, чем 10.
isla
atok
19
Дайте корректное объяснение
Задача 2.18.
Из 10 цифр (от 0 до 9) выбираются 3 наугад (возможны повторения). Обозначим числа (в порядке
появления):
X 1 , X2 , X 3 .
Какова вероятность того, что
X1 > X2 > X3 ?
Задача 2.19.
В контрольной 20 вопросов. Все ответы разные. Вася успел переписать у друга все верные ответы, но
не знает в каком порядке они идут. Отлично ставят ответившим верно на не менее чем 15 вопросов.
Какова вероятность того, что Вася получит отлично?
1
5
4
3
2
1
0
· (C42 + 3) + C20
· 2 + C20
+ C20
· 0 + C20
+ C20
p = 20!
· (C20
· (2C52 + 4))
Задача 2.20.
Сколько существует векторов (x1 , x2 , ..., xk ) таких, что 1 ≤ x1 < x2 < x3 < ... < xk ≤ n?
k
Ответ: Cn - просто выбираем k различных чисел из n, а числа всегда упорядочены :)
Задача 2.21.
There are k books of mine among n books. We put them in a shelf ramdomly. Which is the possibility
that there are p books of my who are placed continuously?
(at least, exactly)
source: aops, 192257
answer: ugly sum?
Задача 2.22. Корректоры очепяток
Вася замечает очепятку с вероятностью 0.7; Петя независимо от Васи замечает очепятку с вероятностью 0.8. В книге содержится 100 опечаток. Какова вероятность того, что Вася заметит 30
опечаток, Петя - 50, причем 14 опечаток будут замечены обоими корректорами?
100!
Ответ:
0.730 0.370 0.850 0.250
14!16!36!34!
Задача 2.23.
На каждой карточке вы можете отметить любые 5 чисел из 100. Сколько карточек нужно купить,
чтобы гарантированно угадать 3 числа из выпадающих в лотереи 7 чисел?
коммент: может быть громоздкие вычисления
Задача 2.24.
Сколькими способами можно поставить в очередь
a
мужчин и
b
женщин, так чтобы нигед двое
мужчин не стояли рядом?
3. Условная вероятность
Задача 3.1. randomized response technique
В анкете для чиновников включен скользкий вопрос: «Берете ли Вы взятки?». Чтобы стимулировать
чиновников отвечать правдиво используется следующий прием. Перед ответом на вопрос чиновник в
тайне от анкетирующего подкидывает специальную монетку, на гранях которой написано «правда»,
«ложь». Если монетка выпадает «правдой», то предлагается отвечать на вопрос правдиво, если
монетка выпадает на «ложь», то предлагается солгать. Таким образом ответ «да» не обязательно
означает, что чиновник берет взятки.
Допустим, что треть чиновников берут взятки, а монетка - неправильная и выпадает «правдой» с
вероятностью 0.2.
a) Какова вероятность того, что чиновник ответит «да»?
б) Какова вероятность того, что чиновник берет взятки, если он ответил «да»? Если ответил «нет»?
коммент: вставить построение несмещенной оценки?
20
Задача 3.2.
Пусть события
A
и
B
независимы, и
P (B) > 0.
Чему равна
P (A|B)?
Задача 3.3.
Из колоды в 52 карты извлекается одна карта наугад. Верно ли, что события «извлечен туз» и
«извлечена пика» являются независимыми?
Ответ: да
Задача 3.4.
Из колоды в 52 карты извлекаются по очереди две карты наугад. Верно ли, что события «первая
карта - туз» и «вторая карта - туз» являются независимыми?
Ответ: нет
Задача 3.5.
Известно, что
независимы и
P (A) = 0, 3, P (B) = 0, 4, P (C) = 0, 5.
P (B|C) = 0, 1. Найдите P (A ∪ B ∪ C).
События
A
и
B
несовместны, события
A
и
C
Задача 3.6.
Имеется три монетки. Две «правильных» и одна - с «орлами» по обеим сторонам. Петя выбирает
одну монетку наугад и подкидывает ее два раза. Оба раза выпадает «орел». Какова вероятность
того, что монетка «неправильная»?
Задача 3.7. [Айвазян, экзамен РЭШ]
Самолет упал либо в горах, либо на равнине. Вероятность того, что самолет упал в горах равна 0,75.
Для поиска пропавшего самолета выделено 10 вертолетов. Каждый вертолет можно использовать
только в одном месте. Как распределить имеющиеся вертолеты, если вероятность обнаружения
пропавшего самолета отдельно взятым вертолетом равна:
а) 0,95 (ясная погода)
б) 0,6 (пасмурно)
в) 0,1 (туман)
Задача 3.8.
На днях Левада-Центр опубликовал итоги опроса, согласно которым 2/3 россиян поддерживают
Путина и 2/5 россиян доверяют опросам Левада-Центра. Договоримся, что доверяющие опросам
всегда отвечают искренне, а недоверяющие могли соврать в ответе на любой вопрос или оба. Исходя
из этих данных, оцените реальную поддержку Путина россиянами.
source: лента ru-math
Задача 3.9.
Два охотника выстрелили в одну утку. Первый попадает с вероятностью 0,4, второй - с вероятностью 0,6. В утку попала ровно одна пуля. Какова вероятность того, что утка была убита первым
охотником?
0.4·0.4
p = 0.4·0.4+0.6·0.6
Задача 3.10.
=
4
13
С вероятностью 0,3 Вася оставил конспект в одной из 10 посещенных им сегодня аудиторий. Вася
осмотрел 7 из 10 аудиторий и конспекта в них не нашел.
а) Какова вероятность того, что конспект будет найден в следующей осматриваемой им аудитории?
б) Какова (условная) вероятность того, что конспект оставлен где-то в другом месте?
Задача 3.11.
Вася гоняет на мотоцикле по единичной окружности с центром в начале координат. В случайный
момент времени он останавливается. Пусть случайные величины X и Y - это Васины абсцисса
P (X > 21 ), P (X > 12 |Y < 21 ). Являются ли события
и ордината в момент остановки. Найдите
21
A = X > 21 и B = Y < 12 независимыми?
π
1
Подсказка: cos( ) =
, длина окружности l = 2πR
3
2
Задача 3.12.
Пусть
P (A) = 1/4, P (A|B) = 1/2
и
P (B|A) = 1/3.
Найдите
P (A ∩ B), P (B)
и
P (A ∪ B).
Задача 3.13.
1
Примерно 4% коров заражены «коровьим бешенством». Имеется тест, позволяющий с определенной
степени достоверности установить, заражено ли мясо прионом или нет. С вероятностью 0,9 зараженное мясо будет признано зараженным. «Чистое» мясо будет признано зараженным с вероятностью
0,1. Судя по тесту, эта партия мяса заражена. Какова вероятность того, что она действительно
заражена?
Задача 3.14.
Роме Протасевичу, искавшему со мной у Мутновского вулкана в тумане серую палатку...
Есть две темные комнаты, А и В. В одной из них сидит черная кошка. Первоначально предполагается, что вероятность нахождения кошки в комнате А равна
темной комнате (если она там есть) с одной попытки равна
попыток поиска кошки в комнате А и
b
α.
Вероятность найти черную кошку в
p. Допустим, что вы сделали a неудачных
B.
неудачных попыток в комнате
а) Чему равна условная вероятность нахождения кошки в комнате А?
b) При каком условии на
(a − b)
эта вероятность будет больше
0.5?
Задача 3.15.
Кубик подбрасывается два раза. Найдите вероятность получить сумму равную 8, если на первом
кубике выпало 3.
Задача 3.16.
i
. Из
10
коробки балы вытащена одна монетка наугад. Она выпала орлом. Какова вероятность того, что это
В коробке 10 пронумерованных монеток,
i-ая
монетка выпадает орлом с вероятностью
была пятая монетка?
1
11
Задача 3.17.
Вы играете две партии в шахматы против незнакомца. Равновероятно незнакомец может оказаться
новичком, любителем или профессионалом. Вероятности вашего выигрыша в отдельной партии,
соответственно, будут равны: 0,9; 0,5; 0,3.
а) Какова вероятность выиграть первую партию?
б) Какова вероятность выиграть вторую партию, если вы выиграли первую?
17
pa = 13 (0.9 + 0.5 + 0.3) = 30
,
115
1
2
2
2
pb = 3 (0.9 + 0.5 + 0.3 )/pa = 170
Задача 3.18.
В каких из перечисленных случаях вероятность наличия флэша больше, чем при полном отсутствии
информации:
а) первая карта из имеющихся - это туз
б) первая карта из имеющихся - это туз бубей
в) на руках имеется хотя бы один туз
г) на руках имеется туз бубей
Solution: (unverified, but no calculation) :
An arbitrary hand can have two aces but a flush hand can’t. The average number of aces that appear in
flush hands is the same as the average number of aces in arbitrary hands, but the aces are spread out
1Цифры
условные. Celui qui ne mange pas de bifsteak au cause de la vache folle - il est fou! Jolivaldt.
tion
22
more evenly for the flush hands, so set (3) contains a higher fraction of flushes.
Aces of spades, on the other hand, are spread out the same way over possible hands as they are over flush
hands, since there is only one of them in the deck. Whether or not a hand is flush is based solely on a
comparison between different cards in the hand, so looking at just one card is necessarily uninformative.
So the other sets contain the same fraction of flushes as the set of all possible hands.
Задача 3.19.
Подбрасывается правильный кубик. Узнав результат, игрок выбирает, подкидывать ли кубик второй
раз. Игрок получает сумму денег равную количеству очков при последнем подбрасывании.
а) Каков ожидаемый выигрыш игрока?
б) Каков ожидаемый выигрыш игрока, если максимальное количество подбрасываний равно трем,
а выигрышем считается результат последнего подбрасывания?
Задача 3.20.
Игрок получает 13 карт из колоды в 52 карты.
а) Какова вероятность того, что у него как минимум два туза, если известно, что у него есть хотя
бы один туз?
б) Какова вероятность того, что у него как минимум два туза, если известно, что у него есть туз
пик?
в) Объясните, почему эти две вероятности отличаются.
Задача 3.21.
N проживают 107
p = 10−7 .
В уездном городе
с вероятностью
человек. Каждый из них обладает редким даром ясновидения
а) Каково ожидаемое количество ясновидящих?
б) Известно, что Петя - ясновидящий, какова вероятность найти еще одного ясновидящего в городе
N?
Задача 3.22.
Ген карих глаз доминирует ген синих. Т.е. у носителя пары bb глаза синие, а у носителя пар BB и
Bb - карие. У диплоидных организмов (а мы такие :)) одна аллель наследуется от папы, а одна - от
мамы. В семье у кареглазых родителей два сына - кареглазый и синеглазый. Кареглазый женился
на синеглазой девушке. Какова вероятность рождения у них синеглазого ребенка?
Задача 3.23.
Предположим, что вероятности рождения мальчика и девочки равны, а пол первого и второго
ребенка независимы.
а) У тети Маши двое детей. Хотя бы один ребенок - мальчик. Какова вероятность того, что другой
ребенок - девочка?
б) У тети Маши двое детей. Тетя Маша наугад выбирает одного своего ребенка и посылает к тете
Оле, вернуть учебник по теории вероятностей. Это оказывается мальчик. Какова вероятность того,
что другой ребенок - девочка?
в) У тети Маши двое детей. Старший ребенок - мальчик. Какова вероятность того, что другой
ребенок - девочка?
Задача 3.24.
В урне 5 белых и 11 черных шаров. Два шара извлекаются по очереди. Какова вероятность того,
что второй шар будет черным? Какова вероятность того, что первый шар - белый, если известно,
что второй шар - черный?
Задача 3.25. Monty-hall (if you don’t know)
Вы играете в «Поле Чудес» и Вам предлагают «3 шкатулки». Назовем их a, b и c. В одной из трех
шкатулок лежит 1000 рублей (Введем соответственно события A, B и C, где A означает «деньги
23
лежат в шкатулке a»). Вы выбираете наугад одну из трех шкатулок.
Ведущий, который знает, где лежат деньги, убирает одну пустую шкатулку, не выбранную Вами
(среди двух не выбранных Вами обязательно есть пустая, если таковых две, то ведущий убирает
любую наугад). Допустим, Вы выбрали шкатулку b, а ведущий после этого убрал шкатулку c.
Найдите условную вероятность того, что приз лежит в выбранной Вами шкатулке. Имеет ли Вам
смысл изменить Ваш выбор?
solution 1:
Задача эквивалентна следующей: игрок выбирает шкатулку. Затем (она не открывается) игрок
выбирает оставить ее или взять обе другие. Очевидно, во втором случае шансы в два раза выше.
Solution 2:
Игрок не получает информации - вероятность не меняется. Лучше сменить выбор.
Альтернативный вариант условия-1
После того, как Вы выбрали шкатулку, ведущий открывает наугад одну из пустых шкатулок (при
этом он может открыть Вашу и разочаровать Вас). Допустим, Вы выбрали шкатулку b, а ведущий
после этого открыл шкатулку c. Найдите условную вероятность того, что приз лежит в выбранной
Вами шкатулке. Имеет ли Вам смысл изменить Ваш выбор?
Альтернативный вариант условия-2
После того, как Вы выбрали шкатулку, ведущий открывает наугад одну из оставшихся шкатулок
(при этом он может оказаться открытой шкатулка с деньгами). Допустим, Вы выбрали шкатулку
b, а ведущий после этого открыл шкатулку c и она оказалась пустой. Найдите условную вероятность того, что приз лежит в выбранной Вами шкатулке. Имеет ли Вам смысл изменить Ваш выбор?
Задача 3.26. multi-stage monty hall
Suppose there are four doors, one of which is a winner. The host says: «You point to one of the doors, and
then I will open one of the other non-winners. Then you decide whether to stick with your original pick or
switch to one of the remaining doors. Then I will open another (other than the current pick) non-winner.
You will then make your final decision by sticking with the door picked on the previous decision or by
switching to the only other remaining door» Optimal strategy?
Solution: stick-switch
source: cut-the-knot – Bhaskara Rao
Задача 3.27.
What is the probability that a random 2 digit base b number will be relativly prime with its digit reversal?
Comment: maybe many cases...
Задача 3.28.
В классе 28 человек, среди них 18 девочек. Класс построили в 4 ряда по 7 человек. Какова вероятность того, что рядом с Вовочкой будет стоять хотя бы одна девочка?
Comment: для Вовочки любая девушка - рядом :)
Задача 3.29.
Неправильную монетку, у которой «орел» выпадает с вероятностью
этом оказалось, что она выпала на «орла» 41 раз. При каком
p
p,
подбрасывали 50 раз. При
вероятность этого события будет
максимальной?
Задача 3.30.
Игральный кубик подбрасывается 100 раз. Найдите ожидаемую сумму очков, дисперсию суммы,
стандартное отклонение суммы.
Задача 3.31.
В школе три девятых класса, «А», «Б» и «В», одинаковые по численности. В «А» классе 30%
osti
24
обожают учителя географии, в «Б» классе - 40% и в «В» классе - 70%. Девятиклассник Петя
обожает учителя географии. Какова вероятность того, что он из «Б» класса?
Задача 3.32.
В урне 7 красных, 5 желтых и 11 белых шаров. Два шара выбирают наугад. Какова вероятность,
что это красный и белый, если известно, что они разного цвета.
Задача 3.33.
Саша едет на день рождения к Маше и ищет ее дом. Ее дом находится южнее. Одна треть встречных
3
прохожих - местные. Местные всегда лгут, неместные говорят правду с вероятностью . Изначально
4
Саша оценивает вероятность того, что дом находится южнее, как a. Саша спросил первого встречного прохожего и получил ответ «севернее». Как Саша изменит свою субъективную вероятность?
Задача 3.34.
Самолет упал в горах, в степи или в море. Вероятности, соответственно, равны
он упал в горах, то при поиске его найдут с вероятностью
0, 7.
В степи -
0, 8,
0, 5, 0, 3 и 0, 2. Если
0, 2. Самолет
на море -
искали в горах, в степи и не нашли. Какова вероятность того, что он упал в море?
Задача 3.35. Русская рулетка.
Давайте сыграем в русскую рулетку... Вы привязаны к стулу и не можете встать. Вот пистолет.
Вот его барабан - в нем шесть гнезд для патронов, и они все пусты. Смотрите: у меня два патрона.
Вы обратили внимание, что я их вставил в соседние гнезда барабана? Теперь я ставлю барабан на
место и вращаю его. Я подношу пистолет к вашему виску и нажимаю на спусковой крючок. Щелк!
Вы еще живы. Вам повезло! Сейчас я собираюсь еще раз нажать на крючок. Что вы предпочитаете:
чтобы я снова провернул барабан или чтобы просто нажал на спусковой крючок?
http://forum.eldaniz.ru/index.php?topic=293.60
Задача 3.36.
1
. A
3
утверждает, что B отрицает, что C заявил, что D солгал. Какова условная вероятность того, что D
Четыре свидетеля, A, B, C и D, говорят правду независимо друг от друга с вероятностью
сказал правду?
Задача 3.37.
Подробности о пожаре (Ах, а правда ли, что тетя Соня забыла выключить утюг?) передаются по
цепочке из четырех человек (А-B-C-D), каждый из которых говорит следующему имеющуюся у
него информацию с вероятностью
p,
а с вероятностью
1−p
говорит совершенно противоположное.
D говорит, что тетя Соня утюг выключила.
Как зависит от
p
условная вероятность того, что тетя Соня действительно выключила утюг?
Задача 3.38.
Есть четыре населенных пункта
вероятностью
A, B , C
и
D.
Прямая дорога между каждыми двумя существует с
p.
а) Какова вероятность того, что можно добраться из
добраться из
A
в
D,
если между
B
и
C
A в D? б) Какова вероятность того, что можно
нет прямой дороги?
Задача 3.39. Мосты
Картинка. На картинке: два берега, посреди мелкие острова, расположенные прямоугольником
размера
n(n − 1).
В результате паводка каждый мост был размыт с вероятностью
1/2
независимо
от других. Какова вероятность того, что с одного берега можно добраться на другой?
Задача 3.40.
В урне лежат 5 пронумерованных (от одного до пяти) шаров. По очереди вытаскиваются два шара.
Какова вероятность того, что разница в номерах будет больше двух? Какова вероятность того, что
25
первым был вытащен шар с номером 2, если разница в номерах была больше двух?
Задача 3.41.
A regular
N
- polygon has vertices numbered 0, 1, 2...,
N −1
in clockwise. Let the vertex 0 be a starting
point. When you roll a dice, you will move the coin clockwise by the number on the dice. Denote the
number of the arriving vertice by
a) Are
X
and
Y
X.
Again roll a dice, you will move from the vertex
X
to the vertex
Y.
independent?
b) Find the value of
N
such that
X
and
Y
are independent
Source: Kyoto University entrance exam/Science , Problem 6, 1st Round, 1990
Задача 3.42.
Будем говорить, что событие
Известно, что
Верно ли, что
A
благоприятствует (способствует) событию
A благоприятствует B , B
A благоприятствует C ?
благоприятствует
B,
если
P (B|A) > P (B).
C.
Ответ: нет (не всегда)
Задача 3.43.
а) Известно, что
б) Известно, что
P (A|B) > P (A). Верно ли, что P (B|A) > P (B)?
P (A|B) > P (B). Верно ли, что P (B|A) > P (A)?
Ответы: а) да; б) нет
Задача 3.44.
На Древе познания Добра и Зла растет 6 плодов познания Добра и 5 плодов познания Зла. Адам и
Ева съели по 2 плода. Какова вероятность того, что Ева познала Зло, если Адам познал Добро?
Задача 3.45.
A sniper has 0.8 chance to hit the target if he hit his last shot and 0.7 chance to hit the target if he missed
his last shot. It is known he missed on the 1st shot and hit on the 3rd shot.
What is the probability he hit the second shot?
answer
8/11
Задача 3.46.
Снайпер попадает в «яблочко» с вероятностью 0.8, если в предыдущий раз он попал в «яблочко»;
и с вероятностью 0.7, если в предыдущий раз он не попал в «яблочко» или если это был первый
выстрел. Снайпер стрелял по мишени 3 раза.
а) Какова вероятность попадания в «яблочко» при втором выстреле?
б) Какова вероятность попадания в «яблочко» при втором выстреле, если при первом снайпер
попал, а при третьем - промазал?
Solution:
a)
b)
p = 0.7 · 0.8 + 0.3 · 0.7 = 0.77
0.7·0.8·0.2
8
p = 0.7·0.8·0.2+0.7·0.2·0.3
= 11
Задача 3.47.
Есть две неправильные монетки. Первая выпадает орлом с вероятностью 0.1, вторая выпадает
орлом с вероятностью 0.9. Из этих двух монеток равновероятно выбирают одну и подбрасывают ее
2 раза.
а) Верно ли, что результат первого и второго подбрасывания независимы?
б) Известно, что выбрали первую монетку. Верно ли, что результат первого и второго подбрасывания
независимы?
Задача 3.48.
Вы равновероятно могли получить письмо из Москвы или из Игарки. Все буквы в названии города в
обратном адресе кроме одной нечитаемы из-за загрязнения на конверте. Единственная различимая
26
буква - это буква «а». Какова условная вероятность того, что письмо пришло из Москвы?
Solution:
Из названия корода случайным образом оставляем одну букву.
0.5 16
p = 0.5 1 +0.5
2 = 1/3
6
6
Задача 3.49.
Вася кидает дротик в мишень три раза. Его броски независимы друг от друга. Известно, что во
второй раз он попал дальше от центра, чем в первый раз. Какова условная вероятность того, что в
третий раз он попадет ближе к центру, чем в первый раз?
Answer:
1
3
Задача 3.50.
В одном мешке лежат только спелые яблоки, в другом - одинаковое количество спелых и зеленых.
Вы случайным образом вытаскиваете яблоко из мешка, оно - спелое, вы кладете его обратно. Какова
вероятность, что следующее яблоко из того же мешка будет зеленым?
Какова вероятность, что следующее яблоко из того же мешка будет зеленым, если было
n
попыток
достать яблоко, и каждый раз вытаскивалось и клалось обратно спелое яблоко?
1
Answer:
2+2n
Задача 3.51.
Three dice are rolled. If no two show the same face, what is the probability that one is an “ace” (one spot
showing.)?
Задача 3.52. Given that a throw with ten dice produced at least one ace, what is the probability of two
or more aces?
Задача 3.53.
A1 и A2 называются условно независимыми относительно события B , если
P (A1 ∩ A2 |B) = P (A1 |B) · P (A2 |B).
а) Приведите пример таких A1 , A2 и B , что A1 и A2 , независимы, но не являются условно независимыми относительно B .
б) Приведите пример таких A1 , A2 и B , что A1 и A2 , зависимы, но являются условно независимыми
относительно B .
Определение. События
Задача 3.54.
В урне 99 белых и один черный шар. Один шар извлекается из урны наугад. Петя сказал, что шар белый. Вася сказал, что шар - белый. Какова вероятность того, что шар действительно белый, если
Петя говорит правду с вероятностью 0.8, а Вася - с вероятностью 0.9, независимо от Пети?
Задача 3.55.
У нас ходят два автобуса - 10-ый и 20-ый. Десятый приходит через десять минут после 20-го; 20-ый
- через 20 минут после десятого. Я прихожу на остановку в случайный момент времени.
а) Сколько мне в среднем ждать автобуса?
б) Сколько мне еще в среднем ждать автобуса, если я уже прождал
Ответ: а)
m
минут?
f rac253
Задача 3.56. Русская рулетка
Шестизарядный револьвер. В нем три пули занимают три соседних места. Барабан крутят один раз.
Задем первый игрок стреляет себе в голову. Если он остается жив, то барабан не перекручивается,
и второй игрок стреляет себе в голову. Затем револьвер возвращается первому игроку и т.д., до тех
пор, пока кто-то не погибнет.
Кем лучше быть в этой игре, первым или вторым?
bika
bkam
27
Ответ: у второго шансы проиграть равны
1
3
Задача 3.57.
In a tournament, there are no ties. There are 7 teams and each team plays each other exactly once. Each
team has a 50% chance of winning each game. The winner is awarded 1 point and loser no points. Total
points are accumulated to decide the ranking of the teams. In the first game, Team A beats team B. What
is the probability that A finishes with more points than B?
source: aops, t=110957
Задача 3.58. Нестандартный кубик
Нестандартный кубик изготавливают следующим образом: на каждой грани равновероятно независимо от других граней пишут одно из чисел от одного до шести. Т.е. на кубике могут оказаться
даже только шестерки. Затем этот кубик подбрасывается два раза.
а) Верно ли, что результаты подбрасываний независимы?
б) Какова вероятность того, что в первый раз выпадет шесть?
в) Какова вероятность того, что во второй раз выпадет шесть, если в первый раз выпало шесть?
г) Какова вероятность того, что шесть выпадет два раза подряд?
д) Чему равна корреляция результатов подбрасываний?
е) Чему равно ожидаемое количество шестерок на кубике, если из
n
подбрасываний оказалось
k
шестерок?
Ответы:
а) нет, т.к. если выпало шесть, то это увеличивает ожидаемое количество шестерок на кубике
б)
в)
г)
1/6 (можно
11/36
11/216
считать, что кубик сначала подбросили, а потом подписали стороны)
Задача 3.59. Suppose ten balls are inserted into a bag based on the tosses of an unbiased coin using the
following rules: insert white ball when the coin turns up heads and insert black ball when the coin turns
up tails.
Suppose someone who knows how the balls were selected but not what their colors are selects ten balls
from the bag one at a time at random, returning each ball and mixing the balls thoroughly before making
another selection. If all ten examined balls turn out to be white, what is the probability to the nearest
percent that all ten balls in the bag are white?
ans: about 7%
4. Разложение в сумму, РВС
Задача 4.1. [Чернова, пример 46]
Кубик подбрасывается n раз. Пусть X1 - число выпадений
Corr(X1 , X6 )
Подсказка: Cov(X1 , X1 + ... + X6 ) вам в помощь...
1, а
X6
- число выпадений 6. Найдите
Задача 4.2.
По 10 коробкам наугад раскладывают 7 карандашей. Каково ожидаемое (среднее) количество
пустых коробок? Подсказка: представьте результат в виде суммы 10-и случайных величин.
Задача 4.3. [Mosteller] Среднее число совпадений
Из хорошо перетасованной колоды на стол последовательно выкладываются карты лицевой стороной наверх, после чего Аналогичным образом выкладывается вторая колода, так что каждая карта
первой колоды лежит под картой из второй колоды. Каково среднее число совпадений нижней и
верхней карт?
enie
28
Задача 4.4. Grimmett, 3.3.3.
В группе 20 человек. Каждый из них подбрасывает по кубику. Найдите ожидаемый выигрыш и
дисперсию выигрыша группы, если:
а) за каждую пару игроков, выкинувших одинаковое количество очков, группа получает один
тугрик
б) за каждую пару игроков, выкинувших одинаковое количество очков, группа получает эту сумму
в тугриках
Задача 4.5. Coupon’s collector problem
Внутри упаковки шоколадки «Веселые животные» находится наклейка с изображением одного из
30 животных. Предположим, что все наклейки равновероятны. Большой приз получит каждый, кто
соберет наклейки всех животных. Какое количество шоколадок в среднем нужно купить, чтобы
выиграть большой приз?
Задача 4.6. [Mosteller]
В
n
урн случайным образом бросают (один за одним)
k
шаров. Найдите математическое ожидание
числа пустых урн.
Задача 4.7.
У Васи
100
рублей, у Пети -
150.
Они играют в орлянку правильной монеткой до тех пор, пока все
деньги не перейдут к одному игроку. Какова вероятность, что победит Вася?
Задача 4.8.
На карточках написаны числа от 1 до
n.
В игре участвуют
n
человек. В первом туре каждый
получает случайным образом по одной карточке. Во втором туре карточки выдаются заново. Призы
раздаются по следующему принципу: Человек не получает приз, только если найдется кто-то другой,
кто получил большие числа в каждом туре. Каково среднее количество человек, получивших приз?
Взято с www.zaba.ru, какая-то олимпиада.
Задача 4.9.
А.А. Мамонтов сидит в 424 аудитории. Эконометрику собираются сдавать несколько человек. На
поиски пустых аудиторий послано 3 студента-разведчика. На втором этаже 9 учебных аудиторий,
5 из них заняты. Каждый из 3 студентов-разведчиков независимо друг от друга заглядывает в 3
аудитории. Если студент обнаруживает пустую аудиторию, то он сообщает ее номер А.А. Мамонтову.
Каково среднее количество обнаруженных пустых аудиторий?
Задача 4.10.
У Маши 30 разных пар туфель. И она говорит, что мало! Пес Шарик утащил (без разбору на левые
и правые) 17 туфель. Сколько полных пар в среднем осталось? Сколько полных пар в среднем
досталось Шарику?
Задача 4.11.
Из колоды в 52 карты извлекается 5 карт. Сколько в среднем извлекается мастей? Достоинств?
Тузов?
Ответ:
Масть:
4 · (1 −
Достоинство:
Туз:
4·
5
52
5
C39
5 )
C52
13 · (1 −
5
C48
5 )
C52
Задача 4.12.
Вася пишет друг за другом наугад 100 букв из латинского алфавита.
а) Каково ожидаемое количество букв, встречающихся в написанном «слове» ровно один раз?
29
б) Как изменилась бы искомая величина,
Ak,n ,
если бы в алфавите было
k
букв, а Вася писал бы
n букв?
Найдите limn→∞ Ak,n , limk→∞ Ak,n
«слово» из
в)
Задача 4.13.
n
За круглым столом сидят в случайном порядке
супружеских пар (всего
2n
человек). Пусть
X
-
число пар, где супруги оказались напротив друг друга.
Найдите
E(X)
и
V ar(X)
Задача 4.14.
Suppose there were
striking the
2m
m
married couples, but that
people at random. Let
X
d
of these 2m people have died. Regard the
be the number of surviving couples. Find:
E(X)
d
and
deaths as
V ar(X)
Задача 4.15. throwing pies
A group of initially N people play the following game. Each one picks another person at random as a
target, and at the voice of «now!» they throw their pies at their selected targets with perfect aim. Each
player hit by a pie must abandon the game; the ones not hit by a pie are called «survivors». They keep
playing until all of them have been hit or only one survivor remains.
a) If at a given stage of the game there are n survivors, what is the expected number of survivors at the
next stage?
b) If at a given stage of the game there are n survivors what is the probability of having exactly k survivors
at the next stage?
c) What is the asymptotic behavior as N tends to infinity of the probability of ending up with one
survivor.
Generalize the problem assuming that the players’ aim is not perfect. Assume that the probability p of
hitting the selected target is constant and the same for everybody.
Задача 4.16.
Над озером взлетело 20 уток. Каждый из 10 охотников стреляет в утку по своему выбору. Каково
ожидаемое количество убитых уток, если охотники стреляют без промаха? Как изменится ответ,
если вероятность попадания равна 0,7? Каким будет ожидаемое количество охотников, попавших в
цель?
Задача 4.17. [9.57, 9.23 Кочетков]
Десять человек садится на первом этаже в лифт шестиэтажного здания. Каждый выходит на
случайно выбираемом им этаже (кроме первого).
а) Сколько остановок (в среднем) сделает лифт?
Ответьте на следующие вопросы с помощью
Sn (k) = 1n + 2n + ... + k n :
б) До скольки этажей (в среднем) лифт вообще не доедет?
в) На каком этаже (в среднем) будет первая остановка?
г) На каком этаже (в среднем) будет последняя остановка?
д) Сколько этажей (в среднем) лифт «проскочит» при подъеме?
Задача 4.18. Судьба Дон Жуана
n знакомых девушек (их всех зовут по-разному). Он пишет им n писем, но, по рассеянности,
раскладывает их в конверты наугад. С.в. X обозначает количество девушек, получивших письма,
написанные лично для них. Найдите E(X).
Ответ: E(X) = 1
У Васи
Задача 4.19.
Вам предложена следующая игра. Изначально на кону 0 рублей. Раз за разом подбрасывается
правильная монетка. Если она выпадает орлом, то казино добавляет на кон 100 рублей. Если
монетка выпадает решкой, то все деньги, лежащие на кону, казино забирает себе, а Вы получаете
30
красную карточку. Игра прекращается либо когда Вы получаете третью красную карточку, либо в
любой момент времени до этого по Вашему выбору. Если Вы решили остановить игру до получения
трех красных карточек, то Ваш выигрыш равен сумме на кону. При получении третьей красной
карточки игра заканчивается и Вы не получаете ничего.
а) Как выглядит оптимальная стратегия в этой игре?
б) Чему при этом будет равен средний выигрыш?
Задача 4.20.
В каждой из двух урн находится по 50 белых и 50 черных шаров. Вася одновременно вытаскивает
по шару из каждой урны и выбрасывает их.
Пусть
X
Найдите
количество раз, когда из обеих урн были одновременно вытащены белые шары.
E(X), V ar(X)
Задача 4.21.
На карточках написаны числа от 1 до
n.
Вася достает их одну за другой наугад. Если номер
карточки является соседним с номером предыдущей карточки, то Вася получает 1 рубль. Пусть
X
- Васин выигрыш.
Найдите
E(X), V ar(X)
Задача 4.22.
Вася называет наугад 50 чисел от 1 до 100 (допускаются повторения), а Петя называет наугад 50
чисел от 1 до 100 (без повторов).
Пусть
X
и
Y
a) Сравните
б) Сравните
это суммы этих чисел.
E(X) и E(Y )
V ar(X) и V ar(Y )
Solution:
E(X) = E(Y ), V ar(X) = V ar(Y )
Задача 4.23.
Кубик подбрасывается до тех пор, пока каждая грань не выпадет по разу. Найдите математическое
ожидание и дисперсию числа подбрасываний.
Задача 4.24.
Правильная монетка подбрасывается
n
раз. Серия - это последовательность подбрасываний из
одинаковых результатов (к примеру, в последовательности ОООРРРО три серии).
а) Каково ожидаемое количество серий?
б) Дисперсия числа серий?
в) А если монетка неправильная и выпадает гербом с вероятностью
p?
Задача 4.25. Рулет
Длинный рулет разрезан на
n
частей. Каждый из
k
гостей по очереди забирает себе один кусочек,
выбираемый случайным образом. В результате остается
n−k
кусочков рулета. Оставшиеся кусочки
рулета лежат «сериями», разделенными «дырками» от забранных кусочков. Каково ожидаемое
число «серий» оставшихся кусочков? К чему стремится эта величина при
n → ∞?
Aвтор: Алексей Суздальцев
Решение 1:
X - число «серий», X = X1 + ... + Xn , где Xi
с i-го кусочка.
n−k
Ответ:
+ (n − 1) nk n−k
= (k + 1) n−k
n
n−1
n
- индикатор, показывающий, начинается ли новая серия
Решение 2:
Закольцуем рулет, добавив в него еще один кусочек (для хозяина дома). Получаем
n−k
альную серию. Вероятность того, что некая серия непуста, равна
.
n
(k + 1)
потенци-
hiki
shar
tnih
nih2
31
(k + 1) n−k
n
Ответ, конечно,
Задача 4.26.
В здании 10 этажей, на каждом этаже 30 окон. Вечером в каждом окне независимо от других свет
включается с вероятностью
p.
a) Чему равно ожидаемое количество «ноликов» на фасаде здания?
б) Чему равно ожидаемое количество «крестиков» на фасаде здания?
в) При каких
p
эти количества максимальны? Минимальны?
Примечание: два разных нолика могут иметь общие точки
Вставить рисунок нолика и рисунок крестика, пример подсчета
Задача 4.27.
В урне
n
шаров пронумерованных 1,2,...
n.
Наугад вытаскивают
k.
Найдите ожидание и дисперсию
суммы номеров.
Задача 4.28.
Кричали женщины «Ура!» и в воздух чепчики бросали! (А.С. Грибоедов)
Приезжающих из армии или от двора встречают
вверх
n
n
женщин. Они одновременно подбрасывают
чепчиков. Ловят чепчики наугад. Женщины, поймавшие свой чепчик уходят. А женщины,
поймавшие чужой чепчик, снова подбрасывают его вверх. Подбрасывание чепчиков продолжается
до тех пор, пока каждая не поймает свой чепчик.
N - количество женщин, поймавших свой чепчик после первого подбрасывания.
E(N ) и V ar(N )
б) Пусть R - количество подбрасываний. Найдите E(R).
в) Пусть S - количество чепчиков, пойманных мадмуазель NN. Найдите E(S).
а) Пусть
Найдите
г) Какова вероятность того, что ни одна женщина не поймает свой чепчик после 1-го подбрасывания?
[Ross,
example 3.13]
Задача 4.29. [Ross, Pro Models, 2.43]
В урне лежат
m
шаров цвета морской волны и
k
шаров цвета «персик». Маша пометила один из
шаров цвета «персик». Шары извлекают в случайном порядке.
а) Какова вероятность того, что меченый шар появится раньше, чем первый шар цвета морской
волны?
б) Каково ожидаемое количество шаров цвета «персик», извлеченных до первого шара цвета морской
волны?
в) Каково ожидаемое количество шаров цвета «персик», извлеченных между первым и вторым
шарами цвета морской волны?
Задача 4.30.
В урне лежат
a шаров абрикосового цвета и b шаров бежевого цвета. Шары извлекаются до тех пор,
пока в урне не останутся шары только одного цвета.
а) Какова вероятность того, что в конце останутся шары цвета абрикос?
б) Каково ожидаемое количество оставшихся шаров?
в) Каково ожидаемое количество оставшихся шаров, если известно, что остались шары цвета беж
mechenii shar
Hint: номер 4.29
Задача 4.31.
В урне лежат
a
абрикосовых шаров,
b
белых шаров и
c
синих шаров. Шары извлекаются наугад
по-очереди без возвращения.
а) Какова вероятность того, что первыми полностью будут извлечены абрикосовые шары?
б) Сколько шаров в среднем останется после того, как впервые полностью будет извлечен какойнибудь цвет? Сколько шаров при этом в среднем будет извлечено?
в) Какова вероятность того, что абрикосовые будут полностью извлечены раньше белых, если
orki
tuza
32
известно, что белые были полностью извлечены позже синих?
mechenii
izvlekaem
shar
do odnotsvetnih
Hint: номера 4.29 и 4.30
Solution:
а)
pabc
- вероятность того, что сначала полностью извлекаются абрикосовые шары, затем полностью
извлекаются белые и в конце - синие.
Заметим, что:
c
- вероятность того, что
pabc + pbac = a+b+c
c
pabc + pacb + pbac = a+c
- вероятность того,
Отсюда находим pacb и все остальные
синий шар - последний
что абрикосовые вышли раньше синих
б) Рассмотрим случай, когда невзятым остается какой-нибудь синий шар. Значит, он лежит либо
1
1
1
после всех белых, либо после всех абрикосовых. Вероятность этог равна pc =
+ b+1
− a+b+1
.
a+1
Значит в среднем остается apa + bpb + cpc шаров.
Source: AMM E2724 by Harry Lass
Задача 4.32.
b шаров цвета беж и c шаров цвета бедра испуганной нимфы.
Извлекается наугад n ≤ a + b + c шаров. Пусть A и B - количество извлеченных шаров цвета абрикос
В урне
a
шаров цвета абрикос,
и цвета беж
а) Какова вероятность того, что
i-ый
извлеченный шар будет абрикосовым?
б) Каково среднее количество извлеченных шаров цвета абрикос?
в) Найдите
V ar(A), Cov(A, B), Corr(A, B)
г) Как изменятся ответы, если после извлечения шара и записывания его цвета шар будет возвращаться обратно в урну?
Задача 4.33. Grimmett, 3.4.4.
В урне
A
находятся
a
шаров цвета «абрикос», в урне
B
-
a
шаров цвета беж. В каждый момент
времени выбирают по одному шару наугад из каждой урны и меняют местами.
Найдите ожидаемое количество шаров цвета «абрикос» в урне
A
после
k
шагов.
Задача 4.34.
Из хорошо перетасованной колоды в 52 карты, содержащей четыре туза, извлекаются сверху карты
до появления первого туза. На каком месте в среднем появляется первый туз?
Задача 4.35.
Из колоды в 52 карты случайным образом извлекаются 26 карт и с сохранением порядка между
собой кладутся наверх колоды.
Сколько в среднем карт при этом остается на своих местах?
Solution:
Остаются на своих местах карты, выбранные сверху колоды и карты, невыбранные снизу колоды.
далее-?
Задача 4.36. [Ross, 2.45]
У кота Базилио
n
пустых копилок и
z
золотых. Базилио наугад бросает один за одним золотые в
копилки. Если золотой падает в копилку, где уже есть золотые, то слышно приятное позвякивание.
Найдите ожидаемое количество позвякиваний.
Задача 4.37. Игла Бюффона, Buffon needle
Плоскость расчерчена параллельными линиями, находящимися на расстоянии в 1 см. Случайным
образом на плоскость бросается веревка длины
a
см. Пусть
X
- количество пересечений веревки с
начерченными линиями.
а) Верно ли, что
E(X)
пропорционально
a?
б) Если вместо веревки взять жесткое кольцо с диаметром 1 см, то чему будет равно
в) Чему равно
E(X)
для веревки длины
a?
X , E(X)?
davi
33
г) Если вместо веревки бросается иголка длины
a < 0.5,
то чему равна вероятность того, что она
пересечет хотя бы одну линию?
Solution (Bill Taylor?):
a) да, т.к. разбив веревку на две части получим, что
б) При
a=π
получаем
E(X) = 2
a получаем E(X) =
в, г) Для произвольного
E(X) = E(X1 ) + E(X2 )
2a
π
Задача 4.38. Треугольник Бюффона?
Плоскость расчерчена параллельными линиями, находящимися на расстоянии в 1 см. Случайным
образом на плоскость бросается треугольник со сторонами
a < 1, b < 1, c < 1.
Какова вероятность того, что линия будет пересекать треугольник?
Решение:
Пользуясь предыдущей задачей:
2(a+b+c)
.
π
Вероятность в два раза меньше (одна линия пересекает треугольник в двух местах):
P = a+b+c
π
Source: aops, t=179733
Ожидаемое число пересечений равно
Задача 4.39. Условная вероятность по Бюффону
Плоскость расчерчена параллельными линиями, находящимися на расстоянии в 1 см. Случайным
образом на плоскость бросается треугольник с периметром
P < 2.
Строго внутрь него случайным
p < P.
образом бросается второй треугольник с периматром
Какова вероятность того, что прямая малый треугольник, если она пересекает большой?
Solution:
2P/π .
2p/π .
Ожидаемое число пересечений большого треугольника:
Ожидаемое число пересечений малого треугольника:
Вероятности пересечений - в 2 раза меньше.
Условная вероятность равна отношению периметров.
Задача 4.40. Условная вероятность по Бюффону-2 (?)
Одна в другой в пространстве находятся две сферы, радиусов
R
и
r.
Случайным образом в про-
странстве проводится прямая. Какова вероятность того, что она пересечет малую сферу, если она
пересекает большую?
Solution:
Переформулируем задачу, как задачу Бюффона (параллельные прямые в пространстве...)
Вероятность в два раза меньше ожидаемого числа пересечений. Ожидаемое число пересечений
2 2
пропорционально площади поверхности. Ответ - отношение площадей поверхностей, т.е. R /r
Коммент:
Как эти все задачи аккуратно оформлять? Ведь нельзя провести «равновероятным» образом прямую в пространстве. Нельзя «равновероятно» бросить иголку на плоскость...
Задача 4.41.
В лесу живет
N
удавов. Каждый из них имеет свою длину. Обозначим
выбранного удава. Сегодня
чим
n
σ2
дисперсию длины наугад
удавов выползли погреться на солнышке на большой поляне. Обозна-
Xn
среднюю длину выползших удавов.
P
Cov(X1 , N
i=1 Xi )?
2
б) Выразите Cov(Xi , Xj ) через σ для i 6= j .
2
в) Выразите V ar(X̄n ) через σ .
а) Чему равно
Задача 4.42. Do men have more sisters than women?
В семье
n
детей. Предположим, что вероятности рождения мальчика и девочки равны. Дед Мороз
спросил каждого мальчика «Сколько у тебя сестер?» и сложив эти ответы получил
X.
Мороз спросил каждую девочку «Сколько у тебя сестер?» и cложив эти ответы получил
Затем Дед
Y.
34
Найдите
E(X)
и
E(Y )?
source: cut-the-knot
solution:
дважды сестра считается дважды
Если в семье
n(n − 1)/2
n
детей, то ожидаемое количество мужских и женских сестер одинаково и равно
(?)
Задача 4.43.
Допустим, что на острове Независимом погода в разные дни независима. Известно, что день оказывается солнечным с вероятностью
вероятностью
1 − p − q.
Пусть
X
p,
пасмурным без дождя с вероятностью
- число солнечных дней, а
Y
q
и дождливым с
число пасмурных дней без дождя за
год.
a) Найдите
Cov(X, Y )
б) Прокомментируйте знак ковариации
Ответ:
Cov(X, Y ) = −365pq ,
чем больше
X,
тем (скорей всего) меньше
Y
Задача 4.44. Девятый вал
Пусть
Xn
- iid, непрерывно распределены, это размер
n-ой
волны. Будем называть волну большой,
если она больше предыдущей и больше последующей.
а) Какова вероятность того, что
n-ая
волна - большая?
б) Какой по счету в среднем появляется очередная большая волна?
в) Попробуйте (не совсем строго) обосновать название "Девятый вал"
Solution.
a) Одна из трех последовательных волн обязательно больше двух других. Поскольку волны iid, то
1/3.
U [0; 1], то равенство вероятности одной третьей соответствует разрезанию куба на
вероятность того, что большой будет именно вторая равна
a) Если волны iid
три пирамиды.
Задача 4.45.
Из грота ведут 10 штреков, с длинами 100м, 200м,... 1000м. Самый длинный штрек оканчивается
выходом на поверхность. Остальные - тупиком. Вася выбирает штреки наугад (естественно, в
тупиковый штрек он два раза не ходит). Какой в среднем путь он нагуляет прежде чем выберется
на поверхность.
Решение:
Вероятность посещения каждого тупикового штрека равна 0.5.
Следовательно:
E(X) = 1000 + 0.5(100 + ... + 900) = 3250
Задача 4.46.
У меня в кармане 3 рубля мелочью. Среди монет всего одна монета достоинством 50 копеек. Я
извлекаю монеты по одной наугад до извлечения 50 копеечной монеты. Какую сумму в среднем я
извлеку?
Решение:
Каждая монета кроме 50 копеечной выбирается с вероятностью 0.5.
2.5
Следовательно, E(S) = 0.5 +
= 1.75
2
Задача 4.47. Модница
В шкатулке у Маши 100 пар сережек. Каждый день утром она выбирает одну пару наугад, носит
ее, а вечером возвращает в шкатулку. Проходит год.
а) Сколько в среднем пар окажутся ни разу не надетыми?
б) Сколько в среднем пар окажутся одетыми не менее двух раз?
в) Как изменятся ответы, если каждый день Маша покупает себе новую пару сережек и вечером
добавляет ее в шкатулку?
Коммент: пункт в не проверялся
35
а)
б)
100 · (0.99)365
100 · (1 − 0.99365 − 365 · 0.01 · 0.99364 )
Задача 4.48. Две корзины, две игры
В корзине А лежит 50 белых и 50 черных шаров.
В корзину Б поместили 100 наугад выбранных шаров из корзины, где изначально лежали 100 белых
и 100 черных шаров.
Игра 1. Из корзины наугад извлекается один шар. Если он белого цвета, Вы получаете 100 рублей.
Игра 2. Из корзины извлекаются все шары. За каждый белый шар Вы получаете 1 рубль.
Сравните ожидаемый выигрыш и дисперсию выигрыша для четырах случаев:
а) Игра 1 с корзиной А
а) Игра 1 с корзиной Б
а) Игра 2 с корзиной А
а) Игра 2 с корзиной Б
Задача 4.49.
В магазине продается 50 видов конфет. Каждый из 10 покупателей покупает один вид конфет,
выбираемый наугад. Сколько видов конфет будет в среднем куплено?
Задача 4.50.
На отрезке выбирается равномерно и независимо друг от друга
(n + 1)
n
точек. Они разбивают отрезок на
часть.
а) Верно ли, что длины полученных частей одинаково распределены?
б) Чему равна ожидаемая длина самого левого части?
в) После разбития отрезка на
(n + 1)
часть еще одна точка равномерно выбирается на отрезке.
Какова средняя длина части, в которую она попадает?
Solution:
а) да, но зависимы - разбиваем окружность на (n
1
б)
n+1
2
в)
- разбиваем окружность на (n + 2) части
n+2
+ 1)
часть
5. Первый шаг
Задача 5.1.
Испытания по схеме Бернулли проводятся до первого успеха. Чему равно ожидаемое количество
испытаний, если вероятность успеха в отдельном испытании равна
1
Ответ:
p
p?
Задача 5.2.
Саша и Маша по очереди подбрасывают кубик. Посуду будет мыть тот, кто первым выбросит
шестерку. Маша бросает первой. Каковы ее шансы на «победу»?
Задача 5.3.
Саша и Маша решили, что будут рожать нового ребенка, до тех пор, пока в их семье не будут дети
обоих полов. Каково ожидаемое количество детей?
Задача 5.4.
Четыре человека играют в игру «белая ворона платит». Они одновременно подкидывают монетки.
Если три монетки выпали одной стороной, а одна - по-другому, то «белая ворона» оплачивает всей
четверке ужин в ресторане. Если «белая ворона» не определилась, то монетки подбрасывают снова.
Сколько в среднем нужно подбрасывания для определения «белой вороны»?
36
Задача 5.5.
Саша и Маша каждую неделю ходят в кино. Саша доволен фильмом с вероятностью 1/4, Маша - с
вероятностью 1/3.
a) Сколько недель в среднем пройдет до тех пор, пока кто-то не будет доволен?
b) Какова вероятность того, что первым будет доволен Саша?
c) Сколько недель в среднем пройдет до тех пор, пока каждый не будет доволен хотя бы одним
просмотренным фильмом?
Задача 5.6.
По ответу студента на вопрос преподаватель может сделать один из трех выводов: ставить зачет,
ставить незачет, задать еще один вопрос. Допустим, что знания студента и характер преподавателя
таковы, что при ответе на отдельный вопрос зачет получается с вероятностью
вероятностью
p2 = 1/8.
p1 = 3/8,
незачет - с
Преподаватель задает вопросы до тех пор, пока не определится оценка.
а) Сколько вопросов в среднем будет задано?
б) Какова вероятность получения зачета?
Задача 5.7.
Вы играете в следующую игру. Кубик подкидывается неограниченное число раз. Если на кубике
выпадает 1, 2 или 3, то соответствующее количество монет добавляется на кон. Если выпадает 4
или 5, то игра оканчивается и Вы получаете сумму, лежащую на кону. Если выпадает 6, то игра
оканчивается, а Вы не получаете ничего.
а) Чему равен ожидаемый выигрыш в эту игру?
б) Изменим условие: если выпадает 5, то набранная сумма сгорает, а игра начинается заново. Чему
будет равен ожидаемый выигрыш?
Задача 5.8. [Grimmett, Stirzaker]
Вася подкидывает кубик. Если выпадает единица, или Вася говорит «стоп», то игра оканчивается,
если нет, то начинается заново. Васин выигрыш - последнее выпавшее число. Как выглядит оптимальная стратегия? Как выглядит оптимальная стратегия, если за каждое подбрасывание Вася
платит 35 копеек?
Задача 5.9.
Саша и Маша подкидывают монетку до тех пор, пока не выпадет последовательность РОО или
ОOР. Если игра закончится выпадением РОО, то выигрывает Саша, если ОOР, то - Маша.
а) У кого какие шансы выиграть?
b) Сколько в среднем времени ждать до появления ООР?
с) Сколько в среднем времени ждать до определения победителя?
sir
Задача 5.10.
Есть три комнаты. В первой из них лежит сыр. Если мышка попадает в первую комнату, то она
находит сыр через одну минуту. Если мышка попадает во вторую комнату, то она ищет сыр две
минуты и покидает комнату. Если мышка попадает в третью комнату, то она ищет сыр три минуты
и покидает комнату. Покинув комнату, мышка выходит в коридор и выбирает новую комнату
наугад (т.е. может зайти в одну и ту же). Сейчас мышка в коридоре. Сколько времени ей в среднем
потребуется, чтобы найти сыр?
Задача 5.11.
Иська и Еська по очереди подбрасывают два кубика. Иська бросает первым. Иська выигрывает,
если при своем броске получит 6 очков в сумме на двух кубиках. Еська выигрывает, если при своем
броске получит 7 очков в сумме на двух кубиков. Кубики подбрасываются до тех пор, пока не
определится победитель.
а) Верно ли, что события
A = {на
двух кубиках в сумме выпало больше 5 очков} и
из кубиков выпала 1} являются независимыми?
B = {на
одном
37
б) Какова вероятность того, что Еська выиграет?
Задача 5.12.
Players A and B play a (fair) dice game. «A» deposits one coin and they take turns rolling a single dice,
«B» rolling first.
If «B» rolls an even number, he collects a coin from the pot. If he rolls an odd number, he put a coin
(coins with same values always). If «A» (plays and) rolls an even number, he collects a coin but if he rolls
an odd number, he does NOT add a coin. The game continues until the pot is exhausted.
Question: what is the probability that «A» wins this game (that is, exhaust the pot) ?
t=138358
Задача 5.13. Уравнение Пелля-0
2
a2 −
√ 3bn = 1.
(2 + 3)
Найдите все решения в целых числах уравнения Пелля:
Hint: Рассмотрите коэффициенты последовательности
Задача 5.14. Уравнение Пелля
В мешке 2 апельсина и 1 яблоко. Если бы Вася сразу извлек два предмета наугад, то, конечно,
вероятность взять два апельсина равнялась бы одной третьей. Перед тем, как Вася берет два
предмета, в мешок добавили
a
апельсинов и
b
яблок, но при этом вероятность взять два апельсина
не поменялась.
Найдите 9-ое наименьшее возможное
a
Задача 5.15. Уравнение Пелля-2
На бумаге в клеточку отмечено
2×n
точек, (нарисовать какие...).
Каждый вертикальный и горизонтальный отрезок Тиша прорисовывает с вероятностью
P (n)
0.5.
Пусть
- вероятность того, что все точки окажутся связаны, но при этом не будет ни одной петли.
Найдите
P (10)
Задача 5.16. Числа Фибоначи и монетки
A fair coin is tossed 2007 times. Find the probability that at no point during the tossing are two heads
flipped consecutively.
Solution:
Count number of binary strings that do not contain two consecutive 1s: Let
fa (n)
denote the number of
binary strings of length n, with last digit a (0 or 1).
f0 (1) = 1; f1 (1) = 1
(There’s one binary string that ends in 0, and one that ends in 1).
The number of binary strings of length n ending with 0, is the sum of the number of binary strings of
length n-1 ending in 0 or 1.
f0 (n) = f0 (n − 1) + f1 (n − 1)
Also the number of binary strings of length n ending with 1, is the sum of the number of binary strings
of length n-1 ending in 0.
f1 (n) = f0 (n − 1)
Adding the previous two equations together;
f0 (n) + f1 (n) = 2f0 (n − 1) + f1 (n − 1) = [f0 (n − 1) + f1 (n −
1)] + [f0 (n − 2) + f1 (n − 2)]
The LHS represents the total number of binary strings of length n without two consecutive heads. This is
the fibonacci sequence since
F1997
So the answer is 1997
2
Fn = Fn−1 + Fn−2
Задача 5.17.
A player is playing the following game. In each turn he flips a coin and guesses the outcome. If his guess
is correct, he gains 1 point; otherwise he loses all his points. Initially the player has no points, and plays
the game until he has 2 points.
(a) Find the probability
pn
that the game ends after exactly
n
flips.
(b) What is the expected number of flips needed to finish the game?
he 7
38
Задача 5.18. [Mosteller]
Два кубика подбрасываются неограниченное число раз. Какова вероятность того, что сумма очков
равная пяти, появится раньше суммы очков, равной семи?
Задача 5.19. Китайский ресторан
Каждый момент времени в китайский ресторан приходит новый посетитель. Если сейчас в ресточеловек, а за конкретным столиком сидит b человек, то вероятность того, что новый
b
θ
. С вероятностью
посетитель сядет за
n+θ
n+θ
отдельный столик.
ране сидит
n
посетитель присоединится к этому столику равна
Каково ожидаемое число занятых столиков к моменту времени
n?
Задача 5.20. [Mosteller] На краю утеса
Пьяница стоит на расстоянии одного шага от края пропасти. Он шагает случайным образом либо
к краю утеса либо от него. На каждом шагу вероятность отойти от края равна 2/3, а шаг к краю
имеет вероятность 1/3. Каковы шансы пьяницы избежать падения?
Задача 5.21. Amoeba
A population starts with a single amoeba. For this one and for the generations thereafter, there is a
probability of 3/4 that an individual amoeba will split to create two amoebas, and a 1/4 probability that
it will die out without producing offspring. What is the probability that the family tree of the original
amoeba will go on for ever?
Solution:
quadratic equation for p
Source: cut-the-knot
Задача 5.22.
Let X be the sum of realized tosses of a fair dice ( for exemple if you toss a dice 3 times and you got 4
,6,1 then X=4+6+1=11). what’s expected number of throwings until you got X>=15?
Solution:
Solve the difference equation
En = (7/6) · En−1 − (1/6) · En−7
to get E(15) about 4.76
Задача 5.23.
Suppose the probability to get a head when throwing an unfair coin is p, what’s the expected number of
throwings in order to get two consecutive heads?
Задача 5.24.
What is the probability of getting two consecutive heads after exactly n throws?
Выведите разностное уравнение (2-го порядка).
(?) Решается ли оно в явном виде - непонятно.
Задача 5.25. Book Index Range
The index of a book lists every page on which certain words appear. To save space these are listed in
ranges; for example, if a word occurs on pages 1, 2, 3, 5, 8, and 9, then its index contains ranges: 1-3, 5,
8-9.
A certain word appears on each page of an n-page book (n > 0) independently with probability p. Find
the expected number of entries in its index entry
Solution:
Let rn(p) be the sought expectation. We shall show that rn(p) = p + (n - 1)p(1 - p)
by induction on n.
When n = 1, (1) becomes r1(p) = p, which is clearly true.
shka
nger
39
Suppose n > 1 and assume (1) holds for rn-1(p), which is the expected number of ranges for an (n-1)-page
book. The addition of one more page may or may not change the total number of ranges. When page n is
added, the number of ranges increases by one if the term occurs on page n and does not occur on page (n
- 1); this happens with probability p(1 - p). Otherwise, the number of ranges does not change. Therefore,
rn(p) = p(1 - p)·[rn-1(p) + 1] + [1 - p(1 - p)]·rn-1(p) = rn-1(p) + p(1 - p)
= p + (n - 1)p(1 - p).
Source: cut-the-knot
Задача 5.26. [Mosteller] Разорение игрока
У игрока М имеется 1 доллар, а у игрока N 2 доллара. После каждого тура один из игроков
выигрывает у другого один доллар. Игрок М более искусен, чем N, так что он выигрывает 2/3 игр.
Игроки состязаются до банкротства одного из них. Какова вероятность выигрыша для M?
Задача 5.27. Сумасшедшая старушка
В самолет, имеющий
n
мест, проданы все билеты. Для посадки в самолет пассажиры выстроились
в очередь (не обязательно по номерам билетов). Среди пассажиров есть сумасшедшая старушка.
Она растолкала всех локтями, первой ворвалась в салон и села на первое понравившееся ей место.
Нормальный пассажир садится на свое место, если оно не занято; если оно занято, то пассажир
садится произвольным образом на любое свободное.
а) Какова вероятность того, что последний в очереди пассажир сядет на свое место?
б) Какова вероятность того, что 7-ой с конца пассажир сядет на свое место?
с) Каково примерно ожидаемое количество пассажиров, севших на свои места?
в) «заразное сумасшествие»: в начале очереди стоит
k
сумасшедших старушек. Как изменятся
ответы?
Solution: (no-recursion)
The xth seat cannot be empty, where x is not equal to 1 or 100, otherwise the xth guy will sit on it. He
can only sit at the 100th or the 1st one. The prob of 1st seat is empty = the prob of the 100th seat is
empty by symmetry, So P =0.5.
The prob that the 1st guy/lady choose the 1st seat is equal to the prob that he/she chooses the 100th
seat. This is how the symmetry argument comes in.
b) вероятности с конца равны:
1/2, 1/3, 1/4
и т.д.
Задача 5.28. Сумасшедшие старушки и благоразумный пассажир
На самолет, имеющий
100 мест, проданы все билеты. Для посадки в самолет пассажиры выстроились
в очередь (не обязательно по номерам билетов). Первые 99 пассажиров - сумасшедшие старушки.
Они садятся на наугад выбранные места. Последний пассажир садится на то место, которое указано
в его билете. Если это место занято, то он с помощью стюардессы сгоняет старушку со своего
законного места. Согнанная с чужого места сумасшедшая старушка становится благоразумной и
садится на свое место по билету. Возможно для этого придется согнать еще одну старушку и т.д.
Каково ожидаемое количество потревоженных старушек?
Source: Колмогоровская студенческая олимпиада (?)
Задача 5.29. Ковбои
Собрались 100 ковбоев. Каждый из них выбрал себе из остальных своего самого главного врага случайным образом. Далее ковбои по очереди стреляют, каждый в своего главного врага. Естественно,
если жив сам, и если жив самый главный враг. Ковбои стреляют без промаха.
Сколько ковбоев в среднем останется в живых?
Какова вероятность, что случайно выбранный ковбой останется в живых?
Допустим, среди этих ковбоев есть ковбои без оружия, т.е. они стрелять не могут, но могут служить
мишенью. Пусть доля ковбоев-пацифистов равна
n.
Solution: 50,
1
1
,
2 2−γ
γ.
Ответьте на предыдущий вопросы при больших
ka 2
aman
-Off
40
Источник: лента
mathr u
Задача 5.30. Сумасшедшая старушка вяжет в дороге.
В сумке у старушки
k > 0
красных лоскутков и
z > 0
зеленых. Старушка вяжет шарфики для
внуков. Для этого она достает из сумки лоскутки наугад по одному и подшивает их друг к другу
пока у них совпадает цвет. Если она извлекает лоскуток, чей цвет отличается от предыдущего, то
она кладет его обратно в сумку, перемешивает лоскутки в сумке и начинает вязать новый шарф.
Какова вероятность того, что последний шарф будет красным?
Задача 5.31. Новый шаман
Прежнего шамана племени забодало носорогом. На его смену пришел молодой и перспективный
шаман. Чтобы вызвать дождь необходимо произнести заклинание «АБРА». Молодой шаман знает,
что в заклинании участвуют всего три буквы: «А», «Б» и «Р»; и что в заклинании нет двух одинаковых букв подряд. Больше ничего о заклинании молодому шаману неизвестно. Поэтому шаман
произносит буквы по одной наугад соблюдая эти два правила.
Сколько времени придется в среднем ждать вождю, прежде чем пойдет дождь?
Source: http://www.artofproblemsolving.com/Forum/viewtopic.php?t=151755
Задача 5.32.
Вася бьет мячом по воротам 100 раз. В первый раз вероятность попасть равна
f rac12,
в каждый
последующий раз вероятность попасть увеличивается - Вася становится метче; при этом разные
удары независимы. Какова вероятность того, что Вася попадет в ворота четное число раз?
Вариация:
Вася нажимает на пульте телевизора кнопку «On-Off» 100 раз подряд. Пульт старый, поэтому в
1
первый раз кнопка срабатывает с вероятностью , затем вероятность срабатывания падает. Какова
2
вероятность того, что после всех нажатий телевизор будет включен, если сейчас он выключен?
Задача 5.33. Прогноз погоды
Пусть погода может быть сухой или дождливой. На следущий день погода остается такой же как
была с вероятностью
p
и меняется с вероятностью
Какова вероятность того, что через
n
(1 − p).
дней погода будет такая же как сейчас?
Solution:
pn = ppn−1 + (1 − p)(1 − pn−1 ) или pn = (2p − 1)pn−1 + 1 − p
p0 = 1, p1 = p
1
1
n
Общее решение имеет вид: pn = (2p − 1) +
2
2
Начальные условия:
Source: aops, t=179732
As a simplified model for weather forecasting, suppose that the weather (either wet or dry) tomorrow will
be the same as the weather today with probability
p.
If the weather is dry on January 1st, what is the probability (in terrms of
p)
that the weather will be dry
on February 1st?
or n
Задача 5.34.
easy
Задача 5.35. Мабиногские овцы без пастуха
1
Каждая партия может закончится выигрышем в 1 очко, с вероятностью ; выигрышем в 2 очка,
4
1
с вероятностью
или проигрышем. Петр играет до первого проигрыша. Какова вероятность того,
2
что он накопит n очков?
And on one side of the river he saw a flock of white sheep, and on the other a flock of black sheep. And
whenever one of the white sheep bleated, one of the black sheep would cross over and become white; and
when one of the black sheep bleated, one of the white sheep would cross over, and become black.
Welsh tale «Peredur the Son of Evrawc» (english translation by Lady Charlotte Guest in «The
Mabinogion»)
Когда Передур подъехал к Мабиногским овцам, 40 были черными и 60 - белыми. Предположим, что
neti
roni
41
блеющая овца выбирается наугад из всего стада.
а) Докажите, что вероятность того, что все овцы рано или поздно станут черными равна вероятности
того, что при 99 подбрасываниях монетки, она выпадет орлом не более 39 раз.
б) Оцените вероятность того, что все овцы рано или поздно станут черными
Задача 5.36.
A queue of
(n + m)
people is waiting at a box office;
n
of them have 5-pound notes and
m
have 10-pound
notes. The tickets cost 5 pounds each. When the box office opens there is no money in the till. If each
customer buys just one ticket, what is the probability that none of them will have to wait for change?
Source: drmath forum
Задача 5.37. [Ross, Probability models]
Игроки по очереди подбрасывают монетку. Выигрывает тот, кто первым выбросит «орла». Монетка
неправильная, и «орел» выпадает с вероятностью
p.
Пусть
f (p)
- вероятность выигрыша того, кто
бросает первым.
Попробуйте интуитивно (не находя
f (p))
определить:
f (p) монотонной?
limp→0 f (p) и limp→1 f (p)
Найдите f (p) и проверьте интуицию
а) Является ли
б)
в)
Задача 5.38. [Problem of the points]
Саша и Маша играют в крестики-нолики до
n
побед (не обязательно подряд). Отдельную партию
p, Маша - с вероятностью 1 − p. Пусть Саше осталось победить s
f (s, m) ожидаемое количество оставшихся партий и P (s, m) - вероят-
Саша выигрывает с вероятностью
раз, а Маше -
m.
Обозначим
ность того, что победит Саша
f (s, m) через f (s − 1, m) и f (s, m − 1)
b) Найдите f (s, 1) и f (1, m)
в) Как выражается P (s, m) через P (s − 1, m) и P (s, m − 1)
c) Найдите P (s, 1) и P (1, m)
а) Как выражается
Comment: No Closed form solution!
Задача 5.39.
Имеется неправильная монетка, выпадающая орлом с вероятностью
p.
Сколько подбрасываний
монетки в среднем потребуется для того, чтобы:
а) Орел выпал два раза подряд
б) Орел выпал два раза не обязательно подряд
в) Какая-нибудь грань выпала два раза подряд
г) Какая-нибудь грань выпала два раза не обязательно подряд
Задача 5.40.
A gambler flips a fair coin and wins a dollar for every heads and loses a dollar for every tail. His strategy
is to stop playing if – a) He makes X dollars, or b) he has played N times (whichever occurs first).
On an average how many times does he flip the coin.
Задача 5.41.
Сколько подбрасываний кубика в среднем потребуется, чтобы: а) Какая-нибудь грань выпала два
раза подряд?
б) Произведение двух подряд выпавших граней оказалось меньше 6?
Задача 5.42.
Вася дошел до последнего уровня игры. У него осталось
ность убить которого с одного выстрела равна
0.2.
40
патронов, а перед ним монстр, вероят-
Сколько выстрелов (в среднем) сделает Вася?
hoot
riad
rnih
etti
42
Задача 5.43. Дополнительные патроны
Вы в тире, и у Вас 100 патронов. С вероятностью
0.01
Вы попадает в глаз Усамы Бен Ладена, за
что получаете 20 дополнительных патронов, с вероятностью
0.05
Вы попадаете в нос Усамы Бен
Ладена, за что получаете 5 дополнительных патронов. Вы стреляете до тех пор, пока патроны не
кончатся. Сколько в среднем Вы сделаете выстрелов?
Source: www.wilmott.com-forum-brainteasers
Задача 5.44.
Consider a random binary tree. Let p be a fixed parameter between 0 and 1. Starting with the complete
infinite binary tree retain each edge randomly and independently with probability p. Our random binary
tree is the portion connected to the root. So for example the binary tree consisting of the root alone will
2
be selected with probability (1 − p) (ie when neither edge out of the root is retained).
Some of these random binary trees will contain an infinite number of vertices. Throw these out. Then
what is the expected number of vertices as a function of p of a random binary tree selected in this way.
Solution:
Case p<0.5
For p < 0.5 the infinite tree clause is a red herring, since the probability of an infinite tree is 0.
Let E be the expected size of the tree connected to the root. E is also therefore the expected size of the
left subtree and also of the right subtree. So
2
E = 1 ∗ (1 − p) + 2(1 + E)p(1 − p) + (1 + 2E)p2
Solving for E we get E = 1/(1 − 2p)
Case p>0.5
Пусть
Tk
- событие, состоящее в том, что осталось дерево с
k
P (Tk ) = pk−1 (1 − p)k+1
вершинами.
(просто считаем число оставшихся и число удаленных ребер для такого дерева)
P
p < 0.5, то
P (Tk ) = 1.
Поэтому при p > 0.5 (что равносильно
Если бы
перемене мест
p
и
1 − p)
получаем
P
P (Tk ) = ( 1−p
)2
p
The
conclusion we can draw is that C is in fact the probability of getting a finite tree for p > 1/2 and since we
are only interested in finite subtrees as final outcomes, Bayes lets us forget the scaling factor and we get
1
the same probability distribution as for p < 1/2. Therefore the answer in the case of p > 1/2 is E =
2p−1
Case p=0.5
Infinity by continuity?
Задача 5.45.
Маша и Саша подбрасывают неправильную монетку, выпадающую орлом с вероятностью
выигрывает, если серия из
событие «Маша выиграла»,
k
B
орлов подряд появится раньше серии из
p.
Маша
решек. Обозначим
A
-
- сначала выпал орел.
а) Составьте два уравнения, связывающих
b) Найдите
r
P (A|B)
и
P (A|B c )
P (A)
Задача 5.46.
В коробке 123 черных шара и 321 белый шар. Извлекается наугад пара шаров. Если оба шара
одного цвета, то место пары кладется белый шар. Если извлеченная пара шаров разного цвета, то
вместо нее кладется черный шар. Так повторяется до тех пор, пока в корзине не останется один
шар. Какова вероятность того, что он будет черным?
Задача 5.47. Спагетти
На тарелке запутавшись и вконец устав перед казнью лежат
n
спагетти. Мы связываем концы
спагетти между собой наугад.
а) Каково ожидаемое количество получающихся колец?
б) Каково ожидаемое количество колец длиной в одну спагетти?
Задача 5.48.
Два игрока по очереди подбрасывают кубик. Выигрывает тот, кто первым выбросит шестерку.
ilki
pers
43
а) Каковы шансы первого игрока на победу?
б) Рассчитайте шансы каждого игрока на победу, если игроков трое
Задача 5.49. Испытания до первого успеха
Сколько в среднем раз надо бросать кость до появления шестерки?
Задача 5.50.
В один конверт кладут сумму
10N , в другой - 10N +1 , где N
- грань выпавшая на кубике. Петя и Вася
берут конверты наугад и каждый открывает свой. Далее их по очереди (Петя-Вася-Петя-Вася-...)
спрашивают «хотел бы ты заплатить 1 рубль за возможность поменяться конвертами?». Реального
обмена не происходит. Им просто по очереди несколько раз задают один и тот же вопрос. Каждый
слышит все предыдущие ответы и говорит правду. Как они будут отвечать?
Задача 5.51.
На столе стоят
n
копилок. Достать содержимое копилки можно двумя способами: либо разбить
копилку, либо открыть дно специальным ключиком. У каждой копилки свой ключик. Мы раскладываем ключи по копилкам наугад (один ключ в одну копилку). Затем разбиваем две копилки.
Далее разбивать копилки запрещается.
а) Какова вероятность того, что мы сможем достать все ключи?
б) Как изменится ответ, если изначально разбить не две, а
k
в) Сколько в среднем можно добыть ключей, если разбивать
копилок?
k
копилок?
г) Какая доля ключей в среднем будет найдена, если разбивать
k
копилок?
Задача 5.52. Копилки-2
На столе стоят
n
копилок. Достать содержимое копилки можно двумя способами: либо разбить
копилку, либо открыть дно специальным ключиком. У каждой копилки свой ключик. Мы связываем ключи в несколько связок. Затем связки ключей раскладываются по копилкам наугад (часть
копилок остается пустыми, в остальных - по одной связке). Затем разбиваем две копилки. Далее
разбивать копилки запрещается.
а) Какова вероятность того, что мы сможем достать все ключи?
б) Как изменится ответ, если изначально разбить не две, а
k
копилок?
в) Сколько в среднем можно добыть ключей, если разбивать одну копилку?
[unsolved,
вроде уже
решилась]
Задача 5.53.
В вершинах треугольника три ежика. С вероятностью
гается по часовой стрелке, с вероятностью
(1 − p)
p
каждый ежик независимо от других дви-
он двигается против часовой стрелки. Сколько в
среднем пройдет времени прежде, чем они встретятся в одной вершине?
При каком
p
ожидаемое время встречи минимально?
Solution:
У системы 4 состояния (1-1-1, 1-2-0, 2-1-0, 3-0-0).
Пишем три уравнения на ожидаемые времена.
3
Решая находим E(T ) =
p(1−p)
Минимум при p = 0.5
Ответ слишком красивый... красивое решение????
Задача 5.54.
Монетка подбрасывается 16 раз. Какова вероятность того, что ни разу не выпадет ни три орла
подряд, ни две решки подряд?
Задача 5.55. Стопка газет
У Пети стопка из
n номеров газеты «Вышка» лежащих в случайном порядке. Петя сортирует газеты
44
следующим образом. Он последовательно просматривает стопку сверху вниз. Если просматриваемый выпуск более свежий, чем лежащий сверху стопки, то Петя перекладывает более свежий
выпуск наверх стопки и начинает просматривать стопку заново.
Сколько «переносов» более свежих номеров наверх в среднем будет сделано до того момента, когда
наверху окажется первый выпуск газеты?
Source:
http://www.artofproblemsolving.com/Forum/viewtopic.php?t=124903
Задача 5.56.
(n − 1) рыцарь. Король Артур подбрасывает игральный кубик. Если кубик выпадает на 1 или 2, то он объявляется победителем. Если на 3 или 4, то
кубик передается соседу слева. Если на 5 или 6, то кубик передается соседу справа. Подкидывания
За Круглым столом сидит Король Артур и еще
и передача кубика повторяются до тех пор, пока не определится победитель.
а) Найдите вероятность победы Короля Артура для
б) Найдите
n
рыцарей.
limn→∞
Задача 5.57.
Жестокий тиран издал новый указ. Отныне за каждого новорожденного мальчика семья получает
денежную премию, но если в семье рождается вторая девочка, то всю семью убивают.
Предположим, что бедные жители страны остро нуждаются в деньгах, но не хотят рисковать своей
жизнью.
а) Какой будет доля мальчиков в стране?
б) Каким будет среднее число детей в семье?
в) Какой будет средняя доля мальчиков в семье?
г) Какой будет доля семей, с одним ребенком?
Задача 5.58.
Пусть
Lt
- число точек, которые симметричное случайное блуждание посетит ровно один раз к
моменту времени
Найдите
t.
E(Lt ).
Задача 5.59.
Сколько раз нужно подбрасывать монетку до появления
n
орлов подряд?
Задача 5.60.
Xi be independent and
1|X1 + X2 + . . . + Xn > x).
Find E(X), V ar(X).
Let
uniformly distribuited on
(0; 1)
. Let
x ∈ (0; 1)
and define
N = min(n ≥
Solution 1:
P (X = n) = P (x1 + . . . + xn−1 ≤ x) − P (x1 + . . . + xn ≤ x)
По индукции можем показать, что P (x1 + . . . + xn ≤ x) =
n
xn−1
Поэтому P (X = n) =
− xn!
(n−1)!
xn
n!
Далее - ряд Тейлора:
E(N ) = exp(x), V ar(x) = exp(x)(1 + 2x) − exp(2x)
Solution 2:
a денег. Обозначим N (a)
g(a) = E(N 2 (a)).
Пусть сейчас накоплено
f (a) = E(N (a))
и
- число оставшихся шагов. Введем две функции
Методом первого шага можно получить уравнения:
Rx
f (a) = a fR(t)dt
Rx
x
g(a) = 1 + a g(t)dt + 2 a f (t)dt
И начальные условия f (x) = g(x) = 1
Далее перейти к дифурам, решить их.
45
Уравнения с интегралами можно пояснить дискретным случаем.
Задача 5.61.
Three players A, B and C take turns to roll a die; they do this in the order ABCABC...
(a) Show that the probability that, of the three players, A is the first to throw a 6, B the second, and C
the third, is 216/1001.
(b) Show that the probability that the first 6 to appear is thrown by A, the second 6 to appear is thrown
by B, and the third 6 to appear is thrown by C, is 46656/753571.
Задача 5.62.
A spider moves on the eight vertices of a cube in the following way: at each step the spider is equally
likely to move to each of three adjacent vertices, independently of its past motion. Let
vertex occupied by the spider, and let
O
be the vertex opposite of
O
I
for the first time.
until the first return to
c) Calculate the expected number of steps until the first visit to
be the initial
I.
a) Calculate the expected number of steps until the spider returns to
b) Calculate the expected number of visits to
I
I.
O.
Ответ: 8, 1, 10, используйте симметрию куба
Задача 5.63.
Suppose Bob and Nick are playing a game. Bob rolls a die and continues to roll until he gets a 2. He keeps
track of all the outcomes of the rolls. He calls the sum of the outcomes of the rolls (including the last
outcome 2)
X.
Nick does the same thing as Bob, only he keeps rolling until he gets a 5 and he calls the
sum of the outcomes of his rolls (including the last outcome 5)
Y
dollars and Nick pays Bob
X
Y.
At the end of the game, Bob pays Nick
dollars. Does one of the boys have an advantage over the other? Which
one?
Ответ: игра справедливая! Перевес в сумме в точности компенсируется последним слагаемым!
Задача 5.64.
Миша должен перебрать ведро яблок (выкинуть гнилые). В ведре 12 яблок, из них 4 гнилых.
Миша выбирает яблоки следующим образом: достает по 3 яблока случающим образом, выкидывает
гнилые, а нормальные кладет обратно в корзину. Он действует, таким образом, до тех пор, пока не
достанет 3 не гнилых яблока. После этого он идет к дедушке и говорит, что все перебрал. Найдите
вероятность того, что дедушка возьмет на проверку гнилое яблоко (дедушка случайным образом
берет всего одно яблоко). source: банк задач первой олимпиады школьников
sol: (a better one?). Вероятность зависит о числа гнилых яблок в ведре. Получаем четыре неизвестных
p1 , p2 , p3 , p4 .
Они легко находятся начиная с
p1 .
Задача 5.65.
Миша предстоит перебрать ведро в котором
n
грибов. Среди грибов одна бледная поганка, осталь-
ные хорошие. Миша осматривает грибы по одному, доставая их наугад, и осмотренные откладывает
в сторону. Бледную поганку Миша определяет безошибочно.
а) Каково математическое ожидание и дисперсия числа осмотренных грибов?
б) Каково математическое ожидание числа осмотренных грибов, если бледных поганок две?
Задача 5.66.
n раз.
+ XK ?
Правильную монетку подбрасывают
a) (easy) Чему равно X1 + X2 + ...
2
2
б) Чему равно E(X1 + ... + XK )?
в) E(max(X1 , ..., XK )), asy=?
solution
n, 2n, cln(n)
ru-math
Пусть
X1 ,...XK
- длины последовательных серий.
46
Задача 5.67.
You pay 1$ to play a game, a fair coin is tossed and if heads you get 3$, if tails nothing. You start with
50$ and play as many times as you like. What is the probability that you will go bankrupt if you keep
playing?
Задача 5.68.
Imagine two people, each of whom tosses a coin repeatedly. Person A keeps tossing until the first time
three successive tosses come down head, tail, head. Person B tosses until they first get a head followed by
two tails.
On average, who will have to wait longer?
At what probability
p
(approximately) of a head are the two expectations equal?
Solutions:
E[TA ] = 10
E[TB ] = 8
3
3
We wanna equate (1 + p − p )/(p − p ) − 1 and
3
2
It simplifies to p − 2p − p + 1 = 0, p ≈ 0.5550
(p2 − 3p + 3)/(1 − p)2 + 1/p − 1.
Задача 5.69.
Монетку подрасывают до выпадения трех орлов подряд. Найдите ожидание и дисперсию числа
подбрасываний.
Задача 5.70.
A fair coin is tossed repeatedly. What is the average number of tosses required to obtain a head?
Answer:
2
Задача 5.71.
A die is rolled repeatedly. On average, how many times must the die be rolled to get a 6?
Answer:
6
Задача 5.72.
A couple decides to keep having children until they have at least one boy and one girl. What is the average
number of children they will have? (Assume no twins.)
Answer:
3
Задача 5.73.
I alternately toss a fair coin and throw a fair die until I either toss a head or throw a 2. If I toss the coin
first, what is the probability that I throw a 2 before I toss a head?
Answer:
1/7
Задача 5.74.
A fair coin is tossed repeatedly. What is the average number of tosses required to obtain two heads in a
row?
Answer:
6
Задача 5.75.
An apple is located at vertex
A
of pentagon
ABCDE ,
and a worm is located two vertices away, at
C.
Every day the worm crawls with equal probability to one of the two adjacent vertices. When it reaches
vertex
A,
it stops to dine. What is the average number of days until dinner?
Задача 5.76.
Five people stand at the vertices of a pentagon, throwing frisbees at each other. They have two frisbees,
initially at adjacent vertices. At each step, each frisbee is thrown either to the left or to the right with
equal probability. This process continues until one person is the target of two frisbees simultaneously;
47
then the game stops. What is the average number of steps taken?
Задача 5.77.
A, B , and C ,
0.5, and C never
A’s
Three men,
are to fight a pistol duel (truel?).
chance is
misses. They are to fire at their choice of target in succession in the
B, C ,
chance of hitting a target is
0.3, B ’s
order A,
cyclically (but a hit man loses further turns and is no longer shot at) until one man is left. What
should
A’s
strategy be?
Задача 5.78.
Erica flips a fair coin. Nate then flips the coin and wins if it matches Erica’s flip. If it doesn’t match, then
Noah flips the coin and wins if it matches Nate’s flip. If Noah doesn’t win, then Erica flips and wins if it
matches Noah’s flip. The game continues until there is a winner. What is the probability that Nate wins
the game?
Задача 5.79.
Two persons,
1 or 2;
Y
X
and
Y,
play with a die an unlimited number of times.
X
wins a game if the outcome is
wins the game in the other cases. A player wins the match if he wins two consecutive games.
Determine the probability that
X
wins the match.
Задача 5.80.
Daniel and Scott are playing a game where a player wins as soon as he has two points more than his
opponent. Both players start at zero, and points are earned one at a time. If Daniel has a
0.6
chance of
winning each point, what is the probability that he will win the game?
Задача 5.81.
Coupons are each randomly numbered 1 to 5. All five numbers are required to win a prize. On average,
how many coupons must be taken to win?
Задача 5.82.
Alice and Bob flip a coin until either HHT or HTT occurs. Alice wins if the pattern HHT comes first;
Bob wins if HTT comes first. What is the probability that Alice will win?
Задача 5.83.
A fair coin is tossed repeatedly. What is the probability that the first occurrence of three heads in a row
is earlier than the first occurrence of two tails in a row?
Задача 5.84.
A fair coin is tossed repeatedly. What is the probability that the first occurrence of HH will occur before
the first occurrence of THTH?
Задача 5.85.
A fair coin is tossed repeatedly. What is the average number of tosses required to obtain the sequence
THTTH?
Задача 5.86.
Player
A
has 1 dollar, and Player
other. Player
A
B
has 2 dollars. Each play gives one of the players 1 dollar from the
is enough better than Player
bankrupt. What is the chance that Player
A
B
that he wins 70% of the plays. They play until one is
wins?
Задача 5.87.
Two players,
A
and
result is heads, then
B,
A
start with
pays
B
a
and
b
dollars, respectively. A fair coin is repeatedly tossed. If the
one dollar. Otherwise,
player goes broke. What is the probability that
A
wins?
B
pays
A
one dollar. The game ends when one
48
Задача 5.88.
A bug is on a number line, starting at 0. A fair coin is tossed repeatedly. If the coin comes up heads, the
bug goes 1 unit left; otherwise, the bug goes 1 unit right. What is the probability that the bug eventually
returns to 0?
Задача 5.89.
From where the drunken man stands, one step toward the cliff would send him over the edge. He takes
random steps, either toward or away from the cliff. At any step, his probability of taking a step away is
2
1
, of a step toward the cliff . What is his chance of falling over the edge?
3
3
Answer: 0.5
Задача 5.90.
2
(and is
3
independent of the microbe’s ancestry), what is the probability that a microbes’s descendants die out?
A microbe either splits into two perfect copies of itself or dies. If the probability of splitting is
Answer:
0.5
Задача 5.91.
A and B , repeatedly play a game of chance. At each turn, either A wins 1 dollar from B with
0.51, or A loses 1 dollar to B with probability 0.49. Both A and B have unlimited capital.
Two players,
probability
What is
A’s
expected peak cumulative loss?
Задача 5.92.
A particle moves at each step 2 units to the right or 1 unit to the left, with probability
0.5
each. The
particle starts at 1. What is the probability that the particle will ever reach the origin?
Задача 5.93.
A coin is tossed repeatedly. What is the probability that at some point, the number of tails will be greater
than twice the number of heads?
Задача 5.94.
Real numbers are chosen at random from the unit interval
(0; 1).
If after choosing the
sum of the numbers so chosen first exceeds 1, what is the expected value of
N -th
number the
N -th
number the
N?
Задача 5.95.
Real numbers are chosen at random from the unit interval
(0; 2).
If after choosing the
sum of the numbers so chosen first exceeds 2, what is the expected value of
N?
Source: previous 26 problems, aops, t=38284
Задача 5.96.
A fair coin is tossed repeatedly. Find the probability of obtaining five consecutive heads before two
consecutive tails.
Задача 5.97.
A and B play a series of games. Each game is independently won by A with probability
probability
(1 − p).
p
and by B with
They stop when the total number of wins of one of the players is two greater than
that of the other player. The player with the greater number of total wins is declared the winner. Find
the probability that A is the match winner.
Source: aops, t=133655
Задача 5.98.
A and B are having a shooting contest. Each of them have 50 bullets. For each bullet, both A and B have
50% probability of hitting the target. After they finish shooting, the total numbers of bullets that hit
49
the target are counted. If A hit the target more than B, A wins. If they hit the target the same number
of times, it’s a tie. Otherwise, B wins. Apparently if A and B both get 50 shots, they have the same
probability of winning. The question is: what is the probability of A winning if A gets 51 shots and B gets
50 shots?
Solution:
1/2.
One way of seeing this is as follows: first look at what happens after both have done 50 shots. Either A
is ahead (with probability, say
(with probability, say,
q ).
p),
or B is ahead (which by symmetry, is also equal to p), or both are tied
So obviously,
2p + q = 1.
Now what are the ways A can win after 51 shots? He
can win if he’s already ahead after 50 shots (which happens with probability
p)
and then the outcome of
the 51st shot is irrelevant, or if he’s tied after 50 shots and has a success on the 51st shot (which occurs
with probability
q · 1/2).
(If A is behind B after 50, he either stays behind or at best ties at the 51st, and
if A and B are tied at 50, then they still stay tied if A misses his 51st shot.) Thus, the probability of A
winning is
p + q/2 = 1/2.
If you know some advanced probability theory, then a simpler argument, called coupling, says that without
loss of generality, you can assume that A and B have exactly the same hits and misses during their first
50 shots. Then A wins if and only if he succeeds on his 51st shot, which occurs with probability
1/2.
Note that 50 isn’t special here; this works for any n and n+1.
n+1
Another simplified version of this problem: If A flips
probability that A gets more heads than B is
and B flips
n
fair coins, what that the
1/2.
Задача 5.99.
1
. Все
2k+1
монеток подбрасываются одновременно. Какова вероятность, что выпадет нечетное число орлов?
Имеется
n
n
неправильных монеток. Монетка номер
k
выпадает орлом с вероятностью
pk =
Solution:
Pn = Pn−1 ·
2n−1
2n+1
+
1
и
2n+1
Pn =
n
2n+1
Задача 5.100. Вариация к началу 2008 года
Дед Мороз развешивает новогодние гирлянды. Аллея состоит из 2008 елок. Каждой гирляндой
Дед Мороз соединяет две елки (не обязательно соседние). В результате Дед Мороз повесил 1004
гирлянды и все елки оказались украшенными. Какова вероятность того, что существует хотя бы
одна гирлянда, пересекающаяся с каждой из других?
Например, гирлянда 5-123 (гирлянда соединяющая 5-ую и 123-ю елки) пересекает гирлянду 37-78 и
гирлянду 110-318. Например, на рисунке:
[picturehere]
зеленая гирлянда пересекается с красной и с синей, а красная и синяя не пересекаются
Задача 5.101.
Имеется
n
монеток. Каждая выпадает орлом с вероятностью
p.
В первом раунде подкидывают все
монетки. Во втором раунде подкидывают те монетки, которые не выпали орлом в первом раунде.
В третьем раунде подкидывают те монетки, которые не выпали орлом ни в первом, ни во втором
раунде. И т.д. до тех пор, пока будут монетки не выпадавшие орлом ни разу. Пусть
X
- число монет,
подброшенных в последнем раунде.
а) Как связаны
E(X)
и
P (X = 1)?
limn→∞ P (X = 1)?
б) Существует ли предел
Задача 5.102.
В озере сейчас живет одна амеба. Каждую минуту каждая амеба равновероятно превращается в 0,
1, 2 или 3 амебы.
а) Какова вероятность того, что популяция амеб в озере рано или поздно вымрет?
б) Сколько в среднем амеб в озере будет через
n
Answers:
а) Уравнение методом первого шага, корень
p=
минут?
√
2 − 1.
50
б)
1.5n
Задача 5.103. Убийцы и мирные граждане
Вы приехали в уездный город
N.
В городе кроме Вас живут
M
мирных граждан и
U
убийц. Каж-
дый день на улице случайным образом встречаются два человека. Если встречаются два мирных
гражданина, то они пожимают друг другу руки. Если встречаются мирный гражданин и убийца,
то убийца убивает мирного гражданина. Если встречаются двое убийц, то оба погибают.
Каковы Ваши шансы выжить в этом городе? Зависят ли они от Вашей стратегии?
Solution:
Если
U
нечетно, то шансы выжить равны нулю. Если
U
четно, то шансы выжить равны
1
U +1
Задача 5.104.
Правильная монетка подбрасывается бесконечное количество раз. Какова вероятность того, что
орлов подряд появятся раньше, чем
t
h
решек подряд?
Answer:
2t −1
2t +2h −2
Может какое устное решение? Ибо красивая формула?
First step,
Задача 5.105.
В углу доски стоит шахматная фигура. Мы двигаем ее наугад (согласно правилам хода для этой
фигуры). Сколько в среднем пройдет ходов прежде чем она вернется в исходную клетку, если эта
фигура:
а) король
б) конь
в) ладья
г) слон
д) ферьзь
Подсказка: находим стационарное распределение, обращаем вероятность
Ответы:
б) 168
Задача 5.106. A coin, which lands on heads with probability
expected number of flips required to get a string of
r
p
is continually flipped. Compute the
heads in a row.
Задача 5.107.
свободная группа, порожденная двумя элементами a и b. Т.е. G состоит из всех ’слов’
−1
−1
−1
−1
конечной длины, состоящих из ’букв’ a, b, a , b . ’Буквы’ a и a , также как и ’буквы’ b и b
Пусть
G
сокращаются. Имеется особое ’слово’-’единица’
’слово’
e.
e, e·a = a etc. В начальный момент времени написано
В каждый следующий момент справа дописывается одна из четырех ’букв’ (производится
сокращение, если это возможно; буквы не переставляются).
а) Какова вероятность того, что когда-нибудь снова будет написано
e?
б) Каков (as) предел средней длины слова?
Ответ:
а)
P = p + (1 − p)P 2
б)
1
2
Задача 5.108.
Пусть Sn - симметричное случайное блуждание. Т.е. S0 = 0, Sn = Sn−1 + Xn , где Xn - iid,
P (Xn = 1) = P (Xn = −1) = 0.5.
P (S2m =2r)
а) Найдите limm→∞
P (S2m =0)
б) Найдите limm→∞ P (S2m = 2r), limn→∞ P (Sn ≥ k)
в) Запись k|m означает, что k делит m (m делится на k ). Найдите limn→∞ P (2|Sn ) и limn→∞ P (3|Sn )
Solution.
51
а) 1
б) Исходя из п. а, получаем 0 (можно воспользоваться ф. Стирлинга);
и
P (Sn ≥ k) → 0.5
limn→∞ P (2|Sn ) не
в)
P (Sn ≥ 1) = P (Sn ≤ −1) → 0.5
существует;
Введем величины:
xn = P (3|Sn ), yn = P (3|Sn + 1)
Тогда: x0 = 1, y0 = 0, z0 = 0
xn = 0.5yn−1 + 0.5zn−1
yn = 0.5xn−1 + 0.5zn−1
zn = 0.5xn−1 + 0.5yn−1
и
zn = P (3|Sn + 2)
Решая систему разностных уравнений (λ1
= −0.5, λ2 = −0.5, λ3 = 1,
двум кратным корням соответ-
ствует два линейно независимых собственных вектора) находим, что
xn → 1/3.
Задача 5.109.
Sn - несимметричное случайное
P (Xn = 1) = p = 1 − P (Xn = −1).
Пусть
блуждание. Т.е.
S0 = 0, Sn = Sn−1 + Xn ,
где
Xn
- iid,
а) Найдите вероятность того, что случайное блуждание когда-нибудь вернется в исходную точку
б) Пусть
p > 0.5.
Найдите ожидаемое время выигрыша
r
рублей.
Задача 5.110.
[Steele,
1.1.]
Корпорация стремится поддерживать курс своих акций на уровне 20$. Поэтому вероятности изменения курса выглядят так:
P (Yn+1 = 21|Yn = 20) = 0.9 и P (Yn+1 = 19|Yn = 20) = 0.1
При k > 20: P (Yn+1 = k + 1|Yn = k) = 1/3 и P (Yn+1 = k − 1|Yn = k) = 2/3
При k < 20: P (Yn+1 = k + 1|Yn = k) = 2/3 и P (Yn+1 = k − 1|Yn = k) = 1/3
Найдите ожидаемое время падения курса акций с 25$ до 18$.
Solution.
x18 = 0
x19 = 2/3x20 + 1/3x18 + 1
x20 = 0.1x19 + 0.9x21 + 1
x21 = 2/3x20 + 1/3x22 + 1
x22 − x20 = 2(x21 − x20 )
x25 − x20 = 5(x21 − x20 )
Решая, получаем: x19 = 77, x20 = 114, , x21 = 117, x22 = 120, x25 = 129
Задача 5.111.
Для лотерии выпущено 10000 билетов, из них 100 билетов являются призовыми и еще 200 дают
право получить еще один билет. Какова вероятность получить приз, если купить один билет?
100
Ответ:
10000−200
Задача 5.112.
I roll 100 standard dice. I get one point for each die for which the number of dots on the top face is greater
than two. What is the probability that the number of points I get is...
a) Even? b) Divisible by three? c) =1 (mod 3)?
source: aops, t=111454
Задача 5.113. breaking sticks
Take a stick and break it at a location selected with uniform density along its length. Throw away the
left-hand piece and break the right-hand one at a location selected with uniform density along its length.
52
Continue forever. What is the probability that one of the discarded left-hand pieces is more than half as
long as the original stick?
Source: aops, t=152328
Solution:
The answer appears to be
log 2.
A possible proof could go as follows: Let
X1 ,X2 ,. . .
random variables with values in
[0; 1]
such that
Xn
is
P
[0, Xn−1 ]. We have to calculate p := n P (Xn ≥ 1/2).
R1
R 1−c
dyP (Xn ≥ c/(1 − y)) = c dz zc2 P (Xn ≥ z) it is easy to prove
Using the recursion P (Xn+1 ≥ c) =
0
P
P
(− log c)k
c)k
inductively that P (Xn ≥ c) = 1 − c
= c k≥n (− log
and thus interchanging the sums over
k<n
k!
k!
P
k
(− log c)
= − log c where c = 1/2.
n and k we get p = c(− log c)
k≥0
k!
distributed uniformly on
Задача 5.114.
Alfred and Bonnie play a game in which they take turns tossing a fair coin. The winner of a game is the
first person to obtain a head. Alfred and Bonnie play this game several times with the stipulation that
the loser of a game goes first in the next game. Suppose that Alfred goes first in the first game, and what
is the probability that he wins the sixth game?
source: aops, t=80995
Solution:
Вероятность выиграть, если ходить первым:
Пусть pn - вероятность выиграть n-ую
pn = 2/3 · (1 − pn−1 ) + 1/3 · pn−1
pn = 2/3 − 1/3 · pn−1 и p1 = 1
p = 0.5 + 0.5 · 0.5 · p; p = 2/3
партию (для А).
Задача 5.115. Consider a random walk on a fractal lattice (see attached picture):
Картинка: из центра 4 направления. Далее каждое направление делится на три, потом еще на три
и т.д. до бесконечности...
Particle starts at origin O.
At every time-step it moves in one of four directions, north south east or west, with equal probability.
The north-south generator does not commute with the east-west generator – e.g. east, north <> north,
east
Clearly the only way the particle can revisit a particular point is by retracing its steps.
Two questions:
1. What is the probability that the particle ever hits the point A?
2. What is the probability of hitting A before B?
Задача 5.116. Google page-rank
Поисковик google рассчитывает рейтинг страницы (pagerank) по следующему алгоритму: предполагается, что если на странице есть
n
ссылок, то пользователь с вероятностью 85% уходит на одну
из этих ссылок (выбирая саму ссылку равновероятно) и с вероятностью 15% уходит на случайно
выбираемую страницу. Рейтинг страницы - это вероятность того, что после длительного блуждания
пользователь окажется на данной странице.
Рассчитайте рейтинг для следующей сети
(картинка)
ссылка: википедия
Задача 5.117. Выборы
В выборах участвуют два кандидата. В начале выборов каждый из них голосует сам за себя.
Затем каждый из миллиона жителей по очереди голосует за одного из кандидатов. При этом если
голосов, а за кандидата В - m голосов, то вероятность того, что
n
.
n+m
Какова вероятность того, что за кандидата А проголосует менее 20% жителей?
за кандидата А было подано
n
очередной избиратель будет голосовать за А равна
Решение:
53
Процедуру голосования можно представить себе так:
Положим на прямую красный шар. Затем по одному будем класть белые шары (миллион штук).
Причем класть их будем равновероятно на любое место между уже положенными шарами, или с
любого края полоски.
В результате положение красного шара (отделяющего голоса за разных кандидатов) распределено
равновероятно.
0.2·106
Ответ:
≈ 0.2
106 +1
source: Prof. Vazirani’s Problems
Задача 5.118.
В коробке лежат 30 зеленых и 50 красных шаров. Мы извлекаем два наугад, один берем в левую
руку, другой - в правую. Шар в левой руке красим в цвет шара, находящегося в правой руке. Затем
возвращаем шары в коробку. Снова извлекаем два наугад и т.д. до тех пор пока в коробке все шары
не окрасятся в один цвет.
а) Какова вероятность того, что шары будут окрашены в красный цвет?
б) Сколько в среднем пар шаров будет извлечено?
Решение:
а) разностное уравнение с н.у., решение
p(k) = k/n,
где
k
- число красных шаров,
n
- общее число
шаров, может что по проще?
решение 2:
если
T
Kt
- число красных шаров - мартингал,
момент остановки, то
E(KT ) = k0 ,
значит
P (KT = n) · n = k0 .
Задача 5.119.
You have a black box with N balls in it - each of a different color. Suppose you take turns as follows randomly pick a ball in each hand, and paint the left hand ball the same color as the right hand ball. You
replace both balls in the box before the next turn.
How many turns do you expect before all balls are the same color?
Solution:
source: Ariel Landau in sci.math in September 1995
solution: Lew Mammel, Jr., Robert Israel
6. Reflection and other random walk
Задача 6.1.
За кандидата
A
подано
a
голосов, за кандидата
B
-
b
голосов,
a > b.
Во время подсчета голосов
бюллетени достают из урны по одному в случайном порядке. Какова вероятность того, что на
протяжении всего подсчета голосов кандидат
A
будет впереди кандидата
B?
Задача 6.2.
Рассмотрим симметричное случайное блуждание.
возвращения к
0.
Заметим, что
Nk
Nk
- количество посещений точки
может принимать значение
P (Nk > 0) = 21 k1
b) Докажите, что P (Nk > j + 1|Nk > j) =
c) Найдите E(Nk )
d) Найдите P (Nk = +∞)
k
до первого
+∞.
a) Докажите, что
1
2
+
1 k−1
2 k
e) Как изменятся ответы, если случайное блуждание будет несимметричным? (вероятности
q = 1 − p)
Solution.
P (Nk > j + 1) = P (Nk > j + 1 ∩ Nk > j) = P (Nk > j + 1|Nk > j)P (Nk > j)
E(Nk ) = P (Nk > 0) + P (Nk > 1) + P (Nk > 2) + ...
1
1
1
1
1 2
E(Nk ) = 2k
+ 2k
1 − 2k
+ 2k 1 − 2k
+ ... = 1
p
и
54
e)
E(Nk ) = (p/q)k
Source: Steele, 1.3.
Задача 6.3.
Частица движется по правильному многоугольнику из
m.
Стартует она в вершине
0.
m+1
вершин, занумерованных от
0
до
Следующая вершина выбирается равновероятно из двух соседних.
Частица останавливается, когда обойдет все вершины.
а) Какова вероятность того, что частица никогда не посетит вершину
k?
б) Какова вероятность того, что частица закончит свой маршрут в вершине
k?
Source: Ross, example 2.52
solution:
P ({частица никогда не посетит вершину k}) = 0
Ak = {частица закончит свой маршрут в k}
Bk = {частица когда-нибудь посетит одно из чисел k-1,k,k+1}
P (Ak ) = P (Ak ∩ Bk ) = P (Bk ) · P (Ak |Bk ) = 1 · P (Ak |Bk )
А вероятность P (Ak |Bk ) - это вероятность того, что симметричное случайное блуждание поднимется
1
на m − 1 раньше, чем опустится до −1. Значит P (Ak |Bk ) =
m
Задача 6.4.
Пусть
Sn
- симметричное случайное блуждание, а
принципа отражения найдите
τ
- время первого посещения точки 1. С помощью
P (τ = 2k − 1).
Solution.
P (τ = 2k − 1) = P (S2k−1 = 1 ∩ {St < 1|t < 2k − 1})
Рассмотрим случай 2k − 1 ≥ 3. Тогда предыдущие два шага частица сделала вверх, что происходит
−2
с вероятностью 2 .
P (S2k−1 = 1 ∩ {St < 1|t < 2k − 1}) = 2−2 · P (S2k−3 = −1 ∩ {St < 1|t < 2k − 3})
Требуется вероятность закончить в точке −1, не касаясь точки 1. Траекторий, касающихся 1, и
заканчивающихся в −1 ровно столько сколько траекторий, заканчивающихся в 3. Следовательно,
k−1
k
P (τ = 2k − 1) = 2−2 (P (S2k−3 = −1) − P (S2k−3 = 3)) = 2−(2k−1) C2k−3
− C2k−3
После арифметических преобразований получаем:
1
k
P (τ = 2k − 1) = 2−2k 2k−1
C2k
Эта формула верна и для 2k − 1 < 3
7. Geometric probability
Задача 7.1.
На бумаге проведена прямая. На бумагу бросают иголку. Какова вероятность, что острый угол
между прямой и иголкой будет меньше 10 градусов?
Задача 7.2.
Вася бегает по кругу длиной 400 метров. В случайный момент времени он останавливается. Какова
вероятность того, что он будет ближе, чем в 50 м от точки старта? Дальше, чем в 100 м?
Задача 7.3.
X и Y - абсцисса и орP (X < 0, 75), P (X ≤ a) для произвольного a, P (X > 0, 5|X + Y > 0, 5),
P (X + Y > 0, 5|X > 0, 5), P (X · Y > 1/3|X + Y < 2/3), P (X > 0, 3|Y < 0, 7)
Внутри единичного квадрата равновероятно выбирается одна точка. Пусть
дината этой точки. Найдите
Задача 7.4.
Внутри единичного квадрата выбирается точка наугад. Какова вероятность того, что она будет
ezke
nete
55
ближе к центру, чем к любой из вершин?
Задача 7.5.
A circle has 2006 points chosen so that the arcs between any two adjacent points are equal. Three of these
points are chosen at random.
a) Find the probability that the triangle is right
b) Find the probability that the triangle formed is isosceles
Solution:
a = b = 2004·1003
3
C2006
Задача 7.6.
На отрезке [0;1] (равномерно и независимо друг от друга) выбираются две точки.
а) Какова вероятность того, что расстояние между ними не более 0,25?
б) Какова вероятность, что из трех частей, на которые они разбили отрезок, можно сложить
треугольник?
Задача 7.7.
На планету (окружность с центром
O,
НЕ круг) сели три корабля, координаты их посадки неза-
висимы и равномерно распределены по окружности. Два корабля
другом, если
A
и
B
могут связаться друг с
∠AOB < π/2.
а) Какова вероятность того, что между кораблями будет связь (возможно непрямая)?
б) Какова вероятность того, что каждый корабль сможет добраться до базы? Запаса горючего
хватает, чтобы проехать расстояние равное радиусу.
в) Какова вероятность того, что наименьший суммарный расход горючего, необходимый для сбора
всех кораблей в одной точке, будет меньше 0,25? Единицы горючего хватает, чтобы один корабль
объехал всю планету.
в) Решите аналогичные а-б задачи в трех измерениях.
Задача 7.8.
Two points on the surface of a sphere are drawn uniformly at random. a) Expected distance?
b) Maximum pdf distance?
solution:
the p.d.f of distance is
sin(x/R). take the mode of this function you get pi/2
The second point chosen lies on a circle with the first point. The maximum distance between the two is
one-half the circumference of this circle, i.e. PI*R, and assuming unit radius, the maximum distance is
PI. Half of the second points chosen will lie at a distance longer than PI/2 and half will lie at a distance
less than PI/2. Thus, the expected distance is PI/2.
Задача 7.9.
A, B , C , D, E и F . Какова вероятность того, что треABC и DEF не пересекаются?
3
Solution. Точки ABC должны идти подряд. Есть 6 способов выбрать три точки подряд и C6 выбрать
6
три точки без ограничений. Итого, 3 = 0.3
C
На окружности наугад выбираются точки
угольники
6
Задача 7.10.
В круге радиуса
r
случайным образом (равномерно) выбирается точка. Пусть
X
- расстояние от
точки до центра круга.
а) Найдите функцию плотности
b) Найдите
X
E(X)
Задача 7.11.
В окружность радиуса
r
вписан правильный
мерно) выбирается точка. Пусть
X
n-угольник.
Внутри него случайным образом (равно-
- расстояние от точки до ближайшей стороны
n-угольника.
56
а) Найдите функцию плотности
b) Найдите
c) Найдите
X
E(X)
limn E(X)
Задача 7.12.
На окружности случайным образом выбираются 3 точки. Эти три точки являются вершинами
треугольника.
а) Какова вероятность того, что центр окружности лежит внутри построенного треугольника?
б) Какова вероятность, что треугольник остроугольный?
Solution1:
Одну точку зафиксируем. Вместо двух других точек выберем две оси. На двух осях есть 4 варианта
выбора точек.
1
4
Solution2:
Рисуем множество на плоскости, ищем площадь.
Solution3:
Пишем интегралы.
Solution of b:
Это один и тот же вопрос.
Задача 7.13.
На окружности случайным образом выбираются
n
точек.
a) Какова вероятность того, что центр окружности лежит внутри многоугольника с вершинами в
этих точках?
б) Какова вероятность того, что эти точки можно накрыть дугой с углом
α = 2π · t,
где
t ∈ [0; 0.5]?
в) Изменится ли ответ задачи, если точки выбираются не на окружности, а на плоскости, так что
все углы равновероятны и вероятность попадания в начало координат равна 0?
c) In the border of a perfectly circular piece of wood we choose N points at random to place legs and
make a table. What is the probability that the table will stand without falling?
Solution of a: это b для случая
t = 1/2
Solution b1:
p(n, t) = n · tn−1 , Для левой точки имеется n вариантов, при заданной левой точке,
(их (n − 1)) должны попасть в отрезок длины 2π · t при длине окружности 2π .
остальные точки
Solution b2:
n(n − 1) способов выбрать левую и правую границу.
Оставшиеся (n − 2) точки должны попасть между ними.
Имеется
Получаем интеграл (перебираем все расстояния между границами):
Rt
n(n − 1)an−2 da = n · tn−1 Solution v: no
0
Задача 7.14.
На окружности случайным образом выбираются точки, до тех пор пока многоугольник, образуемый
точками не будет содержать центр окружности.
Каково ожидаемое количество сторон у такого многоугольника?
Задача 7.15.
На окружности случайным образом выбираются точки, до тех пор пока длина минимальной дуги
их накрывающей не станет больше
2π · t,
где
t ∈ [0; 0.5]
Каково ожидаемое количество точек?
Solution:
n точек лежать внутри дуги, получаем сумму:
1
E(X) = P (X ≥ 1) + P (X ≥ 2) + P (X ≥ 3) + ... = 1 + t0 + 2t1 + 3t2 + ... = 1 + (1−t)
2
Исходя из задачи про вероятность для
-gon
side
pple
57
Задача 7.16.
[0; 1]
На отрезке
случайным образом выбираются
накрыть отрезком длины
n
точек. Какова вероятность, что их можно
t?
Задача 7.17.
Имеется правильный
(2n + 1)-угольник.
Наугад выбираются три различные точки.
a) Какова вероятность того, что центр многоугольника лежит внутри треугольника с вершинами в
выбранных точках?
b) Чему равен предел этой вероятности?
Задача 7.18.
На сфере случайным образом выбираются 4 точки. Эти четыре точки являются вершинами пирамиды. Какова вероятность того, что центр сферы лежит внутри построенной пирамиды?
Одну точку зафиксируем. Вместо трех других точек выберем три оси. На трех осях есть 8 вариантов
выбора точек.
Задача 7.19.
Вася отмечает на яблоке
n точек случайным образом. Затем Петя пытается разрезать яблоко на две
половинки так, чтобы все точки лежали в одной половинке.
Какова вероятность того, что Пете удастся это сделать?
Source: Bay Area Math Meet, Test of Ingenuity 1999 - Problem 20
Задача 7.20.
На окружности (не на круге!) выбирается равномерным образом две точки. Каково ожидаемое
расстояние между ними?
8
у меня получилось
, не перепроверял
2π
Задача 7.21.
На круге выбирается равномерным образом две точки. Каково ожидаемое расстояние между ними?
Страшный интеграл или...[?]
Задача 7.22. IBM, Ponder this
На плоскости взят произвольный треугольник. Внутри него равномерно и независимо друг от друга
выбираются 3 точки. Эти три точки образуют новый треугольник. Каково ожидаемое отношение
площади нового треугольника к площади исходного?
Задача 7.23. IBM, Ponder this
На плоскости взят произвольный квадрат. Внутри него равномерно и независимо друг от друга
выбираются 3 точки. Эти три точки образуют треугольник. Каково ожидаемое отношение площади
треугольника к площади исходного квадрата?
Задача 7.24.
Пусть
X
и
Y
независимы и равномерны на отрезке
[0; 1].
Найдите вероятность того, что
X
будет
Y
ближе к четному числу, чем к нечетному.
Решение:
Графически легко получить сумму бесконечного ряда. Она равна
5−π
.
4
Подсказка: Да поможет вам святой арктангенс!
source: Putnam 1993
Задача 7.25.
Св.
b
распределена равномерно на отрезке
[0; 1].
На плоскости проводится прямая
y = bx.
С какой
58
вероятностью она отрезок с концами в точках
(2; 1)
и
(4; 1)?
Задача 7.26.
и Y независимы и равномерны на отрезке [0; 1]. Найдите вероятность того, что в десятичной
X
первой ненулевой цифрой будет единица?
записи
Y
1
Решение: Графически на квадрате получаем две суммы геометрической прогрессии. p = .
3
Т.е. единица более вероятна чем другие числа. Это как-то связано с распределением Бенфорда.
Пусть
X
Link: http://en.wikipedia.org/wiki/Benford’s_law
Source: http://polymathematics.typepad.com/polymath/2007/10/initial-1s.html
Задача 7.27. Benford’s law
Рассмотрим поподробнее ценники в супермаркетах. Предположим, что вероятность того, что цена
начинается на цифру
n (n ∈ {1, 2, 3, ..., 9})
одинакова во всех странах и зависит только от
n.
Иными
словами, если перевести цены из одной валюты в другую, вероятность того, что на первом месте
стоит цифра
n
не должна измениться.
а) Найдите вероятность того, что цена начинается с цифры 1; б) Найдите вероятность того, что
цена начинается с цифры 9; Comment: доработать... lim? hints?
Задача 7.28.
Three sides of a regular hexagon are chosen at random, and their midpoints are connected. Find the
probability of the resulting triangle being right.
Answer: 0.6
Source: aops, t=172334
Задача 7.29.
A stick is broken into
n
pieces. If three of these pieces are chosen at random, what is the probability that
they form a triangle?
Solution: (n
= 3):
suppose the length of two points is x from one end and y from anohther end. The length of middle portion
would be 1-(x+y), You can plot these x, and y on cartesian axes with x and y < 1 and following set of
constraints: x+y>1-(x+y)
x+1-(x+y)>y
y+1-(x+y).>x
when you plot these three constraints on the cartesian system then you will see that the feasible area
for traingle formation is 1/8 while the feasible area to break a unit length into three parts is 1/2. Hence
Probablity is =1/4
Задача 7.30.
Given a stick of unit length. Break this in two pieces at random. Then break the longest piece in two
pieces at random. What is the probability that you can build a triangle from the three resulting pieces?
Задача 7.31.
У Васи листочек в клеточку размером 5 на 4. Вася наугад отмечает четыре различных узла (может
отметить и крайние). Далее Вася закрашивает выпуклый четырехугольник, образованный этими
вершинами.
Какова вероятность того, получится прямоугольник со сторонами параллельными линиям сетки?
Задача 7.32.
На отрезке
[0; 1] равномерно, независимо друг от друга выбираются 2 точки. Они разбивают отрезок
на три части. Какова средняя длина наименьшей части?
Решение:
можно через интеграл, можно геометрически посчитать объем, получаем
1
9
59
8. Пуассон, экспоненциальное и поток
Задача 8.1.
Число человек, заглянувших в магазин, распределено по Пуассону с параметром
посетителей делает покупку с вероятностью
p.
λ.
Каждый из
Как распределено число покупок?
Задача 8.2. Пуассоновский курятник
Число яиц
X,
которые снесет курочка Ряба, распределено по Пуассону с параметром
яйцо оказывается золотым с вероятностью
Каждое
p.
a) Как распределено количество золотых яиц,
a’) Простых яиц,
λ.
G?
N?
b) Какова вероятность того, что Ряба снесет хотя бы одно простое яйцо и ни одного золотого?
в) Верно ли, что количество простых и золотых яиц независимы?
E(G|X)?
d) E(G)
e) E(X|G)
с)
Задача 8.3.
Предположим, что кузнечики на большой поляне распределены по пуассоновскому закону с
λ=3
на квадратный метр. Какой следует взять сторону квадрата, чтобы вероятность найти в нем хотя
бы одного кузнечика была равна
0, 8?
Задача 8.4.
Маршруты А и Б ходят независимо друг от друга. Вася приходит на остановку в случайным момент
времени и садится в первый попавшийся. Сколько в среднем ему приходится ждать в случае:
а) автобусы ходят регулярно, А через каждые 10 минут, Б - через каждые 15 минут
б) время между двумя автобусами одного маршрута имеет экспоненциальное распределение, с
ожиданием 10 и 15 минут соответственно.
Задача 8.5.
Пусть
N,
количество телефонных звонков, поступающих в фирму за сутки, имеет распределение
Пуассона с параметром
λ = 5.
Найдите
P (N = 3), P (N > 3)
и
P (N = 0), E(N ).
Задача 8.6.
Машины, проезжающие мимо поста ГИБДД на юг представляют собой пуассоновский поток событий с
λS = 200
машин в день. Машины, проезжающие на север - пуассоновский поток с
λN = 170
машин в день. Предположим, что эти потоки независимы.
Каков средний интервал времени между машинами (без учета направления движения)?
Задача 8.7.
Дождь - пуассоновский поток (дождь - поток - хм?..) с параметром
λ
капель на квадратный метр.
Масса капли =.... Вася держит кастрюлю с площадью основания. Ожидаемое количество воды за
два часа? Вероятность ни одной капли в течении 2 минут и т.д.
Задача 8.8.
Маша и Саша пошли в лес по грибы
2
Саша собирает все грибы, а Маша - только подберезовики.
Саша в среднем находит один гриб за одну минуту, Маша - один гриб за десять минут.
a) Какова вероятность того, за 10 минут они найдут ровно 10 грибов?
b) Какова вероятность того, что следующий гриб попадется позже, чем через минуту, если Маша
2Предполагается, что нахождение грибов, равно как отправка смс и нарушители представляют собой Пуассоновский
поток событий.
60
только что нашла подберезовик?
Задача 8.9.
Оля и Юля пишут смс Маше. Оля отправляет Маше в среднем 5 смс в час. Юля отправляет Маше
в среднем 2 смс в час. Какова вероятность того, что Маша получит ровно 7 смс за час? Сколько
времени в среднем проходит между смс, получаемыми Машей от подруг?
Задача 8.10.
Пост майора ГИБДД Иванова И.И. в среднем ловит одного нарушителя в час. Найдите вероятности
событий. Два нарушителя появятся с интервалом менее 30 минут. Следующего нарушителя ждать
еще более 40 минут, если уже целых три часа никто не превышал скорость.
Задача 8.11. [т]
Маленький мальчик торгует на улице еженедельной газетой. Покупает он ее по 15 рублей, а продает
по 30 рублей. Количество проданных газет в неделю - с.в. с распределением Пуассона и средним
значением равным 50. Пусть
N
- оптимальное количество газет. Чему примерно должно быть равно
значение функции распределения в точке
N?
С помощью компьютера найдите
N.
Задача 8.12.
Допустим, что время жизни лампочки распределено экспоненциально. Какова вероятность того,
что лампочка со средним временем жизни
b,
a погаснет раньше лампочки со средним временем жизни
если они были включены одновременно?
Задача 8.13.
Times to gather preliminary information from arrivals at an outpatients clinic follow an exponential
distribution with mean 15 minutes. Find the probability, for a randomly chosen arrival, that more than
18 minutes will be required.
Задача 8.14.
Пусть случайные величины
λY = 15 соответственно.
что X + Y = 50.
и
X
и
Y
независимы и распределены по Пуассону с параметрами
Найдите условное распределение случайной величины
X,
λX = 5
если известно,
Задача 8.15. [Пуассоновское приближение]
а) В гирлянде 25 лампочек. Вероятность брака для отдельной лампочки равна 0,01. Какова вероятность того, что гирлянда полностью исправна?
б) По некоему предмету незачет получило всего 2% студентов. Какова вероятность того, что в
группе из 50 студентов будет ровно 1 человек с незачетом?
в) Вася испек 40 булочек. В каждую из них он кладет изюминку с
p = 0, 02.
Какова вероятность
того, что всего окажется 3 булочки с изюмом?
Задача 8.16.
Вася каждый день подбрасывает монетку 10 раз. Монетка с вероятностью 0,005 встает на ребро.
Используя пуассоновскую аппроксимацию, оцените вероятность того, что за 100 дней монетка
встанет на ребро ровно 3 раза.
Задача 8.17.
Страховая компания «Ой» заключает договор страхования от «невыезда» (не выдачи визы) с туристами, покупающими туры в Европу. Из предыдущей практики известно, что в среднем отказывают
в визе одному из 130 человек. Найдите вероятность того, что из 200 застраховавшихся в «Ой»
туристов, четверым потребуется страховое возмещение.
61
Задача 8.18.
Вася, владелец крупного Интернет-портала, вывесил на главной странице рекламный баннер. Ежедневно его страницу посещают 1000 человек. Вероятность того, что посетитель портала кликнет по
баннеру равна 0,003. С помощью пуассоноского приближения оцените вероятность того, что за один
день не будет ни одного клика по баннеру.
Задача 8.19.
Время устного ответа на экзамене распределено по экспоненциальному закону. В среднем на ответ
одного студента уходит 10 минут.
а) Какова вероятность того, что Иванов будет отвечать более получаса?
б) Какова вероятность того, что Иванов будет отвечать еще более получаса, если он уже отвечает
15 минут?
в) Сколько времени в среднем длится ответ одного студента?
Решение:
R +∞
а)
p(t)dt = e−3 ≈ 0.05
30
б) такой же результат, как в «а»
в)
1/λ = 10
Задача 8.20.
Время между приходами студентов в столовую распределено экспоненциально; в среднем за 10 минут приходит 5 студентов. Время обслуживания имеет экспоненциальное распределение; в среднем
за 10 минут столовая может обслужить 6 студентов.
а) Как распредено количество в очереди?
б) Какова средняя длина очереди?
Подсказка: если сейчас в очереди
n
человек, то через малый промежуток времени
dt
в очереди
может оказаться...
Ответ:
а) геометрическое распределение
λin
б) E(N ) =
λcapacity −λin
Задача 8.21.
Годовой договор страховой компании со спортсменом-теннисистом, предусматривает выплату страхового возмещения в случае травмы специального вида. Из предыдущей практики известно, что
вероятность получения теннисистом такой травмы в любой фиксированный день равна 0,00037. Для
периода действия договора вычислите
а) Наиболее вероятное число страховых случаев
б) Математическое ожидание числа страховых случаев
в) Вероятность того, что не произойдет ни одного страхового случая
г) Вероятность того, что произойдет ровно 2 страховых случая
P.S. Указанные вероятности вычислите двумя способам: используя биномиальное распределение и
распределение Пуассона.
Решение:
«б»
365 · 0.00037 = 0.13505
Следовательно, «a», ближайшее целое равно 0.
Для Пуассоновского распределения:
365
в) P (N = 0) = 0.99963
≈ e−λ
2
363
г) P (N = 2) = C365 0.99963
0.000372
λ = 0.13505
≈ e−λ λ2 /2
Задача 8.22.
Showing independence Let X and Y be independent random variables with an exponential distribution
with parameters
λ1
and
λ2 .
Let
U = min{X, Y }
Show that U and W are independent.
and
V = max{X, Y }.
Also, let
W = V − U.
62
source: aops, t=174061
Задача 8.23. Renyi representation
Let
Xi
be iid random variables with exponential distribution of the same parameter
ordering of
Xi ,
Let
Xi , Y2 is the second smallest and so on.
Y1 , Y2 − Y1 , Y3 − Y2 ... are independent random variables with
parameters nλ, (n − 1)λ, (n − 2)λ ...
i.e.
Y1
λ.
be the
is the smallest value among
Prove that the spacings
distribution with
Yi
exponential
source: aops, t=174504
Задача 8.24.
The arrival of buses at a given bus stop follows Poisson law with rate 2. The arrival of taxis at the same
bus stop is also Poisson, with rate 3. What is the probability that next time I’ll go to the bus stop I’ll see
at least two taxis arriving before a bus? Exactly two taxis?
Solution:
The probability of observing a taxi before a bus is given by
3/(3 + 2) = 3/5
since the waiting times
are independent and exponentially distributed. By the memoryless property both processes then restart
2
and hence the probability of observing (at least) two taxis before the first bus is (3/5) = 9/25. The
2
probability of observing exactly two taxis before the first bus is (3/5) ∗ (2/5) = 18/125.
Задача 8.25.
Пусть
Tk
k -го
- время наступления
события в Пуассоновском потоке с интенсивностью
количество событий, наступивших к моменту времени
а) Как связаны
P (Tk ≤ t)
P (Nt ≥ k)?
Tk будет
плотности Tk
а
Nt
-
и
б) Найдите вероятность того, что
в) Найдите функцию
λ,
t
меньше
t
Задача 8.26.
Let
T1
and
T5
be the time of the first and fifth arrivals in a Poisson process with rate
conditional density of
T1
λ.
a) Find the
given that there are 10 arrivals in the time interval (0,1).
b) Find the conditional density of
T5
given that there are 10 arrivals in the time interval (0,1).
c) Recognize the answers to a) and b)!
Hint A: Используйте то, что
Hint B: Используйте то, что
p
(t)
pTk (t|N1 = 10) = P (N1 = 10|Tk = t) P (NTk1 =10)
условное распределение Tk - это распределение
порядковой статистики
Задача 8.27. Большой секрет
Большой секрет знают
n
человек. Беседы между
процессом с интенсивностью
С вероятностью
p
λ.
i-ым
и
j -ым
человеком являются Пуассоновским
В каждой беседе эти люди, естестенно, говорят о Большом секрете.
каждая беседа может стать достоянием общественности.
Сколько в среднем пройдет времени прежде чем Большой секрет перестанет им быть?
n(n − 1)λ
pn(n − 1)λ
Solution: Общее число бесед - Пуассоновский процесс с интенсивностью
Число разглашенных бесед - Пуассоновский процесс с интенсивностью
1
Ожидаемое время до разглашения:
n(n−1)pλ
Source: Steele, Models for management secrets, Management science, vol. 35, no 2, 1989
Задача 8.28. Волшебный Сундук
Если присесть на Волшебный Сундук, то сумма денег, лежащих в нем, увеличится в два раза.
Изначально в Сундуке был один рубль. Предположим, что «посадки» на Сундук - Пуассоновский
процесс с интенсивностью λ. Каково ожидаемое количество денег в Сундуке к моменту времени
N
λt
Answer: найти E(2 ), вроде бы e
t?
Задача 8.29.
Время работы телевизора «Best» до первой поломки является случайной величиной, распределенной
по показательному закону. Среднее время безотказной работы телевизора фирмы «Best» составляет
63
10 лет.
а) Какова вероятность того, что телевизор проработает более 15 лет?
б) Какова вероятность, что телевизор, проработавший 10 лет, проработает еще не менее 15 лет?
Задача 8.30.
К продавцу воздушных шариков подходят покупатели: мужчины, женщины и дети. Предположим,
что это независимые Пуассоновские потоки с интесивностями 12, 10 и 6 чел/час.
а) Какова вероятность того, что за час будет всего 30 покупателей?
б) Какова вероятность того, что подошло одинаковое количество мужчин, женщин и детей, если за
некий промежуток времени подошло ровно 30 покупателей?
подсказка: сумма Пуассоновских потоков - Пуассоновский поток
9. Компьютерные эксперименты
Задача 9.1.
Нарисуйте 3 реализации броуновского движения, используя 5, 10, 100 и 1000 слагаемых.
Задача 9.2.
В одной партии игрок выигрывает
2n
рублей с вероятностью
2−n , n ∈ N.
Постройте 3 реализации
зависимости среднего выигрыша от числа партий.
Задача 9.3.
Постройте график для riffle-shuffle
Задача 9.4.
P (Xi = 1) = p. Постройте эмпирический закон
распределения длины самой длинной серии из единиц для n = 20 и p = 0.9.
Т.е. по горизонтали должно быть отложено i = 0..20, а по вертикали, частота с которой самая
длинная серия из единиц была длины i.
Пусть
X1 , X2 ,..., Xn
- iid последовательность из 0 и 1.
Задача 9.5. Мыльная пленка
Пьяница падает в канаву, где его штрафует милиционер.
Сосинский
Задача 9.6.
Сгенерите 1000 равномерно распределенных на отрезке [0; 1] с.в. Xi . Рассчитайте значения слу2
чайных величин X̄ и σ̂ . Сравните их с E(Xi ) и V ar(Xi ). Постройте гистограмму (эмпирическую
функцию плотности) для полученных 1000 чисел.
Задача 9.7.
Две независимые равномерно распределенные с.в. делят отрезок
[0; 1]
на три части. Проведите по-
добный эксперимент по разделению отрезка 1000 раз. Постройте эмпирические функции плотности
длин левой, средней и правой частей. Каков (эмпирический) вывод?
Задача 9.8.
Сгенерите 10 равномерно распределенных на отрезке
[0; 1] с.в. Рассчитайте сумму X = X1 + ... + X10 .
Повторите эксперимент 1000 раз. Постройте эмпирическую функцию плотности полученных 1000
значений сумм.
Задача 9.9.
Проведя 1000 экспериментов на компьютере найдите 5%-ое пороговое значение статистик U1 (Mann+
Whitney, n1 = 5 и n2 = 4) и T
(Wilcoxon Signed Rank Test, n = 9) для двусторонней альтернативной
гипотезы. Сравните его с асимптотическим.
mula
fera
64
10. Probabilistic method
Задача 10.1. [Williams, 4.2]
Для
s>1
обозначим
ξ(s) =
P∞
1
n=1 ns .
а) Докажите, что события
−s
P (X = n) = nξ(s) .
{X делится на p} и {X делится
Рассмотрим случайную величину
X
с
на
q}
независимы, если
p
и
q
- два
различных простых числа
1
= p is prime (1 − p−s )
ξ(s)
не делится на квадрат ни одного простого числа}
Q
б) Докажите формулу Эйлера:
P ({X
P (X = xk|X mod k = 0)
д) Пусть с.в. Y имеет такое же распределение как и X , и они независимы. Пусть с.в. H
общий делитель X и Y . Найдите P (H = n).
в) Найдите
г) Найдите
- наибольший
Задача 10.2.
В круге радиуса 16 выбрано 650 точек. Докажите, что существует кольцо с внутренним радиусом 2
и внешним радиусом 3, которое содержит не менее 10 точек.
Задача 10.3. [Grimmett, Stirzaker]
На сфере 10% точек окрашены белым, а остальные 90% - красным цветом. Форма окраски неизвестна. Верно ли, что можно так вписать куб, что все его вершины будут одноцветными?
Задача 10.4.
Пусть
A
- произвольная матрица. Докажите, что существует вектор
vAv t ≥ tr(A).
v
состоящий из единиц или
минус единиц, такой что
Solution:
n
i,j aij xi xj over the points of {−1, 1} . That is to
say, let xi be a random variable taking -1,1 with the same probability. Now we want to find the expected
Just take the mean value of the quadratic function
f=
P
value of f. This expected value is the sum of expected values of
seen that this expected value for
i 6= j
is 0 and is
aii
aij xi xj
for different i,j’s. Now it is easily
otherwise. So the expected value of f is just
P
i
aii
which is the trace of the matrix which is equal to the sum of eigenvalues.
Source: http://www.artofproblemsolving.com/Forum/viewtopic.php?t=153979
Задача 10.5.
Let
S
be a finite set of points in the plane such that no three of them are on a line. For each convex
polygon
P
whose vertices are in
S , let a(P ) be the number of vertices of P ,
P . Prove that for every real number x:
and let
b(P )
of points of S which are outside
P a(P )
(1 − x)b(P ) = 1, where the sum is taken over all convex polygons with vertices in
P x
be the number
S.
Remark. A line segment, a point, and the empty set are considered as convex polygons of 2, 1, and 0
vertices respectively.
Solution:
Let’s colour the points black and white and Let the probability that a point has colour black be x. Now
a(P )
look at any convex polygon P . The probability that all its vertices are black is x
and that all the points
b(P )
outside it are white is (1 − x)
. The given summation is the probability that there exists a polygon with
vertices black and all points outside it are white. But we know this is true (i.e. 1) as all we have to do is
consider the convex hull of the black points. Now this is true for all
x ∈ (0, 1).
But this is a polynomial
with finite degree and the number of points at which it evaluates 1 is infinite. so, this is an identity and
holds
∀x ∈ R
Продумать
Задача 10.6. Erdos-Ko-Rado theorem
[Alon, Spencer, Probabilistic method]
[нуждается в обработке.........]
Если: Ω = {0, 1, ..., n − 1}. И H - набор
пересекающихся подмножеств
Ω,
состоящих из
k ≤ n/2
anie
65
элементов, т.е., если
То: число элементов
A ∈ H и B ∈ H,
k−1
#H ≤ Cn−1
то
A ∩ B 6= ∅.
Доказательство:
а) Пусть
As
- множество из
k
чисел подряд, начиная с числа
ется с нуля). Сколько множеств
б) Пусть
σ
As
может входить в
- случайная перестановка, и
i
s
(при необходимости, счет продолжа-
H?
случайный номер. Найдите
P (A ∈ H)
в) Завершите док-во
Задача 10.7. Turan’s theorem [Grimmett, Stirzaker]
Пусть
G
вершины
- конечный граф без петель и двойных ребер. Обозначим
k
dk
- число ребер, исходящих из
(степень вершины). На графе можно отметить несколько вершин так, чтобы ни одна из
них не была напрямую связана с другой. Докажите, что максимальное количество таких вершин
P
n ≥ k dk1+1
Задача 10.8. [by idea of Sasha Serova]
В копилке 40 золотых и 60 серебряных монет.
а) Какова вероятность того, что при выборе 20 монет ровно
б) Чему равна сумма этих вероятностей (i
i
окажутся серебряными?
= 0..20)?
в) Предложите вероятностное доказательство формулы
P
i
n
Cai Cbn−i = Ca+b
Задача 10.9.
Со стола упало 10 чашек. Каждая разбивается с вероятностью 0.5.
а) Какова вероятность, что разобьется ровно
k
чашек?
б) Предложите вероятностное доказательство формулы
P
i
Cni = 2n
Задача 10.10. In a square of side 1, you have been able to capture 51 ants, who are possibly panicking.
You have a glass, whose radius is 1/7.
Show that you can at any moment position the glass so as to encompass at least 4 of them.
Solution:
Накроем стаканом случайную (равномерно выбираемую) область, целиком лежащую внутри квадрата.
1 2
. Здесь δ - это
7
площади четырех ненакрываемых кусков в углах квадрата. Получаем, что ожидаемое число больше
4−π
3-х (δ =
). Значит, есть точка, где накроется 4 муравья.
49
Ожидаемое число муравьев, попадающих в эту область равно
(1 − δ) · 51 · π
11. Optimisation, Game theory
Задача 11.1.
Есть две пустые урны, 50 белых и 50 черных шаров. Сначала наугад выбирается одна из урн, затем
из нее выбирается один шар наугад. Как следует разложить шары по урнам (в каждой урне должен
лежать хотя бы один шар), чтобы вероятность вытащить белый шар была максимальной?
Задача 11.2. [Mosteller]
Чтобы подбодрить сына, делающего успехи в игре в теннис, отец обещает ему приз, если он выиграет
подряд по крайней мере две теннисные партии против своего отца и клубного чемпиона по одной из
схем: отец - чемпион - отец или чемпион - отец - чемпион по выбору сына. Чемпион играет лучше
отца. Какую схему следует выбрать сыну?
Задача 11.3. [Mosteller] Выборка с возврашением или без возвращения?
Две урны содержат красные и черные шары, не различимые на ощупь. Урна А содержит 2 красных
и 1 черный шар, урна В-101 красный и 100 черных шаров. Наудачу выбирается одна из урн, и вы
получаете награду, если правильно называете урну после вытаскивания двух шаров из нее. После
вытаскивания первого шара и определения его цвета вы решаете, вернуть ли в урну этот шар перед
66
вторым вытаскиванием. Какой должна быть ваша стратегия?
Задача 11.4.
There are n different pairs of sox in a drawer. One sock is taken out at a time until a matching pair has
been found.
a) If n is known what is the most likely value of r, the number of sox drawn out?
b) If r is known what value of n gives the greatest chance of this happening?
Ugly answers?
Задача 11.5. Четыре шкатулки
Имеется 4 внешне одинаковых шкатулки. Внутри каждой шкатулки написан ее номер. Внутри
шкатулки номер 4 также лежит 1 млн. рублей. Вы играете в следующую игру:
Вы выбираете шкатулки одну за одной. После выбора шкатулки, Вы можете либо забрать ее, либо
продолжить игру. Игра заканчивается в тот момент, когда Вы решаете забрать шкатулку или когда
оказывается, что номер внутри выбранной только что шкатулки меньше, чем номер внутри предыдущей шкатулки. Номера внутри шкатулки определяет ведущий, Вы же не знаете содержимого
шкатулок. Вашем выигрышем является содержимое последней выбранной шкатулки.
Какова оптимальная стратегия?
Каков ожидаемый выигрыш при оптимальной стратегии? Source: www.wilmott.com-forum-brainteasers
Задача 11.6.
Пусть
A
- корреляционная матрица. Чему равен наибольший возможный определитель?
Solution:
Trace(M)=sum(eigenvalues), Det(M)=product(eigenvalues)
For correlation matrix, since trace=N, where N dimension average(eigenvalues)=1, Then, log(Det)=sum(log(eieg
since log has negative convexity
Hence Det<=1
Source: wilmott
Задача 11.7.
[rather
hard] I am trying to find someone whom I know is in a 5-story building, say a bookstore, and
based on what I know about her, the probabilities of finding her on each floor are:
5th: 15%
4th: 40%
3rd: 10%
2nd:30%
1st: 5%
I start from the 1st floor, and it takes me 1 minute to take an escalator up or down to an adjacent floor,
and 5 minutes to completely search a floor, after which I will either have found her and we can leave, or I
will know she is not on that floor. Of course, I can stop and search the floors in any order, so the time I
spend searching does not have to follow in adjacent floors, but I can only travel between adjacent floors.
Assuming she stays where she is, in which order should I search the floors to minimize my expected time
until finding her.
Solution (maybe incorrect):
2, 4, 5, 3, 1 has an expected time of 13.8 minutes, this is the minimum.
My algorithm for solving requires some guesswork. Take everything one decision at a time:
(1) Search the first floor or go up? If I search the first floor, 95% of the time I add five minutes to my time.
5% of the time save the amount of time it takes to search floors 2 to 5. I know I can search these floors in
24 minutes, 5% of 24 is less than 95% of 5, so I skip the first floor.
(2) Search the second floor or go up? If I skip this floor, I clearly have to go all the way to four, since
it wouldn’t make sense to skip 2 but search 3. If I do that and don’t find her, I have to come back to 2.
67
Doing 2 first saves four minutes of travel time and 10% of the time costs five minutes. Clearly, I search
the second floor.
(3) Search the third floor or go right up to four? This is a little more complicated. If I skip the third floor,
I clearly have to go four, five then three. If I search the third floor, then I do four and five. Doing four and
five takes 14 minutes if she is on three. Searching the third floor costs me five minutes 6/7 of the time and
saves me 14 minutes 1/7 of the time; clearly I skip.
(4) Now of course I search the fourth floor. After that I clearly prefer the fifth floor to the third, because it
has a higher probability and the cost of skipping it is higher. On my way back down, of course I search the
third floor, because it has a higher probability than one and a shorter travel time. After that, only one is left.
Задача 11.8. Grimmett, 2.7.16. Transitive coins
Есть три неправильные монетки (A,
B , C ),
выпадающие с вероятностью
3/5
на «орла». Игроки по
очереди выбирают себе монетку. Затем подкидывают. У кого больше очков, тот и выиграл. Очки
считаются так:
монетка
монетка
монетка
A
B
C
- 10 очков за «решку» и 2 очка за «орла»
- 4 очка за «решку» и 4 очка за «орла»
- 3 очков за «решку» и 20 очков за «орла»
У какого игрока больше шансов выиграть в этой игре, если каждый игрок максимизирует вероятность выигрыша?
Задача 11.9.
If X, Y, and Z are 3 random variables such that X and Y are 90% correlated, and Y and Z are 80%
correlated, what is the minimum correlation that X and Z can have?
Задача 11.10.
Имеется дерево (граф) с 2006 ребрами. А и В - это две вершины этого дерева. Мы движемся случайным образом от А к В не поворачивая назад. На каждой развилке мы выбираем равновероятно
каждое из возможных ребер.
а) При какой форме графа и при каком выборе точек А и В вероятность попасть из А в В будет
минимальной?
б) Чему будет равна эта вероятность?
Задача 11.11.
Есть три рулетки: на первой равновероятно выпадают числа 2, 4 и 9; на второй - 1, 6 и 8; на третьей
- 3, 5 и 7. Сначала первый игрок выбирает рулетку себе, затем второй игрок выбирает рулетку себе.
После этого рулетки запускаются, и выигрывает тот, чья рулетка покажет большее число.
Каким игроком лучше быть в этой игре? Почему?
Задача 11.12.
Вы хотите приобрести некую фирму. Стоимость фирмы для ее нынешних владельцев - случайная
величина, равномерно распределенная на отрезке [0;1]. Вы предлагаете владельцам продать ее за
называемую Вами сумму. Владельцы либо соглашаются, либо нет. Если владельцы согласны, то Вы
платите обещанную сумму и получаете фирму. Когда фирма переходит в Ваши руки, ее стоимость
сразу возрастает на 20%.
а) Чему равен Ваш ожидаемый выигрыш, если Вы предлагаете цену 0,5?
б) Какова оптимальная предлагаемая цена?
Задача 11.13. Дама пик
Устав от вереницы женихов, разборчивая невеста уединилась. Перед ней колода из 36 карт, хорошо
перемешанная. На этот раз принцесса должна предсказать появление дамы пик. То есть принцесса
открывает одну за одной карты из колоды и в любой момент может остановиться и сделать заявление «Следующая карта будет королем». Если следующей после заявления картой действительно
будет король, то принцесса выиграла. Оптимальная стратегия?
alda
osti
vila
68
Solution:
игра не меняется, если делать ставку на последнюю карту.
все стратегии оптимальны и дают выигрыш в
4/36
Задача 11.14. Пьяный водитель
На шоссе подряд идут два поворота. Внешне повороты не отличимы друг от друга. Будучи слегка
«навеселе» Вася возвращается домой. Васе нужен второй по счету поворот, но из-за своего состояния он не может определить, какой по счету поворот он проезжает. Поэтому он поворачивает с
вероятностью
p,
а с вероятностью
q = 1−p
едет прямо на каждом повороте. Если Вася повернет на
первом повороте, то заедет в соседнюю деревню и получит полезность 2. Если он свернет на втором
повороте, то получит полезность 5. Если Вася проедет оба поворота, то получит полезность 1.
Найдите оптимальное
p.
Задача 11.15. Ящик с носками
В ящике лежат красные и черные носки. Если из ящика наудачу вытягиваются два носка, то
вероятность того, что оба они красные, равна 1/2.
а) Каково минимальное возможное число носков в ящике?
б) Каково минимально возможное число носков в ящике, если число черных носков четно?
Задача 11.16. Mechanism design
Поп нанял Балду, чтобы тот предсказывал ему погоду. Дождь идет с вероятностью
p
народным приметам Балда точно знает
p.
Благодаря
накануне вечером. Балда выдает Попу свою оценку
p̂
вероятности дождя завтра.
а) Как будет вести себя Балда, если Поп выплачивает ему награждение по принципу: если дождь
действительно был, то выплачивается
p̂,
если дождя не было, то
1 − p̂?
б) Поп решил заставить Балду выдавать точные оценки. В случае дождя Поп платит
в случае сухой погоды. Какую
f
f (p̂), и f (1 − p̂)
следует выбрать Попу?
в)???
Подсказка: при решении задачи Поп столкнется со странным дифференциальным уравнением, у
которого много решений, но 9-ти классов церковно-приходской школы ему вполне должно хватить...
Коммент: в условии задачи опечатка - в ЦПХ было 4 класса. Хотя кто знает, когда там проходили
диф. уры?
Задача 11.17. Поиск функции плотности
Два спортсмена готовятся к соревнованиям. Каждый из них выбирает свой уровень усилий
ei ∈ [0; 1].
Побеждает тот, кто приложил больше усилий при подготовке. Если оба приложили одинаковое количество усилий, то не побеждает никто. Победитель получает платеж равный 1. Издержки по
2
приложению усилий равны Ci = 2ei для каждого игрока.
а) Существует ли равновесие по Нэшу в чистых стратегиях?
б) Найдите равновесие по Нэшу, в котором уровень усилий каждого игрока имеет функцию плотности
p(t),
отличную от нуля на
[a; b].
Задача 11.18.
Энтропией с.в.
X
называется число
E
1
log2 p(X)
, где
p(t) = P (X = t)
для дискретных с.в. или
p(t)
- функция плотности для непрерывных с.в.
Энтропия измеряет количество информации (в битах), получаемое при наблюдении с.в.
имеется монетка, выпадающая гербом с вероятностью
θ.
С.в.
X
X.
Пусть
равна единице, если монетка выпа-
дает гербом, и нулю в противном случае.
При каком
θ
энтропия будет максимальной?
Задача 11.19.
Маша и Саша играют в быстрые шахматы. У них одинаковый класс игры и оба предпочитают
играть белыми, т.е. вероятность выигрыша того, кто играет белыми равна
p > 0, 5. Партии играются
rnih
69
до 10 побед. Первую партию Маша играет белыми. Она считает, что в каждой последующей партии
белыми должен играть тот, кто выиграл предыдущую партию. Саша считает, что ходить белыми
нужно по очереди. При каком варианте правил у Маши больше шансы выиграть?
Задача 11.20. О пользе гадания на кофейной гуще замолвите слово. . . [Winkler]
Маша пишет на бумажках два любых различных натуральных числа по своему выбору. Одну
бумажку она прячет в левую руку, а другую - в правую. Саша выбирает любую Машину руку.
Маша показывает число, написанное на выбранной бумажке. Саша высказывает свою догадку о
том, открыл ли он большее из двух чисел или меньшее. Если Саша не угадал, то Маша выиграла.
Перед выбором руки и высказыванием догадки Саша может обратиться к потомственной гадалке в
пятом поколении Глафире Лукитичне (500% гарантия, снятие порчи и сглаза без греха и ущерба для
здоровья, исправляет некачественную работу шарлатанов). Глафира Лукитична называет наугад
1 1 1 (ничего не зная о Маше!) одно из чисел
1 2 ; 2 2 ; 3 2 ; ... с вероятностями соответственно равными
1 1 1 ; ; ; ... . Другими словами, действия Саши (выбор левой или правой руки и высказываемая вер2 4 8
сия) могут зависеть от слов гадалки. Какую стратегию Саше следует выбрать, чтобы гарантировать
себе (вне зависимости от действий Маши!) вероятность выигрыша строго более 50%?
Задача 11.21. Наш суд - самый гуманный суд в мире!
Как известно, истина распределена равномерно на отрезке [0;1]. Истец называет число [0;1] - желаемую компенсацию за моральный ущерб. Одновременно с истцом ответчик называет свою оценку
ущерба (на том же отрезке). Суд обязывает ответчика выплатить истцу моральный ущерб в том
объеме, который оказался ближе всего к истине.
а) Как следует играть истцу и ответчику?
б) А если истина распределена нормально
N (0; 1)?
Задача 11.22. Маша и Саша came back!
Маша наблюдает реализацию двух независимых случайных величин
номерно на
[0; 1].
X
и
Y,
распределенных рав-
Она выбирает, значение какой из них рассказать Саше. Саша выигрывает, если
угадает, какая из величин наибольшая (та, значение которой он узнал от Маши, или другая). Маша
выигрывает, если Саша ошибется. Найдите оптимальные стратегии.
Задача 11.23.
В мешке лежат бочонки. На каждом из них написана цифра. На одном бочонке написана цифра
1, на двух бочонках - цифра 2,..., на девяти бочонках - цифра 9. Маша вытаскивает один бочонок
наугад. Саша не знает, какой бочонок достала Маша. Саша может задавать Маше вопросы, на
которые можно отвечать только «да» или «нет».
а) Как выглядит стратегия, минимизирующая ожидаемое число вопросов, необходимых чтобы
угадать цифру?
б) Как выглядит стратегия, минимизирующая число вопросов, достаточных, чтобы угадать цифру
в самом неблагоприятном случае?
Задача 11.24.
Перед Машей колода в 52 карты. Маша открывает карты одну за одной. Изначально у Маши 1
доллар. Маша может делать любую ставку в пределах имеющейся у нее суммы на цвет следующей
карты. Найдите оптимальную стратегию и ожидаемый выигрыш, который приносит эта стратегия.
Предположим бесконечную делимость денег.
Задача 11.25. [Mosteller] Выигрыш в небезобидной игре
Игра состоит из последовательности партий, в каждой из которых вы или ваш партнер выигрывает
очко, вы - с вероятностью
p
(меньшей, чем 1/2), он - с вероятностью
1 − p.
Число игр должно быть
четным. Для выигрыша надо набрать больше половины очков. Предположим, что вам известно, что
p = 0, 45,
и в случае выигрыша вы получаете приз. Если число партий в игре выбирается заранее,
70
то каков будет ваш выбор?
Задача 11.26. Разборчиая принцесса-1
К принцессе случайным образом выстроилась очередь из
n
женихов. Цель принцессы - выбрать
самого богатого (даже второй по богатству жених королевства не устраивает принцессу). Женихи
заходят по очереди. Когда заходит очередной претендент, принцесса узнает его годовой доход и
должна либо выбрать его, либо перейти к следующему. Вернуться к предыдущему нельзя. Предполагается, что все варианты расположения женихов по доходу равновероятны.
а) Найдите
gn
- вероятность того, что принцесса выбрала самого богатого жениха, если известно,
что она остановилась на
n
-ом претенденте, доход которого был больше, чем у предыдущих претен-
дентов.
б) Обозначим
hn
- вероятность выигрыша принцессы в случае, если она пропускает
женихов, а далее играет наилучшую стратегию. Докажите, что
в) Как выглядит
hn
gn = hn ?
уравнение на hn правее
hn
n
первых
не возрастает.
левее точки
г) Составьте разностное
limn→∞ h0 .
Причем здесь e?
точки
gn = hn .
д) Найдите
Тигр:
Причем здесь Березовский?
Задача 11.27. Разборчиая принцесса-2
Имеется 100 женихов. Доход
i-го
жениха равен
Xi .
Величины
Xi
- iid, равномерны на отрезке
[0; 1].
Женихи заходят по очереди. Когда заходит очередной претендент, принцесса узнает его годовой
доход и должна либо выбрать его, либо перейти к следующему. Вернуться к предыдущему нельзя.
Принцесса максимизирует ожидаемую полезность.
Как выглядит оптимальная стратегия?
Задача 11.28.
Подбрасываются два различных неправильных кубика. Возможно ли, что вероятность того, что
сумма равна
i, pi ∈ (2/33; 4/33)?
Возможно ли получение всех чисел от 2 до 12 сделать равновероятным?
Solution:
Заметим, что
a1 b6 + a6 b1 < 2a1 b1
и
a1 b6 + a6 b1 < 2a6 b6 .
Перемножаем два неравенства, получаем противоречие.
Source: IMS2007
Задача 11.29.
Усама бен Ладен хочет перенести 1000 тротиловых шашек из одной пещеры в другую. При транспортировке каждая шашка взрывается с вероятностью
p.
Если взрывается одна шашка, то взрываются
и все остальные. Сам Усама при этом чудом остается жив. Какими партиями нужно переносить
шашки, чтобы минимизировать среднее число переносок?
1
Ответ: −
≈ p1
ln(1−p)
Вариация?:
source: чудо-сканер у Жени Надоршина, по мотивам истории со сканированием Mikosch
Задача 11.30.
Известно, что случайная величина
X
принимает значения из отрезка
[0; 1].
а) Найдите наибольшую возможную дисперсию.
б) Приведите пример случайной величины, имеющей такую дисперсию.
Source: aops, 122082
Задача 11.31.
Петя с Васей играют в крестики-нолики на поле
3 × 3.
Вася начинает игру крестиками. Петя играет
в первый раз и поэтому ставит нолики случайным образом в свободные клетки, и Васе известно об
этом. Какова должна быть стратегия Васи, чтобы вероятность его выигрыша была наибольшей?
71
Чему будет равна эта вероятность?
Ответ: центр-угол-далее по смыслу. (?).
p = 95/96.
Задача 11.32.
Человек хочет купить чудо-пылесос, и конечно, подешевле. Посещение каждого магазина связа-
c > 0. Цена чудо-пылесоса в
[0; M ]. Предположим также, что
но с издержками, равными
каждом магазине имеет равномерное
распределение на отрезке
если человек решит вернуться в уже
посещенный магазин, то цена там уже могла измениться, и поэтому уже посещенный магазин можно
рассматривать как новый. Магазинов, где продается чудо-пылесос, бесконечно много.
a) Как выглядит оптимальная стратегия?
б) Сколько в среднем магазинов будет посещено при использовании оптимальной стратегии?
в) Чему равны ожидаемые издержки покупателя (цена+издержки посещения магазинов)?
г) Чему равна ожидаемая цена покупки?
д) Верно ли, что зависимости от
c
и
M
имеют ожидаемый знак?
е) Что изменится, если предположить, что цена в посещенных магазинах не меняется и покупатель
имеет возможность вернуться в уже посещенный магазин?
Answers:
а) Если текущее
p > p̄,
то ищем дальше, если
p < p̄,
то берем
Ожидаемые общие издержки удовлетворяют уравнению:
T C = c + P (p < p̄)E(p|p < p̄) + (1 − P (p < p̄))T C
p̄
Отсюда T C =
+ cM
2
√p̄
Оптимальное p̂ =
2cM
б)
1
P (p<p̄)
в) В точке оптимума
TC =
√
2cM
е) Ничего. Возвращаться в уже посещенный магазин не имеет смысла при использовании оптимальной стратегии, т.к. желание вернуться означает то, что из него не надо было уходить
Задача 11.33. Автомат с кофе и горячим шоколадом
Горячий шоколад в автомате стоит 15 рублей. Автомат принимает монеты достоинством 1 рубль,
2 рубля и 5 рублей. У меня на руках имеется одна пятирублевая, три двухрублевых и четыре
рублевых монеты. Каждая монета может застрять в автомате с вероятностью
p. Застрявшая монета
не защитывается и обратно не выдается. Я хочу купить горячий шоколад или по крайней мере
минимизировать средние потери вызванные застреванием монеты.
а) В каком порядке нужно бросать монеты?
б) Какими должны быть вероятности застревания монет, чтобы мне было безразлично в каком
порядке их кидать?
Решение:
а) 1, 2, 5
б) вероятности должны быть пропорциональны достоинству монет:
Рассмотрим монеты достоинством
1
и
n.
Пусть они застревают с вероятностями
Ожидаемые потери от последовательности
Ожидаемые потери от последовательности
Они равны при
p
и
x.
1, n: p + (1 − p)x(1 + n)
n, 1: x + (1 − x)p(1 + n)
x = pn
Если монет больше двух, а вероятности пропорциональны, то две соседние можно переставить
местами, что не скажется на ожидаемых потерях
Задача 11.34.
Кинотеатр обещает вручить приз первому человеку в очереди, чей день рожденья совпадает с
днем рождения кого-нибудь из впереди стоящих. Допустим, что Вы можете выбрать любое место в
очереди. Какое Вы бы выбрали?
Answer: оптимальное место находится руками, 20, хотя саму вероятность выигрыша без компьютера
72
посчитать очень трудоемко.
Задача 11.35. Mahler’s theorem and
π
Теорема Малера (1953) утверждает, что для любых натуральных p > 1 и q > 1 выполняется
p
−42
неравенство: |π − | > q
. У Вас 1 рубль. Теперь представим, что вы делаете ставки на очень
q
отдаленные цифры числа π . Т.е. казино выбирает, с какого знака числа π начинается игра. Вы
знаете выбор казино, но это число настолько велико, что Ваших вычислительных возможностей
не хватает на вычисление этих знаков. Вы ставите любое количество денег из имеющихся у Вас
на любую цифру. Можно поставить на несколько цифр сразу, можно делать сколь угодно мелкую
ставку. Ставка сделанная на верную цифру возвращается в 10 кратном размере. Ставка сделанная
на неверную цифру уходит к казино.
Можете ли Вы гарантировать себе выигрыш в долгосрочном периоде?
Решение:
Например, из теоремы Малера следует, что начиная с n-го знака в π не может идти 41n нулей
1041n − 1 часть и ставим их на все последовательности,
подряд. Следовательно делим доллар на
кроме последовательности из нулей.
Задача 11.36.
Известно, что
P (X ∈ (−1; 3)) = 0
и
E(X) = 0.
Чему равна минимально возможная дисперсия
X?
Задача 11.37.
В киосках продается «открытка-подарок». На открытке есть прямоугольник размером 2 на 7. В
каждом столбце в случайном порядке находятся очередная буква слова «подарок» и звездочка.
Например, вот так:
П
*
*
А
*
О
К
*
О
Д
*
Р
*
*
Прямоугольник закрыт защитным слоем, и покупатель не видит, где буква, а где - звездочка.
Следует стереть защитный слой в одном квадратике в каждом столбце. Можно попытаться угадать
n > 0 букв слова ’подарок’
50 · 2n−1 рублей. Если открыта
любое число букв. Если открыто
и не открыто ни одной звездочки, то
открытку можно обменять на
хотя бы одна звездочка, то открытка
остается просто открыткой.
а) Какой стратегии следует придерживаться покупателю, чтобы максимизировать ожидаемый
выигрыш?
б) Чему равен максимальный ожидаемый выигрыш?
Задача 11.38.
В забеге участвуют две лошади, Метель и Пурга. Вася верит, что Метель побеждает с вероятностью
1/2. Петя верит, что Пурга побеждает с вероятностью 1/3. Каждый из них согласен участвовать в
споре, если получают положительную ожидаемую прибыль.
Можете ли вы гарантировать себе безрисковое получение 1 рубля?
Ответ: да, два линейных уравнения
Задача 11.39.
At a horse race, a horse named "Winner"has 25% chance to win and is posted at 4 to 1 odds. (For every
dollar gambler bets, he receives $5 if horse wins and nothing if it loses). If gambler has square root utility
function:
What fraction of his money should he bet on "Winner"?
Задача 11.40. Kelly criterion
У Васи имеется капитал
ность выигрыша
p.
S.
Вася может сделать любую ставку в рамках своего капитала. Вероят-
В случае выигрыша Вася получает свою ставку обратно в удвоенном размере, в
случае проигрыша - теряет ставку.
73
а) Какую часть капитала нужно поставить чтобы максимизировать ожидаемую прибыль, выраженную в процентах?
б) Какую часть капитала нужно поставить, чтобы максимизировать ожидаемый логарифм своего
итогового капитала?
Задача 11.41. Say we have 4 multiple choice questions with 4 choices each. The answers are marked
a,b,c,d. We are told that the right answer to each question has a different mark. I mean, one of the answer
is a, one is b, one is c, and one is d.
The subject is marine biology and we have no idea what the right answers are.
a) Is there a way to get a better expected score than by just random guessing?
b) What strategy minimize variance of score? Maximize?
Задача 11.42. Phone Call
Alice tries to call Bob, who is not at home now. For every second, Bob has a probability p of comming
back home, and he won’t leave once he is back. Alice will lose c dollars per second until she finally reaches
Bob through his home-phone. Each phone call costs Alice D dollars, even if nobody answers it. How
should she arrange her calls? (Assume Bob immediately picks up the phone when his home-phone rings if
he is at home. Neglect the time of bell ringing.)
Source: Chenyang’s Favorate Problems
Задача 11.43. Unfair Coins
There are two unfair coins. One coin has .7 probability head-up; the other has .3 probability head-up. To
begin with, you have no information on which is which. Now, you will toss the coin 10 times. Each time, if
the coin is head-up, you will receive $1; otherwise you will receive $0. You can select one of the two coins
before each toss. What is your best strategy to earn more money? Source: Chenyang’s Favorate Problems
12. Нормальное распределение и ЦПТ
Задача 12.1.
X ∼ N (0; 1); Z
Y = X · Z . Найдите:
Пусть
равновероятно принимает значения
а) закон распределения
б)
1
и
−1; X
Y;
Cov(X, Y );
в) условное распределение
г) верно ли, что
X +Y
X,
если известно, что
Y = y.
нормально?
P (Z = 1) = 1 − P (Z = −1) = p?
X ∼ N , Y ∼ N , Corr(X, Y ) = 0, то X + Y ∼ N ?
д) Что изменится, если
е) Верно ли, что если
Задача 12.2.
Пусть
X ∼ tn .
Как распределена величина
Y = X 2?
Задача 12.3.
Пусть
X ∼ N (0; 1)
Найдите функцию плотности для
Z=
1
X2
Задача 12.4.
Пусть
Xi
- iid
N (0; 1).
Найдите
P (X12 + X22 > 6.37 · (X32 + X42 + X52 )).
Задача 12.5.
Пусть
Xi
- iid
N (0; 1).
а) Как распределена случайная величина
X1
?
|X2 |
и
Z
независимы. Рассмотрим
74
б) Найдите
p
P X5 > 2.3 X12 + X22 + X32 + X42 .
Задача 12.6.
Пусть
Y ∼ χ2n
и
W ∼ tn .
Найдите
Задача 12.7.
Найдите
P (Y > 2),
если
P9
Y =
E(Y ), E(W )
i=1
Xi2 ,
а
Xi
и [т?]
- iid
V ar(Y ).
N (0; 1).
Задача 12.8.
Пусть
Y1
свободы,
χ2 распределение с 5-ю степенями свободы, а Y2 - χ2
причем Y1 и Y2 независимы. Как распределена их сумма?
имеет
распределение с
15-ю степенями
Задача 12.9.
На плоскости выбирается точка со случайными координатами. Абсцисса и ордината независимы и
распределены
N (0; 1). Какова вероятность того, что расстояние от точки до начала координат будет
больше 2,45?
Задача 12.10.
Пусть
X ∼ N (0; 1).
Найдите
P (X > 0, 5), P (−1 < X < 2), P (X 2 > 3), P (X < 0, 3), P (|X| < 0, 8)
Задача 12.11.
X ∼ N (4; 9), Y ∼ N (−5; 16), Z ∼ N (20; 100) ; X , Y и Z - независимы. Найдите P (X > 8),
P (X ∈ [1; 5]), P (Y ∈ [−10; −3)), P (Z > 100), P (X + Y > 3), P (|Z| > 10), P (4Y + Z > 15).
Пусть
Задача 12.12.
Монетку подбрасывают 1000 раз. Пусть
S
- общее количество «орлов». Найдите
P (S > 550),
P (S < 480), P (S < 400).
Задача 12.13.
2
Доходность акций компании А представляет собой с.в. X ∼ N (50; 5 ), а доходность акций компании
2
B — с.в. Y ∼ N (80; 9 ). Определите вероятность того, что средний доход по пакету из восьми акций
А и двух акций В составит не менее 75. Известно, что
Cor(X, Y ) = −0, 4.
Задача 12.14.
Страховая компания заключила 16000 договоров. В среднем страховой случай наступает у одного
S - количество
P (1550 < S < 1650), P (S < 2000).
человека из 10. Пусть
наступивших страховых случаев. Найдите
P (S > 1800),
Задача 12.15.
Дневные расходы электроэнергии на предприятии - случайная величина, с матожиданием 1400 КВт
и стандартным отклонением 50 КВт. Какова вероятность того, что за 70 дней средние дневные
расходы будут меньше 1340 КВт? больше 1500 КВт? от 1300 КВт до 1500 КВт?
Задача 12.16.
Пусть
X ∼ N ((
P (2X1 + X2 < 5).
2
9 −1
); (
)).
3
−1 16
Найдите
E(X1 ), E(X1 + 2X2 ), V ar(X1 − X2 ), P (X1 > X2 ),
Как распределена случайная величина
X1
при условии, что
X2 = 6? X2 = −3?
Задача 12.17.
Пусть
X ∼ N (7; 16).
Найдите
E(X|X > 11), E(X|X < 10), E(X|X ∈ [0; 10]).
Задача 12.18.
В большом-большом городе
N
80% аудиокиосков торгуют контрафактной продукцией. Какова
tion
75
вероятность того, что в наугад выбранных 90 киосках более 60 будут торговать контрафактной
продукцией? менее 50? от 40 до 80? от 70 до 75?
Задача 12.19. [Айвазян, экзамен РЭШ]
В поселке 2500 жителей. Каждый из них в среднем 6 раз в месяц ездит в город, выбирая день
поездки независимо от других людей. Поезд ходит в город один раз в сутки.
а) Какой наименьшей вместимостью должен обладать поезд, чтобы он переполнялся в среднем не
чаще 1 раза в 100 дней?
б) Сколько в среднем человек будет ехать в таком поезде, если предположить, что при переполнении
часть людей полностью откажется от поездки?
Solution:
Xi - индикатор того, едет
E(Xi ) = 51 , V ar(Xi ) = 52
P (S > v) = 0.01
v−500
= 2.33
1000
v = 2830 ???
ли сегодня
i-ый
человек
Задача 12.20.
Маша никогда не готовится к зачетам и экзаменам и рассчитывает только на везение. Предположим,
что Машина оценка - случайная величина, с матожиданием 5 баллов и стандартным отклонением 3
балла. Какова вероятность того, что при сдаче 100 экзаменов Машин средний балл будет меньше 4?
больше 6? от 4,8 до 5,1? от 4,9 до 5?
Задача 12.21.
Количество опечаток в газете - с.в. с матожиданием 10 и дисперсией 25. Какова вероятность того,
что по 144 газетам среднее количество опечаток не превысит 11? будет от 10 до 10,5? Будет больше
9,5? меньше 20?
Задача 12.22. [полезно запомнить]
2
Пусть X ∼ N (µ; σ ). Найдите P (|X
− µ| > 2σ), P (|X − µ| > 3σ).
Задача 12.23.
( X1 X2 )
Случайный вектор
( 2 −1 )
имеет нормальное распределение с математическим ожиданием
и ковариационной матрицей
(
9
−4, 5
).
−4, 5 25
Найдите
P (X1 + 3X2 > 20).
Задача 12.24.
Пусть величины
X
и
Y
имеют совместное нормальное распределение с нулевым средним, единичной
дисперсией и корреляцией
a) Представьте
X
и
Y
ρ.
в виде
X = ax1 Z1 + ax2 Z2 , X = ay1 Z1 + ay2 Z2 ,
так, чтобы
Z1 , Z2
имели нулевое
среднее, единичную дисперсию и не были бы коррелированы.
X >0∩Y >0
P (XY > 0)?
P (XY > 0) как функции от ρ
б) Что представляет собой множество
в осях
(Z1 , Z2 )?
в) Чему равна вероятность
г) Постройте график
Solution
a, чтобы матрица A
θ = π − arccos(ρ)
a) Можно так выбрать числа
б) Угол с градусной мерой
в)
была симметричной.
P (XY > 0) = θ/π
Задача 12.25.
Пусть величины
нием
0
X, Y , Z
- имеют совместное нормальное распределение, с математическим ожида-
B.
P (XY Z > 0)?
и некоей ковариационной матрицей
Как зависит от
B
вероятность
76
Solution:
−X , −Y , −Z , то у них такое же математическое ожидание
матрица. Значит P (XY Z > 0) = P ((−X)(−Y )(−Z) > 0). Но эти
Никак. Если рассмотреть величины
и такая же ковариационная
вероятности в сумме дают 1, значит они равны по 0.5.
Задача 12.26.
Пусть
X
динатами
Y независимы и имеют стандартное нормальное распределение. Отметим точку с коор(X, Y ) на плоскости. Пусть Z - квадрат расстояния до начала координат, а W - угол с
и
осью абсции.
а) Как распределены
Z
и
W?
б) Независимы ли они?
2
Ответы: Z ∼ χ2 , W ∼ U [0; 2π], независимы
Задача 12.27.
Ползи улитка по склону Фудзи, ползи улитка до самых высот... Басе (?)
По склону горы Фудзи ползет улитка. Каждое утро она принимает решение либо ползти вверх (с
вероятностью 0,9), либо целый день спать. Если улитка спит, то она во сне сползает вниз на 2 м.
Какова вероятность того, что за полгода улитка достигнет вершины Фудзи?
Тем, кто случайно забыл, напомним:
Высота Фудзи - 3770 м. Скорость виноградной улитки - 7 см в минуту.
Допустим, что приняв решение ползти вверх, улитка ползет вверх 7 часов, а остальное время
любуется видами (не сползая при этом вниз).
Задача 12.28. [Старый знакомый]
pX (t) = c · exp(−8t2 + 5t). Найдите E(X), σX .
5
1
полный квадрат, E(X) =
, σX =
16
4
Известна ф. плотности:
Ответ: выделяем
Задача 12.29.
Предположим, что каждый пятый горожанин предпочитает эскимо. Сколько горожан следует опросить, чтобы вероятность того, что выборочная доля горожан, предпочитающих эскимо, отличалась
от истинной доли менее чем на 0,05, равнялась 0,8?
Задача 12.30.
Пусть
X ∼ N (0; 1).
Выпишите ф. плотности
p(x|X > 1), p(x|X < 1), p(x|X ∈ [0; 1]).
Задача 12.31.
Пусть доналоговая прибыль предпринимателя равна
10X + 100,
где
X ∼ N (0; 1).
Если прибыль
меньше 110, то налог отсутствует, если прибыль больше 110, то налог равен 10%. Найдите ожидаемую посленалоговую прибыль.
Задача 12.32.
Пусть
X ∼ N (10; 16).
Найдите
a
в случаях:
P (X > a + 1) = 0, 5, P (|X − 10| < a) = 0, 25.
Задача 12.33.
Считая вероятность рождения мальчика равной 0,51, вычислите вероятность того, что среди 10000
новорожденных мальчиков будет больше, чем девочек.
Задача 12.34.
Пусть вероятность выпадения монетки «орлом» равна 0,63.
a) Какова вероятность, что в 100 испытаниях выборочная доля выпадения орлов будет отличаться
от истинной менее, чем на 0,07?
b) Каким должно быть минимальное количество испытаний, чтобы вероятность отличия менее чем
на 0,02 была больше 0,95?
77
Задача 12.35.
Каждый из 160 абонентов шлет в среднем 5 смс в сутки. Какова вероятность того, что за двое суток
они пошлют в сумме более 1700 сообщений?
Задача 12.36.
Пусть
X ∼ N (0, σ 2 ).
а) Найдите функцию плотности
б) Найдите
E(|X|)
|X|
(можно найти без функции плотности).
Задача 12.37.
Определите математическое ожидание и дисперсию случайной величины, если ее функция плотно2
сти имеет вид p(t) = c · exp(−2 · (t + 1) ).
Задача 12.38.
X1 , X2 , X3 - iid, N (0; 1). Для с.в. Y = X1 · X3 + X2 , W = X1 + X2 /(|X3 | + 1), Q =
(X1 + X2 )/(|X3 | + 1) найдите условные ф. плотности p(y|x3 ), p(w|x3 ), p(q|x3 ).
Пусть
Задача 12.39.
Известно, что
ln Y ∼ N (µ; σ 2 ).
Найдите
E(Y ), V ar(Y ).
Задача 12.40.
X ∼ N (0; 1). Найдите ф. плотности для Y = |X|, E(Y ). Верно ли,
0, 5(|X| + X)? Найдите E(max(X, 0)). Как изменится ответ, если X ∼ N (0; σ 2 )?
Пусть
что
max {X, 0} =
Задача 12.41. [т]
~ ∼ N (( 2 ); ( 9 −1 )). Как
X
3
−1 16
Найдите E(X1 |X2 = 4), E(X1 |X2 = a)
Пусть
выглядит (с точностью до константы) функция
p(x1 |x2 )?
Задача 12.42.
Каждый день цена акции равновероятно поднимается или опускается на один рубль. Сейчас акция
стоит 1000 рублей. Введем случайную величину
день. Найдите
E(Xi )
и
V ar(Xi ).
Xi ,
обозначающую изменение курса акции за
i
-ый
С помощью центральной предельной теоремы найдите вероятность
того, что через сто дней акция будет стоить больше 1030 рублей.
Задача 12.43.
Сейчас акция стоит 100 рублей. Каждый день цена может равновероятно либо возрасти на 8%, либо
упасть на 5%.
a) Какова вероятность того, что через 60 дней цена будет больше 170 рублей?
б) Чему равно ожидаемое значение цены через 60 дней?
Задача 12.44.
В среднем 20% покупателей супермаркета делают покупку на сумму свыше 500 рублей. Какова
вероятность того, что из 200 покупателей менее 81% сделают покупку на сумму не более 500 рублей?
Задача 12.45.
В самолете пассажирам предлагают на выбор «мясо» или «курицу». В самолет 250 мест. Каждый
пассажир с вероятностью 0.6 выбирает курицу, и с вероятностью 0.4 - мясо. Сколько порций курицы
и мяса нужно взять, чтобы с вероятностью 99% каждый пассажир получил предпочитаемое блюдо,
а стоимость «мяса» и «курицы» для компании одинаковая?
Как изменится ответ, если компания берет на борт одинаковое количество «мяса» и «курицы»?
Answers:
K = 170, M = 120
(симметричный интервал) или
K = M = 168
(площадь с одного края можно
78
принять за 0)
Вариант: театр, два входа, два гардероба а) только пары, б) по одному
Задача 12.46.
3
The mean selling price of new homes in a city over a year was 115000$. The population standard deviation
was 25000$. A random sample of 100 new homes sales from this city was taken. What is the probability
that the sample mean selling price was more than $ 110000? between $113000 and $117000?between
$114000 and $116000?
Задача 12.47.
The number of hours spent studying by students on a large campus in the week before final exams follows
a normal distribution with standard deviation 8,4 hours. A random sample of these students is taken to
estimate the population mean number of hours studying. How large a sample is needed to ensure that the
probability that the sample mean differs from the population mean by more than 2,0 hours is less than 0,05?
Задача 12.48.
A corporation receives 120 applications for positions from recent college graduates in business. Assuming
that these applicants can be viewed as a random sample of all such graduates, what is the probability
that between 35% and 45% of them are women if 40% of all recent college graduates in business are women?
Задача 12.49.
A video rental chain estimates that annual expenditures of members on rentals follow a normal
distribution with mean $100. It was also found that 10% of all members spend more than $130 in a year.
What percentage of members spend more than $140 in a year?
Задача 12.50.
It was found that 80% of seniors at a particular college had accepted a job offer before graduation. For
those accepting offers, salary distribution was normal with mean $29000 and standard deviation $4000.
For a random sample of sixty seniors, what is the probability that less than 70% have accepted job offers?
For a random sample of six seniors who have accepted job offers, what is the probability that the average
salary is more than $30000? A senior is chosen at random. What is the probability that he or she has
accepted a job offer with a salary of more than $30000?
Задача 12.51.
Портфель страховой компании состоит из 1000 договоров, заключенных 1 января и действующих
в течение года. При наступлении страхового случая по каждому из договоров компания обязуется
выплатить 30 тыс. рублей. Вероятность наступления страхового события по каждому из договоров предполагается равной 0,05 и не зависящей от наступления страховых событий по другим
контрактам. Каков должен быть совокупный размер резерва страховой компании для того, чтобы
с вероятностью 0,95 она могла бы удовлетворить требования, возникающие по указанным договорам?
Задача 12.52.
В данном регионе кандидата в парламент Обещаева И.И. поддерживает 60% населения. Сколько
нужно опросить человек, чтобы с вероятностью 0,99 доля опрошенных избирателей, поддерживающих Обещаева И.И., отличалась от 0,6 (истинной доли) менее, чем на 0,01?
Задача 12.53.
Обозначим долю людей, предпочитающих мороженое с шоколадной крошкой буквой
p.
Чтобы оце-
нить ее, Вася и Петя опросили 100 человек. Затем Вася ушел домой, а Петя опросил еще 200 человек.
Какова вероятность того, что Васин результат будет отличаться от Петиного более, чем на четыре
процентных пункта, если
При каком
3Problems
p
p = 0.6?
эта вероятность будет максимальной?
are shamelessly borrowed from Newbold
79
Задача 12.54.
На плоскости случайным образом выбираются 3 точки:
A, B
и
и нормально стандартно распределены. Какова вероятность
диаметром
C . Абсциссы и ординаты независимы
того, что C лежит внутри круга с
AB ?
Solution:
...?
Задача 12.55.
Найдите
E(max{X, Y }),
где
X
и
Y
нормально стандартно распределены и независимы.
Задача 12.56.
Data from a large population indicate that the heights of mothers and daughters in this population follow
the bivariate normal distribution with correlation 0.5. Both variables have mean 5 feet 4 inches, and
standard deviation 2 inches.
a) Among the daughters of above average height, what percent were shorter than their mothers?
b) Amont the daughters of above average height, what percent have an above average mother?
Solution:
Можно проигнорировать среднее и дисперсию, тогда задача примет вид:
D
и
M
N (0; 1) и имеют корреляцию 0.5
P (D < M |D > 0) или P (M > 0|D > 0)
распределены
Вопрос:
Можно взять требуемый двойной интеграл перейдя к полярным координатам.
geometric sense of correlation
Comment: maybe 12.24 be of use?
Задача 12.57.
Пусть
X1
и
X2 имеют
ρ.
совместное нормальное распределение, причем каждая
Xi ∼ N (0; 1),
а корре-
ляция равна
а) Выпишите в явном виде (без матриц) совместную функцию плотности
б) Пусть
ρ = 0.5.
Какое условное распределение имеет
X1
при условии, что
X2 = −1?
Задача 12.58.
(X, Y ) имеет двумерное нормальное распреE(X+Y ) = 10, E(X−Y ) = 30, V ar(X) = V ar(Y ) = 4 и V ar(X+Y ) = 6.
E(X), E(Y ), Cov(X, Y ), Cov(3X, −6Y ) и Corr(X, Y )
P (X − 2Y > 48)
Известно, что
X
и
Y
нормальны в совокупности (вектор
деление). Также известно, что
а) Найдите
б) Найдите
Ответы:
E(X) = 20, E(Y ) = −10, Cov(X, Y ) = −1, Cov(3X, −6Y ) = 18
b) E(X − 2Y ) = 40, V ar(X − 2Y ) = 24
a)
и
Corr(X, Y ) = − 14
Задача 12.59.
Z ∼ N (0, 1).
yZ− 12 y 2
1{Z+z≥0} = FZ (y + z)
Докажите, что E e
Пусть
Задача 12.60.
Складывают
n
чисел. Перед сложением каждое число округляют до ближайшего целого. Появ-
ляющуюся при этом ошибку можно считать равномерно распределенной на отрезке [−0.5; 0.5].
1
Определите, сколько чисел складывают, если с вероятностью
получаемая сумма отличается от
2
настоящей больше чем на 3 (в любую сторону).
Задача 12.61. Let X follow a N(0,1), and let N be the cdf of X. Compute A = E(N(X+1)).
Compute E(N(X)) (easy)
Answer:
p
n(x) be the pdf of X , namely 1/ (2π)exp(−x2 /2).
R +∞
A = x=−∞ N (x + 1)n(x)dx
Let
80
= int int n(y) n(x) dx dy, x = -infinity...infinity, y = -infinity .. x+1
= int F(x,y), (x,y) in D(x,y)
where, F(x,y) = n(x)n(y), and D(x,y) is the domain of integration, i.e. (x,y) / y < x+1
Now, moving to polar coordinates, we see that...
A = int 1/2pi exp(-r?/2) r dr dtheta, over a domain D’
... the function being integrated does not depend on theta, i.e. is invariant by rotation.
Coming back to cartesian coordinate, and rotating D yields:
A = int F(x,y) over D”, where D” = (x,y), x > -sqrt(2)/2
and thus, A = 1- N(sqrt(2)/2).
13. Без эксперимента. Мат. ожидание и дисперсия
Задача 13.1.
Случайная величина задана таблицей:
ωi
ω1 ω2 ω3 ω4
3
2
2
P (ωi ) 18
8
8
8
X(ωi ) 5 −3 2 0
Определите P (X > 1), P (X = 5),
1
P ( X−3
< 0). Верно ли, что события
P (X = 5|X > 0), P (X > 0|X = 5), P (X 2 > 7), P (X 3 < 0),
A = {|X| > 2} и B = {X > 0} являются независимыми?
Задача 13.2.
Случайная величина задана таблицей:
ωi
ω1 ω2 ω3 ω4 ... ωn ...
1
P (ωi ) 12 14 18 16
... 21n ...
Y (ωi ) 2 4 6 8 ... 2n ...
Найдите
P (Y > 14), P (Y 2 > 100|Y < 12), P (Y < 5|Y < 9), P (2Y < 16), P (Y 2 > Y + 6).
Задача 13.3.
Как распределена сумма
а)
б)
в)
X
X
X
и
и
и
Y
Y
Y
X +Y?
Если:
λX и λY .
биномиально с параметрами (nX , p) и (nY , p).
2
2
нормально с параметрами (µX , σX ) и (µY , σY ).
независимы, распределены по Пуассону с ожиданием
независимы, распределены
независимы, распределены
Задача 13.4.
X и Y одинаково
Ответ: V ar(X)
Пусть
распределены и независимы. Какой смысл имеет величина
1
E[(X
2
− Y ) 2 ]?
Задача 13.5.
Придумайте случайную величину со средним значением 5 и дисперсией 16.
Подсказка: можно ограничиться экспериментом с подбрасыванием правильной монетки.
Задача 13.6.
V ar(X) = σx2 , V ar(Y ) = σy2 . В каких
2
2
2
2
Ответ: [σx + σy − 2σx σy ; σx + σy + 2σx σy ]
Пусть
пределах может лежать
V ar(X + Y )?
Задача 13.7.
а) С помощью неравенства Чебышева укажите границы в которых находится
б) Чему равна указанная вероятность, если
Предполагается, что
E(X)
и
V ar(X)
X
нормально распределена?
существуют.
P (|X − E(X)| > 2σ).
81
Задача 13.8.
С помощью неравенства Чебышева, укажите границы, в которых находятся величины; рассчитайте
также их точное значение
P (−2σ < X − µ < 2σ), X ∼ N (µ; σ 2 )
b) P (8 < X < 12), X ∼ U [0; 20]
c) P (−2 < X − E(X) < 2), X - имеет экспоненциальное
а)
распределение с
λ=1
Задача 13.9.
Известно, что случайная величина
;
P (X = 2) = 0, 1 и E(X) = −0, 7.
V ar(X).
X
P (X = 1) = 0, 3
случайной величины X и
принимает три значения. Также известно, что
Определите чему равно третье значение
найдите
Задача 13.10.
E(X) и E(Y ),
FX (t) ≥ FY (t) для всех t.
Сравните
если известно, что функции распределения удовлетворяют соотношению
Задача 13.11.
Пусть
X
- случайная величина. Рассмотрим функцию
y(t) = E((X − t)2 ).
Найдите
E(y(X)).
Задача 13.12.
Используя свойства математического ожидания, докажите, что
V ar(X) = E(X 2 ) − (E(X))2 .
Выве-
Z = X − 2Y
будет
дите аналогичную формулу для ковариации.
Задача 13.13.
Докажите неравенство треугольника:
σX+Y ≤ σX + σY
Задача 13.14.
Пусть
V ar(X) = aV ar(Y )
и
Corr(X, Y ) = 0.7.
При каком значении
a
между
X
и
отсутствовать линейная взаимосвязь?
Задача 13.15.
С.в.
X1 , X2 ...
независимы,
P (Xi = 0) = P (Xi = 1) = 1/2, Z =
P+∞
1
i=1 2i Xi . Найдите
E(Z), σZ .
Задача 13.16.
Случайная величина задана таблицей:
ωi
ω1 ω2 ω3 ω4
3
2
2
P (ωi ) 18
8
8
8
X(ωi ) 5 −3 2 0
1
2
Найдите E(X), E(X ), E(
), E(X 2 |X > 0), E(X|X 2 < 10),
X+10
и
P (X ≤ t).
Задача 13.17.
ωi
ω1 ω2 ... ωn ...
P (ωi ) 12 14 ... 21n ...
Y (ωi ) 2 4 ... 2n ...
Найдите
E(Y ).
Задача 13.18.
Z имеет вид:
 1 − t, t ∈ [0; 1]
t + 1, t ∈ [−1; 0]
p(t) =

0, t ∈
/ [−1; 1]
2
2
Найдите E(Z), E(Z ), E(Z|Z > 0), E(Z ||Z| < 1/2), P (Z ≤ t)
Функция
 плотности с.в.
для произвольного
t,
постройте
82
график
P (Z ≤ t)
Задача 13.19.
Экономика
описывается системой уравнений
y = c − ai + IS
m − p = hy − ki + LM
Эндогенными переменными являются: y и i.
y - логарифм выпуска
i - процентная ставка
m - логарифм денежной массы
p - логарифм уровня цен
a > 0, h > 0, k > 0.
E(IS ) = E(LM ) = 0, Шоки IS и LM - независимы
а) Чему равна V ar(y), если для достижения нужного E(y), Центробанк
инструмента m?
б) Чему равна V ar(y), если для достижения нужного E(y), Центробанк
инструмента i (в этом случае кривая LM принимает вид i = const)?
использует в качестве
использует в качестве
Задача 13.20.
E(X) будет выполнено E(X 2 ) = V ar(X)? При каком условии на E(X) и E(Y )
E(XY ) = Cov(X, Y )?
При каком условии на
будет выполнено
Задача 13.21.
При каком значении числа
a
функция
f (a) = E((Y − a)2 )
будет наименьшей? Чему будет равно
наименьшее значение функции?
Ответ:
a = E(Y ), fmin = V ar(Y )
Задача 13.22.
При каком значении числа
a
функция
f (a) = E|Y − a|
будет наименьшей?
Решение:
При
a
равном медиане.
Возьмем мат. ожидание от
|Y − a| ≥ |Y − m| + (m − a)(P (Y < m) − P (Y > m))
Задача 13.23.
Пусть с.в.
X
принимает только натуральные значения.
P+∞
а) Верно ли, что X =
i=1 1X≤i ?
б) Возьмите математическое ожидание от левой и правой частей
Задача 13.24.
Автор книги получает 50 тыс. рублей сразу после заключения контракта и 5 рублей за каждую
проданную книгу. Автор предполагает, что количество книг, которые будут проданы - это случайная
величина с ожиданием в 10 тыс. книг и стандартным отклонением в 1 тыс. книг. Чему равен
ожидаемый доход автора? Чему равна дисперсия дохода автора?
Задача 13.25.
При каких условиях верно, что:
2
2
а) E(X ) = (E(X))
b)
E(XY ) = E(X)E(Y )
Задача 13.26. [Задача о божественной регрессии]
Вася знает абсолютно все характеристики
X,
Вася наблюдает только случайную величину
а про
X,
Y
ему известны только
E(Y )
и
Cov(X, Y ).
а хочет спрогнозировать случайную величину
stka
nsen
83
Y.
Строго говоря, Васина цель - построить с.в.
сия ошибки прогноза
Ŷ = α + βX ,
так чтобы
E(Ŷ ) = E(Y ),
и диспер-
V ar(Y − Ŷ ) была бы минимальной. Найдите оптимальные коэффициенты α и β
Задача 13.27.
Эксперты Вася и Петя сидят у телевизора и делают прогноз, сколько баллов получит выступающая
фигуристка. Результат фигуристки - случайная величина
Y,
Васин прогноз -
X1 ,
Петин -
X2 .
E(Y − X1 ) = E(Y − X2 ) = 0. Также известно, что ошибки прогнозов некоррелированы,
Cov(Y − X1 , Y − X2 ) = 0. Дисперсии ошибок прогнозов различны и равны V ar(Y − X1 ) = σ12 ,
V ar(Y − X2 ) = σ22 .
а) Поясните смысл условий E(Y − Xi ) = 0 и Cov(Y − X1 , Y − X2 ) = 0.
b) Рассмотрим прогноз X вида X = a1 X1 + a2 X2 и соответствующую ошибку прогноза e = Y − X .
Найдите числа a1 и a2 , так чтобы математическое ожидание ошибки равнялось нулю, а дисперсия
Известно, что
т.е.
была бы минимальной.
с) Верно ли, что из условия
σ12 > σ22
следует то, что
a1 < a2 ?
Задача 13.28.
Случайным процессом с дискретным временем называется последовательность случайных величин...X−2 ,
X−1 , X0 , X1 , X2 ... Случайный процесс называется стационарным (weak stationary), если E(Xt ) существует и не зависит от t; Cov(Xt , Xt−k ) существует и не зависит от t. Допускается, что Cov(Xt , Xt−k )
зависит от k .
а) Верно ли, что сумма двух стационарных процессов стационарна?
б) Верно ли, что сумма двух независимых стационарных процессов стационарна?
Ответы: а) - нет, б) - да
Задача 13.29.
а) Известно, что
б) Известно, что
E(Z) = −3 и E(Z 2 ) = 15. Найдите V ar(Z), V ar(4 − 3Z) и E(5 + 3Z − Z 2 ).
V ar(X + Y ) = 20 и V ar(X − Y ) = 10. Найдите Cov(X, Y ) и Cov(6 − X, 3Y ).
Задача 13.30.
Пусть
X ∼ U [0; 1], f (a) = E(X ∧ a), g(a) = E(X ∨ a)
f (a), f (f (a)), g(a), g(g(a))
Постройте
Задача 13.31.
При каких условиях
E( X1 ) =
1
?
E(X)
Задача 13.32. Бабушки и молоко
Баба Маша и баба Катя покупают молоко у бабы Нюры. Баба Маша каждый день покупает ровно
1 литр, а баба Катя - ровно на 10 рублей. В этом месяце баба Нюра продавала молоко в среднем по
10 рублей за литр. Цена каждый день могла быть разная. Кто больше купил молока? Кто больше
заплатил?
Трактовка 1. Средняя цена считается как среднее ежедневных цен.
Трактовка 2. Баба Маша и баба Катя - единственные клиентки бабы Нюры, а средняя цена считается
как выручка делить на количество проданного молока.
Задача 13.33.
Вася забрасывает удочку
забрасывании равна
a) Пусть
б) Пусть
N
раз, где
N
случайная величина. Вероятность поймать рыбку при одном
p.
N ≡ n. Чему
E(N ) = n.
равна вероятность того, что Вася поймает хотя бы одну рыбку?
Докажите, что вероятность поймать хотя бы одну рыбку не превосходит
вероятности из пункта «а»
Hint: неравенство Иенсена
84
Задача 13.34.
Допустим, что закон распределения
X
имеет вид:
X
1
2
3
Prob θ 2θ 1 − 3θ
E(X) , V ar(X)
каких θ среднее будет наибольшим? При каких - наименьшим?
каких θ дисперсия будет наибольшей? При каких - наименьшей?
а) Найдите
б) При
в) При
Ответы:
E(X) = 3 − 4θ, θ ∈ [0; 1/3], θmax = 0, θmin = 1/3
Задача 13.35.
В городе Туме случайным образом выбрали семейную пару. Стандартное отклонение возраста мужа
оказалось равным 5 годам, а стандартное отклонение возраста жены - 4 годам. Найдите корреляцию
возраста жены и возраста мужа, если стандартное отклонение разности возрастов оказалось равным
2 годам.
14. Функция плотности, условная функция плотности для одномерных
Задача 14.1.
1/5, t ∈ [2; 7]
0, t ∈
/ [2; 7]
P (4 ≤ X ≤ 6), P (X ≥ 5), P (X = 5), P (X > 5), P (X > 5|X > 3), P (−∞ < X < +∞)
Пусть у случайной величины
X
функция плотности имеет вид
p(t) =
. Определите
Задача 14.2.
Известно, что функция плотности случайной величины
p(x) =
Найдите значение константы
ления величины
X
имеет вид:
cx2 , 5A; 8 x ∈ [−2; 2]
0, 8 = 0G5
A, P (X > 1), E(X), E( X 31+10 )
и постройте график функции распреде-
X.
Задача 14.3.
Найдите
а)
б)
в)
sin
X
X
X
P (X ∈ [16; 23]),
если
E(X) = 20, V ar(X) = 25
равномерно распределена на отрезке [0; 30]
распределена экспоненциально и E(X) = 20
нормально распределена,
Задача 14.4.
Пусть величина
ны
X
распределена равномерно на отрезке
[0; π].
Найдите функцию плотности величи-
Y = sin(X)
Задача 14.5. [Mosteller, доп. вопросы] Короткий кусок стержня
а) Если стержень ломается случайным образом на две части, то какова средняя длина меньшего
куска?
б) Каково среднее отношение длины короткого куска к длине длинного куска?
в) Каково среднее отношение длины длинного куска к длине короткого куска?
д) Какой вид имеют функции плотности для случаев b, c?
е) Мода отношения длины короткого куска к длине длинного?
ж) Мода отношения длины длинного куска к длине короткого?
Задача 14.6. [Mosteller] Сломанный стержень
Стержень ломается случайным образом на три части. Найти средние длины короткого, среднего и
длинного кусков.
85
Задача 14.7.
Пусть
pX (t)
X.
Y = aX + b,
- функция плотности с.в.
a) Найдите функцию плотности с.в.
b) Как изменится ответ, если
если
a > 0.
a < 0?
Задача 14.8.
Цель игры - получить число, стоящее как можно ближе к единице, но не больше единицы. У Пети
две попытки. Сначала он узнает число, равномерно распределенное на
[0; 1].
Далее он выбирает
ограничиться ли этим числом, или прибавить к нему еще одно имеющее такое же распределение.
Затем Вася, зная петин результат тоже получает две попытки, но он не складывает получаемые
числа, а довольствуется последним заказанным числом.
а) У кого какие шансы на выигрыш?
б) Как выглядит оптимальная стратегия?
коммент: изложить поаккуратнее
Задача 14.9.
X распределена равномерно на [0; 1]. Найдите плотность
X
1
3
Y = ln 1−X
, Z = − ln X ( λ > 0 ), W = X , Q = X − 1/X .
λ
Пусть
распределения случайных величин
Задача 14.10.
Пусть
F (t)
R
- функция распределения случайной величины.
+∞
(F (t + a) − F (t))dt
Найдите
−∞
R +∞
Answer:
(F (t + a) − F (t))dt = a
−∞
Задача 14.11.
Пусть вероятность того, что случайная величина X лежит в промежутке от [a; b], где b
R b −t
e dt. Чему равны P (X = 17), P (0 ≤
определяется формулой P (a ≤ X ≤ b) =
a
P (1 ≤ X ≤ 2|X > 1), P (X ≥ 0), P (X < 0)?
≥ a ≥ 0,
X ≤ 1),
Задача 14.12.
С.в.
X
распределена равномерно на отрезке
[2; 8].
Найдите
E(X), E(X|X > 4), E(X 2 ).
Задача 14.13.
X ∼ U [0; 1], Z = X + Y ,
Y
0
3
P rob 0.4 0.6
Пусть
а
Y
задана табличкой:
а) Постройте функцию распределения
Z,
если
X
б) Постройте функцию распределения
Z,
если
Y =
в) Найдите
E(Z)
и
Y
независимы
0, X ∈ [0; 0.4]
3, X ∈
/ [0; 0.4]
в обоих случаях
Задача 14.14.
X - неотрицательная с.в. с функцией плотности p(t)
g(t) = c · tp(t) также будет функцией плотности?
Пусть
и
E(X) < ∞.
При каком
c
плотности
имеет
функция
Задача 14.15.
Распределение
pX (x) = λe−λx
X называется экспоненциальным,
x > 0. Найдите E(X), V ar(X).
с.в.
для
если
ее
функция
вид
Задача 14.16.
Пусть цена акции
A имеет ф. плотности p = 34 max {1 − (x − 1)2 , 0}. У Васи есть опцион-пут, дающий
ему право продать акции по цене 1,2 рубля (опцион пут). Какова вероятность исполнения опциона?
86
Каков ожидаемый Васин доход?
Задача 14.17.
Убыток от пожара - равномерно распределенная с.в. на
[0; 1].
Если убыток оказывается больше 0,7,
то страховая компания выплачивает компенсацию 0,7. Чему равны средние потери?
Задача 14.18.
3
max {1 − (x − 1)2 , 0}. У Васи есть опцион-колл,
4
дающий ему право купить акции по цене 1 рубль. Каков ожидаемый Васин доход?
Пусть цена акции
A
имеет ф. плотности
p =
Задача 14.19.
Пусть
[Ну
X
- с.в. с
pX (t) > 0
для всех
t.
Как распределены
Y = FX (X)
и
Z = − ln Y ?
хоть когда-то должна же функция распределения зависеть от случайной величины!
]
Задача 14.20.
Число
X
выбирается равномерно на отрезке
[0; 1]. Затем число Y
выбирается равномерно на отрезке
[0; X].
a) Найдите условную функцию плотности
b) Найдите
c) Найдите
pY |X (x, y)
pX,Y (x, y) и pY (y)
E(Y ), V ar(Y ), P (X + Y > 1)
Задача 14.21.
Величина
Z
равномерно принимает любое значение из отрезка [−3; 5]. Как выглядит ее функция
X = Z 2 . Найдите P (X ≤ 3) и P (X ≤ 7). Найдите P (Z < 2|X < 4), P (Z > X),
плотности? Пусть
P (X > 3|Z > 1).
Задача 14.22.

 1 − t, t ∈ [0; 1]
t + 1, t ∈ [−1; 0]
Пусть у случайной величины X функция плотности имеет вид p(t) =

0, t ∈
/ [−1; 1]
делите P (0, 4 ≤ X ≤ 0, 6), P (X ≥ 0, 5), P (X > −0, 5|X < 0, 3), P (−∞ < X < +∞). Верно
события A = {|X| > 0, 5} и B = {X > 0} являются независимыми?
. Опрели, что
Задача 14.23.
X1 и X2
[0; 1].
Найдите P (max{X1 , X2 } > 0, 5)
Функцию плотности fmax{X1 ,X2 } (t)
Пусть случайные величины
независимы,
X1
распределена равномерно на
[−2; 1], X2
-
равномерно на
a)
б)
Задача 14.24.
Z = max(X1 , X2 ) и случайные величины X1 и X2 независимы. Найдите функцию распреFZ (t), если известны функции распределения FX1 (t) и FX2 (t). Для случая X1 ∼ U [0; 5],
X2 ∼ U [0; 3] найдите E(Z).
Пусть
деления
Задача 14.25.
Пусть X1 , X2 и X3 Y = max {X1 , X2 , X3 }.
независимы и равномерны на отрезке
[0; 1].
Найдите функцию плотности
Задача 14.26.
Пусть
X1 , X2 , ..., Xn независимы и экспоненциально
E(min{X1 , ..., Xn }), E(max{X1 , ..., Xn })
Найдите
Solution:
распределены с параметром
λ.
87
max ∼ X1 +
X2
2
+ ... +
Xn
n
Задача 14.27.
X, Y
Пусть
и
Z
независимы и равномерны на
[0; 1].
Какова вероятность того, что можно построить
треугольник со сторонами таких длин?
Задача 14.28.
С.в.
X
[a; b]. a) Найдите E(X)
E(X|X > c), V ar(X|X > c)
распределена равномерно на отрезке
b) Пусть
c ∈ [a; b].
Найдите
и
V ar(X).
Задача 14.29. Про расстояние между минимумом и максимумом
Вася называет три числа на интервале
[0; 1]
наугад.
а) Какова вероятность того, что разница между наибольшим и наименьшим будет меньше 1/2?
R
б) Пусть
- разница между наибольшим и наименьшим числом. Найдите функцию плотности
случайной величины
R
Решение:
x < y < z.
а) Рассмотрим случай
dx +
R1 R
1/2
x
1
R1
y
·dz · dy · dx =
В этом случае вероятность равна
P =
R 1/2 R
0
xx+1/2
R x+1/2
y
·dz · dy ·
1
12
Так как возможно 6 вариантов расположения трех чисел, то искомая вероятность равна P
1
b) Взяв t вместо
получаем функцию распределения, а затем и функцию плотности p(r)
2
Geometric (intuitive) solution is welcome!
1
= 6· 12
= 12
= 6r(1 − r)
Задача 14.30.
Случайные величины
X1 ..., Xn
независимы и равномерно распределены на отрезке
[0; 1]
Найдите:
Xmin , E(Xmin )
b) Функцию плотности Xmax , E(Xmax )
c) E(Xmax |Xmin > c)
а) Функцию плотности
1
Solution: a)
n+1
n
b)
n+1
1
c) intuition c
n+1
n
+ 1 n+1
Задача 14.31.
Имеется равномерная на
[0; 1]
случайная величина. Как ее преобразовать, чтобы получилась экспо-
ненциальная со средним значением
m?
Задача 14.32.
Пусть
X
равномерна на
[−2; 1],
Y
а
Y
- расстояние от числа
X
до числа
(−1).
Найдите фукнцию плотности
Задача 14.33.
Пусть
X
величина
распределена экспоненциально с параметром
X/2?
λ = 1.
Какую функцию плотности имеет
Как называется такое распределение?
Answer: экспоненциальное,
λ = 2.
Задача 14.34.
X распределена экспоненциально с параметром λ = 1.
части X . Найдите среднее значение дробной части.
e1−t e−2
Answer: p(t) =
,
e−1 e−1
Пусть
Найдите функцию плотности дробной
Задача 14.35.
Известно, что
X
равномерно распределена на
[−1; 1]
и
Y = tg( Xπ
).
2
form
etki
chi2
88
Найдите функцию плотности
Y
k
[0; 1]
Задача 14.36. Максимум равномерных в степени
Пусть величины
X1 ,
...,
M = max{Xi }.
Xn равномерны
E(M k )
на
и независимы. Пусть
k
- натуральное число и
Найдите
n
n+k
Challenge: find intuitive, without integral, explanation (for k=1 we may break the interval into (n+1)
Answer:
equidistributed parts)...
Задача 14.37.
Пусть
U
Как распредлена величина
Ответ:
[0; 1].
Y = 1 + bln(U/ln(q)c?
- распределена равномерно на отрезке
Y
имеет геометрическое распределение.
Задача 14.38.
Пусть св.
X
принимает значения из промежутка
закон распределения
[a; b]
(возможно не все),
X.
Ответ: это максимально возможная дисперсия для такой величины,
значения
a
и
V ar(X) =
X
(b−a)2
. Найдите
4
равновероятно принимает
b.
15. Арифметика
Задача 15.1.
Докажите, что
Sn −E(Sn )
X̄n −E(X̄n )
√
=
=√
V ar(Sn )
V ar(X̄n )
X̄n −µ
q
, где
σ 2 /n
Xi
- iid,
E(Xi ) = µ, V ar(Xi ) = σ 2
Задача 15.2.
p ∈ [0; 1), G = p + p2 + p3 + p4 + ... и S = 1p + 2p2 + 3p3 + 4p4 + ...
а) Чему равняется сумма G − pG? Чему равняется G?
б) Чему равняется S − pS ?
dG
в) Чему равняется p
?
dp
г) Чему равняется S ?
Пусть
Задача 15.3.
R1
xa (1 − x)b dx
0
a) Проинтегрировав по частям, докажите, что
Пусть
б) Как
f (a, b) =
f (a, b) выражается
f (a, b)
через
b
f (a, b) = f (a + 1, b − 1) a+1
f (a + b, 0)?
с) Найдите
Задача 15.4.
Пусть
P
pi = 1
Докажите, что
P
p̂i = 1
P (np̂i −npi )2
и
npi
=n
P
p̂2i
pi
−1
Задача 15.5.
Может ли ковариационная матрица иметь вид
(
4 2
4 7
9 0
9 2
), (
), (
), (
)?
−1 9
7 9
0 −1
2 1
Задача 15.6.
Xi ∼ N (0; 1) и независимы. Пусть Z1 = a1 (X1 − X2 ), Z2 = a2 (X1 + X2 − 2X3 ), Z3 =
a3 (X1 + X2 + X3 − 3X4 ) и т.д.PНайдите такие ai > 0, чтобы V ar(ZP
i ) = 1. Для этих ai : Найдите E(Zi ),
Pn−1 2
3
2
2
2
Cov(Zi , Zj ). Докажите, что i=1 (Xi − X̄) = Z1 + Z2 , и что [т] ni=1 (Xi − X̄)2 = i=1
Zi .
Пусть
89
Задача 15.7.
Xi независимы, причем ∀i E(Xi ) = µ, а V ar(Xi ) = σ 2 . Найдите V ar(X1 + X2 ), E(X1 + X2 + ... +
Xn ), V ar(X1 + X2 + ... + Xn ), E(X̄), V ar(X̄), E(X12 ), Cov(X1 + X2 , X2 + X3 ), E(X1 · X2 ), E((X1 − X̄)2 ),
E(σ̂ 2 ).
Пусть
Задача 15.8. [Почему степеней свободы (n − 1)?]
P
2
i −X̄)
2
Пусть {Xi } - iid N (µ; σ ) и σ
b2 = (Xn−1
. Для n = 2 докажите, что σ
b2 можно представить в виде
2
Y
b2
σ
b2 = 11 , где Y1 ∼ N (0; σ 2 ). Как выражается Y1 через X1 и X2 ? a) Для n = 3 докажите, что σ
Y 2 +Y 2
можно представить в виде σ
b2 = 1 2 2 , где {Yi }- iid N (0; σ 2 ). Как выражаются Yi через {Xj }? b) Как
выглядит данное представление для произвольного n? Подсказки: Y3 = const · (X1 + X2 + X3 − 3X4 ),
для доказательства независимости двух нормальных случайных величин достаточно доказать, что
их ковариация равна нулю.
Задача 15.9.
Докажите, что
Pn
1
i=1 i
limn→∞
ln n
=1
Задача 15.10.
Верно ли, что
V ar(a1 X1 + a2 X2 ) = ( a1 a2 ) · C · ( a1 a2 )t ,
где
C
- ковариационная матрица?
Задача 15.11.
Упростите
E(6X −2), V ar(5−3X), Cov(7X −2, 4−5Y ), σ(2−3X) , cor(5+2X, 6−7Y ), cor(5−3X, 6X +8).
V ar(X + Y ), V ar(2X + 3Y ) и V ar(X − Y ).
Представьте в виде суммы
16. Двумерные распределения и ковариации
Задача 16.1.
Пусть
P (A|B) > P (A).
Что можно сказать про
Cov(1A , 1B )?
Задача 16.2.
pX|Y (x|y),
Чему равна
если случайные величины
X
и
Y
независимы?
Задача 16.3.
Y X=1
1
?
2
?
3
0
2
?
0
?
3
?
?
0
; Известно, что
P (Y = 1|X = k) = 1/3, P (X = k|Y = 1) = k/6
а) Заполните пропуски
б) Найдите
E(XY ).
Задача 16.4.
Y X=0
3
6
1
?
?
?
2
0, 1 0, 05 ?
Известно, что
а) Заполните пропуски
б) Найдите
E(X/Y ).
Задача 16.5.
Y X=1
2
3
1
0, 1
0, 2 0, 3
2 0, 15 0, 15
?
3 0, 05
0
0, 05
X
и
Y
независимы,
P (Y = 2|X = 0) = 1/4.
для всех
k.
90
а) Заполните пропуски
P (X > 2), P (X = 1|Y = 1), P (Y = 1|X = 2), P (X = 1|X = 2), P (X = 1 ∩ Y = 1).
б) Найдите
Задача 16.6.
Совместный закон распределения
X
и
Y
задан таблицей:
Y = −2 Y = 0 Y = 1
X = −1
X=0
Найдите
0,1
0,1
0,2
p
0,1
p, P (|X| ≥ |Y |), E(XY ), E(X|Y = 0), Cov(X, Y ), Corr(X, Y )
0,2
Задача 16.7.
Пусть
X
и
Y
независимы, одинаково непрерывно распределены.
Верно ли, что
E(X|X > Y ) = E(max{X, Y })?
Задача 16.8.
Совместный закон распределения случайных величин
X
и
Y
задан таблицей:
Y = −1 Y = 0 Y = 2
X=0
0, 2
c
0, 2
X=1
0, 1
0, 1
0, 1
Найдите
c, P (Y > −X), E(X · Y 2 ), E(Y |X > 0), Cov(X, Y ), Corr(X, Y )
Задача 16.9.
Совместная функция плотности имеет вид:
2 − x − y, если x ∈ [0; 1], y ∈ [0; 1]
0,
иначе
Найдите P (Y > 2X), E(Y ), E(XY ) Cov(X, Y ), E(X|Y > 0, 5), частную (предельную) функцию
плотности pY (t), условную функцию плотности pX|Y (x|y), E(X|Y ). Верно ли, что величины X и Y
pX,Y (x, y) =
являются независимыми? Ответы:
P (Y > 2X) = 7/24, E(XY ) = 1/6), E(X|Y > 0.5) =
X и Y зависимы
5
E(X) = E(Y ) = 12
7/48
3/8
= 7/18
Задача 16.10.
Пусть
X
Y E(XY ), Cov(X, Y ), Corr(X, Y )
- сумма очков, выпавших в результате двукратного подбрасывания кубика. Пусть
разность очков (число на первом минус число на втором). Найдите
Задача 16.11.
Найдите ковариацию, корреляцию и дисперсию суммы двух независимых случайных величин.
Задача 16.12.
Пусть
X
и
многочлена
Y равномерны на [0; 1]
t3 − X 2 t + Y = 0.
и независимы. Найдите закон распределение числа корней
Задача 16.13.
You roll 2 dice. X is the number of 1s shown and Y is the number of 6’s. Each of X, Y can take the values
0, 1or 2. What is the joint distribtuion p(x,y) the covariance cov(X,Y) and E(X|Y)?
Solution: The covariance of X and Y is -1/18. The expected value of X, conditional on Y, is (2-Y)/5.
Задача 16.14.
Вероятность дождя в субботу 0.5, вероятность дождя в воскресенье 0.3. Корреляция между наличием дождя в субботу и наличием дождя в воскресенье равна
r.
Какова вероятность того, что в выходные вообще не будет дождя?
91
Ответ:
0.5r ·
√
0.21 + 0.35
Задача 16.15.
Пусть
X
p1 , Пусть Y имеет геометрическое
min{X, Y }? Прокомментируйте
имеет геометрическое распределение с параметром
p2 . Как распределена
p = 1 − (1 − p1 ) · (1 − p2 )
распределение с параметром
Ответ: геометрически с
величина
Задача 16.16.
Время обслуживания клиента в окошке А - с.в., имеющая экспоненциальное распределение с
в окошке В - с.в.
B , имеющая экспоненциальное распределение с λ = 12. Величины и B
(A + B), P (2A > B).
λ = 7,
независимы.
Найдите функцию плотности
Задача 16.17.
Вася решает тест путем проставления каждого ответа наугад. В тесте 5 вопросов. В каждом вопросе 4 варианта ответа. Пусть
X
- число правильных ответов,
Y
- число неправильных ответов и
Z =X −Y.
а) Найдите
б) Найдите
в) Найдите
P (X > 3)
V ar(X) и Cov(X, Y )
Corr(X, Z)
Задача 16.18.
Какова вероятность того, что квадратное уравнение с действительными коэффициентами имеет два
различных корня?
2
а) ax + bx + c = 0, где a,
2
b) ax + bx + c = 0, где a,
2
с) x + bx + c = 0, где b и
b и c равномерны на [−1; 1]
b и c равномерны на [0; 1]
c равномерны на [0; 1]
Задача 16.19.
В коробке 6 красных и 2 зеленых пуговицы. Пуговицы вытаскивают наугад до появления двух
одноцветных. Пусть
X
- общее количество извлеченных пуговиц в ходе эксперимента, а
Y
- количе-
ство извлеченных в ходе эксперимента зеленых пуговиц. Какова вероятность того, что эксперимент
окончится при извлечении третьей пуговицы? Найдите
E(X), V ar(X), Cov(X, Y ).
Задача 16.20.
Из урны с 5 красными и 4 синими шарами достаются 3 шара. Найдите закон распределения числа
2
красных шаров N . Найдите E(N ), V ar(N ), Cov(N, N ).
Задача 16.21.
X - максимальный
X , E(X), V ar(X), Cov(6X − 3, X + 4)
Из урны с 5 занумерованными шарами достается 3 шара. Пусть
трех номеров. Найдите закон распределения
из полученных
Задача 16.22.
Ω = {a, b, c}, X(a) = 1, X(b) = 2, X(c) = −2 P (a) = P (b) =
величины Y , такие, что E(Y ) = 0, Cov(X, Y ) = 0 и V ar(Y ) = 1.
Пусть
1
,
4
P (c) =
Задача 16.23.
Y = −1 Y = 0 Y = 2
X=0
0, 2
0, 1
0, 1 ;
X=1
0, 3
0, 1
0, 2
2
a) Найдите E(X), E(Y ), E(XY ), E(X ), V ar(X), V ar(Y ), Cov(X, Y ).
б) Найдите P (X = 1|Y = −1).
в) Постройте функции распределения с.в. X и Y .
1
. Найдите все случайные
2
92
P (X > Y ), P (X + 2Y < 0, 5)
г) Найдите
Задача 16.24.
√
3
√
6 − x − y, if x, y ≥ 0, x + y ≤ 3 6
;
pX,Y (x, y) =
0, otherwise
2
a) Найдите E(X), E(Y ), E(XY ), E(X ), V ar(X), V ar(Y ), Cov(X, Y ).
б) Найдите функции плотности с.в. X , Y и Z = X + Y .
в) Постройте функции распределения с.в. X и Y .
г) Найдите P (X > Y ), P (X + 2Y < 0, 5)
Задача
16.25.
if x, y ≥ 0, x + y ≤ 1
otherwise
2
a) Найдите E(X), E(Y ), E(XY ), E(X ), V ar(X), V ar(Y ), Cov(X, Y ).
б) Найдите функции плотности с.в. X , Y и Z = X + Y .
в) Постройте функции распределения с.в. X и Y .
г) Найдите P (X > Y ), P (X + 2Y < 0, 5)
2,
0,
p=
Задача 16.26.
Совместная функция плотности имеет вид
x + y, если x ∈ [0; 1], y ∈ [0; 1]
0,
иначе
Найдите P (Y > X), E(X), E(X|Y > X), Cov(X, Y ), частную (предельную) функцию плотности
pY (t), условную функцию плотности pX|Y (x|y), E(X|Y ). Верно ли, что величины X и Y являются
pX,Y (x, y) =
независимыми?
Задача 16.27.
X
Пусть
равновероятно принимает значения -1, 0, +1. а) Найдите
Cov(X, Y ),
если
Y = X 2.
X и Y независимы?
Cov(X, Y ) = 0, X и Y зависимы
б) Верно ли, что
Ответ:
Задача 16.28.
Пусть
X
и
Y
X равновероятно
суммы (X + Y ).
независимы и имеют одинаковый закон распределения. С.в.
мает натуральные значения от 1 до 3. Найдите закон распределения
прини-
Задача 16.29.
X
и
Y
независимы и равномерны на отрезке
б) Найдите функцию плотности
[0; 1].
a) Найдите функцию плотности
Z =X +Y.
Z = XY
Ответы:

t, если t ∈ [0; 1)
2 − t, если t ∈ [1; 2)
p(t) =

0, иначе
−ln(t), если t ∈ (0; 1)
p(t) =
0, иначе

Задача 16.30.
c, частные
функции плотности pX (t)
c · (x + y), if x ∈ [1; 2] , y ∈ [0; 2]
p(x, y) =
0, otherwise
Найдите
а)
pY (t), E(X),E(Y ), E(X · Y ), Cov(X, Y ) :
c, if x ≥ 0, y ≥ 0, x + y ≤ 1
; б) p(x, y) =
0, otherwise
и
Задача 16.31.
С точностью до константы найдите функцию плотности с.в.
В «б» дополнительно найдите значение c.
2
2
) ; б) pX,Y (x, y)
pX,Y (x, y) = c · exp(− x +y
2
а)
=
c
1+x2 +x2 y 2 +y 2
X.
Верно ли, что
X
и
Y
независимы?
umma
93
Задача 16.32.
A(0, 0), B(2, 0) и C(1, 1) ф. плотности имеет вид:
а) p(x, y) = c(x + 2y) ; б) p(x, y) = cxy ; в) p(x, y) = c.
Вне треугольника плотность равна нулю. Найдите c, pX (t) и pY (t), E(X), E(Y ), E(X · Y ).
В треугольнике образованном точками
Задача 16.33.
Возможно ли, что
U =X +Y, X
и
Y
- iid, а
U
- равномерна на
[0; 1]?
Задача 16.34. Внутри круга радиуса 1 равномерно выбирается точка. Пусть
X
и
Y
- ее абсцисса
и ордината. Найдите совместную функцию плотности p(x, y), частную функцию плотности
2
условную функцию плотности p(x|y), E(X|Y ), E(X |Y ), Cov(X, Y ).
Являются ли
X
Y
и
p(x),
независимыми?
Задача 16.35. Вася может получить за экзамен равновероятно либо 8 баллов, либо 7 баллов. Петя
может получить за экзамен либо 7 баллов - с вероятностью 1/3; либо 6 баллов - с вероятностью 2/3.
Известно, что корреляция их результатов равна 0.7.
Какова вероятность того, что Петя и Вася покажут одинаковый результат?
Задача 16.36.
Suppose X, Y are random variables with joint density
a) What is the density of
b) What is
Y?
What is
f (x, y) = a3 xe−ay
for
0<x<y
and 0 otherwise.
E(Y )?
E(X|Y = 1)?
Answers:
3
p(y) = a2 y 2 e−ay ,
y>0
E(Y ) = 3/a
E(X|Y = 1) = 1
source: aops, t=177445
Задача 16.37.
Пусть
X 1 , X2 ,
...,
Xn
- iid
U [0; 1]
а) Найдите функцию плотности для порядковой статистики
б) Что изменится, если
Xi
- iid с функцией плотности
p(t)
X(k)
и функцией распределения
F (t)?
Solution:
http://en.wikipedia.org/wiki/Order_statistic
Задача 16.38.
Рассмотрим кольцо, задаваемое системой неравенств:
x2 + y 2 ≥ 1
и
x2 + y 2 ≤ 4.
Случайным образом,
равномерно на этом кольце, выбирается точка.
Пусть
X
и
Y
- ее координаты.
а) Чему равна корреляция
б) Зависимы ли
X
и
X
и
Y?
X
несут в себе одинаковую информацию об
Y?
Ответы:
а) 0, т.к. и рост, и падение
б) Зависимы, т.к.
X
содержит информацию об
Y
Y
Задача 16.39.
Say
X
density
f (x) = 3x−4 if x > 1, and 0 otherwise.
Y = (X1 X2 ...X16 )1/16 . Find E(Y ) and V ar(Y ).
has a density
f.
Let
Now say
X1 , ..., X16
are independent with
Solution:
Найдем функцию плотности
ром
ln(Xi ).
Окажется, что это экспоненциальное распределение с парамет-
λ=3
Находим закон распределения суммы
Находим закон распределения
Y.
ln(Xi ).
94
Source: aops, t=187872
Задача 16.40.
Say X and Y are independent random variables with densities f and g, respectively:
f (x) = e−x if x > 0
g(y) = 2e−2y if y > 0
Calculate a density function for
Y /(X + 1)
Solution:
We begin by considering the cumulative distribution function of Z = Y /(X + 1):
Y
R∞
R∞
FZ (z) = Pr X+1
≤ z = Pr[Y ≤ z(X + 1)] = x=0 Pr[Y ≤ z(X + 1)|X = x]fX (x) dx = x=0 FY (z(x +
1))fX (x) dx . Since Y is exponentially
with mean 1/2, it follows that the CDF of Y is simply
R ∞ distributed
e−2z
−2y
−2z(x+1) −x
FY (y) = 1 − e . Hence FZ (z) = x=0 (1 − e
)e dx = 1 − 1+2z
. Therefore the density of Z is the
4e−2z (1+z)
derivative of the CDF, or fZ (z) =
.
(1+2z)2
Source: aops, t=187870
Задача 16.41.
С.в.
X
принимает значения 1 и 2 с вероятностями 0,3 и 0,7 соответственно. С.в.
Y
принимает те же
X и Y равна r.
Z = XY если r = 0.5
значения с вероятностями 0,5 и 0,5. Корреляция между
a) Найдите закон распределения величины
б) При каком
r
значение
E(XY )
будет максимальным?
Задача 16.42.
Пусть величины X и Y имеют совместную функцию плотности:
p(x, y) = 41 (1 + xy), при |x| < 1, |y| < 1.
а) Верно ли, что X и Y независимы?
2
2
б) Верно ли, что X и Y независимы?
Задача 16.43.
Пусть
от
Xi
X1
и
X2
независимы и имеют стандартное нормальное распределение; величина
равновероятно принимает значения 1 или -1. Пусть также
а) Верно ли, что
б) Верно ли, что
Y
независимо
Zi = Y Xi .
Z1 и Z2 независимы?
Z12 и Z22 независимы?
Задача 16.44.
Пусть совместный закон распределения X и Y задан следующим образом:
1
3
5
8
p1,1 = p−1,1 = 32
, p−1,−1 = p1,−1 = p1,0 = p0,1 =
, p−1,0 = p0,−1 =
, p0,0 =
.
32
32
32
а) Верно ли, что X и Y независимы?
2
2
б) Верно ли, что X и Y независимы?
Задача 16.45.
Z ∼ U [0; 2π], X = cos(Z), Y = sin(Z).
X и Y независимы?
Найдите Corr(X, Y )
Пусть
а) Верно ли, что
б)
Задача 16.46.
X ∼ N (0; 1) и Y = X 2 − 1.
Верно ли, что X и Y независимы?
Найдите Corr(X, Y )
Пусть
а)
б)
Задача 16.47.
Приведите пример таких
X
и
Y,
что
P (X = Y ) = 0,
однако
X
и
Y
одинаково распределены.
ting
etka
95
Задача 16.48.
Приведите пример таких
X, Y
и
Z,
что
X
и
Y
одинаково распределены, а
XZ
и
YZ
имеют разное
распределение.
Задача 16.49.
X∼U
1−
− 1|
[0; 1], Y =πX
X , Z = |2X πX
, с помощью этих величин определим
πX
X1 = tg 2 , Y1 = tg 2 , Z1 = −2tg 2 ,
πY
πZ
X2 = tg πX
, Y2 = tg
, Z1 = −2tg
,
2
2
2
a) Верно ли, что X , Y и Z одинаково распределены?
б) Верно ли, что X1 , X2 , Y1 , Y2 одинаково распределены?
в) Верно ли, что Z1 и Z2 одинаково распределены?
г) Найдите E(X1 + Y1 + Z1 ) и E(X2 + Y2 + Z2 ). Почему они не равны?
Пусть
также:
Задача 16.50.
Ω = {ω1 , ..., ω9 }, где ω1 ,...,ω6 - перестановки чисел 1, 2 и 3, а ω7 = (1, 1, 1), ω8 = (2, 2, 2),
ω9 = (3, 3, 3). Пусть каждый исход ω9 имеет вероятность f rac19. Проведем одно испытания и
определим величину Xk , равную тому числу, которое находится на месте с номером k , k = 1, 2, 3.
а) Верно ли, что величины X1 , X2 и X3 попарно независимы?
б) Можно ли выразить X3 через X1 и X2 ?
Пусть
Задача 16.51. Пусть
X, Y , Z
- независимы и равномерны на
Z
Как распределена величина (XY ) ?
[0; 1].
Ответ: равномерно :)
Here is my “intuitive” explanation:
Z
Let W = log(XY ) = Z(logX + logY ).
We know that the log of a uniform random variable has the distribution of an exponential random variable,
therefore
logX
and
logY
are 2 independent expo(1). We can interpret them as the inter-arrival times of a
poisson process with intensity 1. Given the second arrival time T2 of a PP(1), it is well known that the
first arrival time is uniformly distributed between 0 and T2. An other way of saying that is
Z ∗ T 2 has the
same distribution of the first arrival time of a PP(1), with Z Unif[0,1]. We can conclude by seeing that
T 2 = logX + logY , so W = Z ∗ T 2
W
PP(1)) and e
is Unif[0,1]
Задача 16.52. Пусть
X, Y
is an exponential random variable with parameter1 (first arrival of a
- независимы и равномерны на
[0; 1].
Как распределена величина
Z=
X
?
Y
17. Смешанные распределения
Задача 17.1.
Q pQ (t) = ....
Пусть
случайная величина, принимающая значения на
Вася узнает значение
он передает монетку
[0; 1]
и имеющая функцию плотности
Q и изготавливает монетку, выпадающую орлом с данной вероятностью. Затем
Пете. Петя знает p(t), но не знает, какое конкретно значение приняло Q. Петя
подкидывает монетку и она выпадает орлом.
Как Пете следует подправить свое мнение
pQ (t|монетка
pQ (t)?
Т.е. найдите условную функцию плотности
выпала орлом).
Задача 17.2.
C.в.
X ∼ U [0; 1]. Вася изготавливает неправильную монетку, которая выпадает «орлом» с вероятноx и передает ее Пете. Петя не знает x. Он подкинул монетку один раз. Она выпала «орлом».
стью
Какова вероятность того, что она снова выпадет «орлом»? Как выглядит ответ, если Пете известно,
что монетка при
n dlia
подбрасываниях k раз выпала
ravnomernoi monetki
Подсказка: задача 15.3
орлом?
96
Задача 17.3.
Петя сообщает Васе значение случайной величины, равномерно распределенной на отрезке [0; 4]. С
1
3
вероятностью
Вася возводит Петино число в квадрат, а с вероятностью
прибавляет к Петиному
4
4
числу 4. Обозначим результат буквой Y .
Найдите
P (Y < 4)
и функцию плотности случайной величины
Y.
Вася выбирает свое действие независимо от Петиного числа.
Задача 17.4.
Допустим, что оценка
X
за экзамен распределена равномерно на отрезке

 0, if X < 30
X, if X ∈ [30; 80]
Y рассчитывается по формуле Y =
 100, if X > 80
2
Найдите E(Y ), E(X · Y ), E(Y ), E(Y |Y > 0).
[0; 100].
Итоговая оценка
.
Задача 17.5.
Рассмотрим треугольник с вершинами
(0; 0), (1; 0)
и
границе треугольника (не внутри треугольника!). Пусть
(1; 1). Точка A выбирается равномерно на
X и Y - абсцисса и ордината получившейся
точки.
E(XY ), Cov(X, Y )
вопрос, если точка A
а) Найдите
б) Тот же
выбирается равномерно внутри треугольника
source: aops, t=173773
√
5( 2−1)
imho:
6
18. Данетки
Corr(X, Y ) > 0.
E(XY ) > E(X)E(Y )?
n001
1. Известно, что
n002
2. Сумма двух нормальных независимых случайных величин нормальна?
n003
3. Сумма любых двух непрерывных случайных величин непрерывна?
n004
4. Нормальная случайная величина может принимать отрицательные значения?
n005
5. Пуассоновская случайная величина является непрерывной?
n006
6. Сумма двух независимых равномерно распределенных величин равномерна?
n007
7. Дисперсия суммы зависимых величин всегда больше суммы дисперсий?
n008
8. Дисперсия пуассоновской с.в. равна ее математическому ожиданию?
n009
9. Если
n010
10. Теорема Муавра-Лапласа является частным случаем центральной предельной?
n011
11. Для любой случайной величины
n012
12. Если
n013
13. Функция распределения случайной величины является неубывающей?
n014
14. Дисперсия случайной величины не меньше, чем ее стандартное отклонение?
X
- непрерывная с.в.,
X
Верно ли, что
E(X) = 6
и
V ar(X) = 9,
то
Y =
X−6
3
∼ N (0; 1)?
E(X|X > 0) ≥ E(X)?
- случайная величина, то
V ar(X) = V ar(16 − X)?
97
E(X 2 ) ≥ (E(X))2 ?
n015
15. Для любой случайной величины
n016
16. Если ковариация равна нулю, то случайные величины независимы?
n017
17. Значение функции плотности может превышать единицу?
n018
18. Если события
n019
19. Для любых событий
n020
20. Функция плотности может быть периодической?
n021
21. Для неотрицательной случайной величины
n022
22. Если
n023
23. В тесте Манна-Уитни предполагается нормальность хотя бы одной из сравниваемых выборок?
n024
24. График функции плотности случайной величины, имеющей
X ∼ χ2n
A
и
и
B
не могут произойти одновременно, то они независимы?
A
и
B
верно, что
Y ∼ χ2n+1 , X
Y
и
P (A|B) ≥ P (A ∩ B)?
E(X) ≥ E(−X)?
- независимы, то
X
не превосходит
Y?
t-распределение
симметричен отно-
сительно 0?
n025
25. Мощность больше у того теста, у которого вероятность ошибки 2-го рода меньше?
n026
26. Если
n027
27. При прочих равных 90% доверительный интервал шире 95%-го?
n028
28. Несмещенная выборочная оценка дисперсии не превосходит квадрата выборочного среднего?
n029
29. Если гипотеза отвергает при 5%-ом уровне значимости, то она будет отвергаться и при 1%-ом
X ∼ tn ,
то
X 2 ∼ F1,n ?
уровне значимости?
n030
30. У t-распределения более толстые «хвосты», чем у стандартного нормального?
n031
31. P-значение показывает вероятность отвергнуть альтернативную гипотезу, когда она верна?
n032
32. Если t-статистика равна нулю, то P-значение также равно нулю?
n033
33. Если
n034
34. Пусть
n035
35. Математическое ожидание выборочного среднего не зависит от объема выборки, если
n036
n037
X ∼ N (0; 1),
Xi
то
X 2 ∼ χ21 ?
удава в сантиметрах, а Yi - в дециметрах. Выборочный коэффициент
1
корреляции между этими наборами данных равен
?
10
- длина
i-го
Xi
одина-
ково распределены?
36. Зная закон распределения
распределения пары
X
и закон распределения
Y
можно восстановить совместный закон
(X, Y )?
37. Если ты отвечаешь на 10 данеток наугад, то число правильных ответов - случайная величина,
имеющая биномиальное распределение с дисперсией
4?
n038
n039
n040
n041
98
38.
Если
P (A) > 0
P (Ac ) > 0,
и
то
E(X) = P (A) · E(X|A) + P (Ac ) · E(X|Ac )?
39.
Если закон распределения величины
xi
0
1
Вероятность
0.5
0.5
, то
X
X
задан табличкой
- нормально распределена.
40.
Если события
A
и
B
независимы, события
B
A
и
B
несовместны, события
и
C
независимы, то события
A
и
C
независимы
41.
Если события
B
и
C
несовместны, то события
A
и
C
несовместны
42.
Если
P (A) > P (B)
то
P (A|B) > P (B|A)?
19. Unsorted
Задача 19.1. Гладиаторы
A и B . Каждый гладиатор
команде A всего NA гладиаторов с
На арене две команды гладиаторов,
неизменной по ходу игры. В
NB
гладиаторов с силами
B
{bi }N
i=1 .
обладает определенной силой,
NA
силами {ai }i=1 , в команде B -
Игра проходит в виде последовательных турниров, в каждом из
которых участвует по одному гладиатору от каждой стороны.
Если в очередном турнире встречаются гладиаторы с силами a и b , то вероятность победы первого
a
определяется величиной
. Гладиатор, проигравший турнир, выбывает из игры, выигравший a+b
возвращается в команду. Исходы турниров независимы. Это означает, что гладиаторы не устают,
но и не приобретают опыта. Стратегия команды предписывает, какого гладиатора выдвигать на
очередной турнир в зависимости текущего состава команды. Игра ведется до полного выбывания
из игры одной из команд.
а) Зависит ли вероятность победы команды
A
от используемой стратегии? Если Вы считаете, что
да, то укажите оптимальную стратегию; если нет, то докажите.
б) Допустим, что при выборе гладиатора для очередного турнира команда может учитывать не
только свой собственный текущий состав, но и состав команды-соперника. Соответственно изменяется и понятие стратегии. Будет ли зависеть вероятность победы команды от стратегии в этом
случае?
Задача 19.2. Парадокс гладиаторов-вампиров. [Winkler]
В отличие от обычного гладиатора, у победившего гладиатора-вампира сила увеличивается на силу
побежденного им гладиатора-вампира. В остальном правила поединка такие же, как в предыдущей
задаче. Зависит ли вероятность победы команды
A
от используемой стратегии?
Задача 19.3. Двумерное случайное блуждание
Выходя из начала координат 0, частица с равной вероятностью сдвигается на один шаг либо на
юг, либо на север, и одновременно (и тоже с равной вероятностью) на один шаг либо на восток,
либо на запад. После того как шаг сделан, движение продолжается аналогичным образом из нового
положения и так далее до бесконечности. Какова вероятность того, что частица когда-нибудь
вернется в начало координат?
Задача 19.4. Трехмерное случайное блуждание
Как и в предыдущей задаче, частица выходит из начала координат 0 в трехмерном пространстве.
Представим себе точку 0 как центр куба со стороною длины 2. За один шаг частица попадает в
один из восьми углов куба. Поэтому при каждом шаге частица с равной вероятностью сдвигается
99
на единицу длины вверх или вниз, на восток или на запад, на север или на юг. Какова доля частиц,
возвращающихся в начало, при неограниченном времени блуждания?
Задача 19.5.
В киосках продается ’открытка-подарок’. На открытке есть прямоугольник размером 2 на 7. В
каждом столбце в случайном порядке находятся очередная буква слова ’подарок’ и звездочка.
Например, вот так:
П
*
*
А
*
О
К
*
О
Д
*
Р
*
*
Прямоугольник закрыт защитным слоем, и покупатель не видит, где буква, а где - звездочка.
Следует стереть защитный слой в одном квадратике в каждом столбце. Можно попытаться угадать
n > 0 букв слова ’подарок’
50 · 2n−1 рублей. Если открыта
любое число букв. Если открыто
и не открыто ни одной звездочки, то
открытку можно обменять на
хотя бы одна звездочка, то открытка
остается просто открыткой.
а) Какой стратегии следует придерживаться покупателю, чтобы максимизировать ожидаемый
выигрыш?
б) Чему равен максимальный ожидаемый выигрыш?
Подсказка : Думайте!
Задача 19.6.
Задача, дороги из А в Б через С1, С2, С3
Вопросы про вероятности
Задача 19.7.
In the game of Racko you have 10 slots for cards to go in. At the start, cards are dealt to each player
and put in order that they were dealt starting from the «highest slot». The cards are numbered 1-60. The
object is to get all of your cards in order, by means of drawing and switching in a discard pile. When all
your cards are in order, you call «Racko» and the hand is over.
What is the probabilty that you will be dealt a «Racko»?
Source: aops
Задача 19.8.
X1 , X2 , . . . , Xn are independent
X1 + X2 + ...Xn < 1?
random variables, uniformly distributed on
[0, 1].
What is
the probability that
Solution:
We can compose a recurrence relation for the densities of sums in
pn (x) = xn /n! for x ⊂ [0, 1].
[0, 1]. A simple convilution formula gives
the result
Задача 19.9. You break a bar of length 1 unit into 2 pieces, choosing the break point uniformly along the
length. What’s the mean length of the smaller piece?
solution:
The largest piece is uniform on
More general problem. Let
using
(n − 1)
uniform
[0, 1]
Zn
[1/2, 1],
hence the mean is
3/4.
be the smallest piece arising from breaking up the unit stick into n pieces
random variables, say,
X1 , X2 , ..., Xn−1 .
We prove that
P (Zn > c) = (1 − cn)n−1 .
Ror any
0 < c < 1/n,
because then a simple integration by parts calculation shows that
E(Zn ) = 1/n2 .
To prove this equality, it’s enough to show that
P (X1 < X2 < · · · < Xn−1 , Zn > c) =
Because then I can multiply by
Xi ’s,
(n − 1)!, i.e.,
{Zn > c}.
to cover all possibilities for
(1 − cn)n−1
.
(n − 1)!
adding this last probability over all possible orderings of the
Now finally, how do I prove this last equality? Saying that
100
X1 < ... < Xn
and
Zn > c
is equivalent to saying that
X1 > c, X2 − X1 > c, X3 − X2 > c, . . . , Xn−1 − Xn−2 > c, 1 − Xn−1 > c.
which is equivalent to the following range for the
Xi ’s:
X1 ∈ (c, 1 − (n − 1)c)X2 ∈ (X1 + c, 1 − (n − 2)c) . . . Xk ∈ (Xk−1 + c, 1 − (n − k)c) . . . Xn−1 ∈ (Xn−2 + c, 1 − c)
So now, to calculate the probability of this event, we just have to integrate 1 over this region, i.e., we have
to calculate the integral
1−(n−1)c
Z
Z
1−(n−2)c
Z
1−c
···
c
x1 +c
1 dxn−1 dxn−2 · · · dx1
xn−2 +c
To work through the first (n-2) integrals, we note the following equality for any 2 <= k <= n-1, which is
simple enough to calculate:
Z
1−(n−k)c
xk−1 +c
(1 − (n − k)c − xk )n−(k+1)
(1 − (n − (k − 1))c − xk−1 )n−k
dxk =
(n − (k + 1))!
(n − k)!
Then using induction, the integral we want to calculate reduces to integrating
Z
1−(n−1)c
(1 − (n − 1)c − x1 )n−2
dx1
(n − 2)!
c
But if we think of
k = 1,
x0
as equalling
0,
then the integral equality above for
2≤k ≤n−1
also applies when
which gives us the answer we want.
The size of the kth largest piece is given by
1
n
1
1
+ ··· +
n
k
.
Задача 19.10.
Suppose I draw a ball one at a time with replacement from an urn with 100 balls and there’s a unique
label for each of the balls so I know which one has been drawn. Find the expected number of draws I have
to make in order to get all 100 balls. solution:
Recall that the problem concerns a prudent shopper who tries, in several attempts, to collect a complete
set of
N
different coupons. Each attempt provides the collector with a coupon randomly chosen from
N
known kinds, and there is an unlimited supply of coupons of each kind. It is easy to estimate the expected
waiting time to collect all
E{time
to collect
N
coupons:
N
coupons}
=1+
N
1
N
+
+ · · · + N = N (log N + γ + O( ))
N −1 N −2
N
Similarly,
E{time
to collect
N
coupons}
2
N
N
N
+
+ ··· + N
N −1 N −2
+1
2
1
N
1
= N (log N + γ + O( ) − log − γ − O( ))
N
2
N
= N log 2 + O(1)
=1+
Задача 19.11. You are waiting for a bus. They arrive
λ = 1/10 according to a Poisson process (so we have an average time of 10min between buses).
time between bus arrivals distributed as U [0, 20] (the average is also 10min).
a) with rate
b) with
If you take a random time what is your average wait for a bus?
Solution:
U [0, 20] distribution we have as follows. Consider the lengths of times between buses, X , this has
density pX (x) = 1/20 on [0, 20] and 0 outside [0, 20] interval. The probability density that you pick an
interval of length t is,
p{pick an interval of time x} = xp(x)/EX = t/100.
The mean picked interval is thus 40/3 (take the expectation of the above density). But, when you pick an
interval, the time you wait until the end of it is on average half of it’s length, which gives the answer 20/3.
a) For
101
b) We use the basic property of Poisson processes with intensity
arrivals is distribited as
Exp(λ).
λ = 1/10
that time interval between two
Using the "lost of memory"property of Exponential random variable
p{X > y + x|X > y} = p{X > x}.
we get that the average time till the next arrival is
10
min. Another funny consequence here is that even
after waiting 10 mins, the expected time is still unchanged: it’s 10 more minutes again.
Задача 19.12. Let
that
Yn
divides
Yn
13
be the sum of n rolls of a fair 6 faced die and 0 for n=0. Determine the probability
as n approaches infinity.
Решение 1. The answer is
p{Yn
divides
1/13.
We have
13} = 1/6 · (p{Yn−1 + 1
13} + p{Yn−1 + 2
divides
13} + · · · + p{Yn−1 + 6
divides
divides
13}).
The probability above is conditional to the first roll. Let
Un (0) = p{Yn
divides
13}, Un (1) = p{Yn +1
13}, . . . , Un (k) = p{Yn +k
divides
divides
13}
for
k = 0, . . . , 12.
We have
Un (0) = 1/6 · (Un−1 (1) + · · · + Un−1 (6)),
We also have
Un (0) + · · · + Un (12) = 1.
When
Un (1) = 1/6 · (Un−1 (2) + ... + Un−1 (7)),
n → ∞,
U (0) = 1/6 · (U (1) + ... + U (6)),
etc...
Un (k)
converge to
U (k),
we have:
U (0) + ... + U (12) = 1.
k = 0, . . . , 12.
Решение 2. Consider a 13 × 13 matrix, such that the (i, j) entry represents the following probability: given
that Yn is i mod 13, what is the probability that Yn+1 is jmod13? This matrix is "doubly stochastic"because
So
U (k) = 1/13
assuming that
etc...
for all
the sum of the entries in each row and each column equals 1. This means that the asymptotic distribution
of
Yn
is uniform. This requires some application of Markov chain theory.
Задача 19.13. Let
X1, X2, . . . , Xn
are independent
U (0, 1)
r.v.Let
Z
is the random variable that equals
k
for which the sum
Sk = (X1 + X2 + · · · + Xn )
exceeds
1
for the first time. Find
n
EZ.
solution:
1 is 1/n!. So the probability the sum
goes over 1 for the first time on the n-th random variable is 1/(n − 1)! − 1/n! = (n − 1)/n!. The expectation
is the sum of n ∗ (n − 1)/n! = 1/(n − 2)! for n = 2 to ∞, which equals the sum of 1/n! for n = 0 to infinity,
which is e.
Without much ado, here is a simpler and more general solution. Let f (x) be the expectation of number of
steps for exiting barrier x for the first time. The expectation that is in excess of 1 is
Z x
p(t)f (x − t) dt = f (x) − 1
The probability that
uniform random variables sum to less than
0
where
p(t)
(0, 1).
is the characteristic function for
with respect to
x
For
x ≤ 1,
we differentiate the above integral equation
and turn it into a simple differential equation. Solve it, we have
f (x) = ex , x ∈ [0, 1].
For
x > 1,
we have
x
Z
f (t) dt = f (x) − 1.
x−1
Differentiate and solve, we obtain the following recursive solution.
x
Z
x
f (x) = e f (n) −
e−t f (t − 1) dt , x ∈ (n, n + 1], n ∈ N.
n
Задача 19.14. you throw a fair coin until you get 8 heads in a row. What’s the probability you will see 8
consecutive tails (exactly) in the sequence prior to stopping?
answer=1/2
Задача 19.15. You have an unfair coin. Probability of tossing a head is p. What is the expected number
of tosses needed to get N consecutive heads?
102
Expected time to get N consecutive heads = Expected time to get (N-1) consecutive heads + p*1 +
(Expected time to get N consecutive heads + 1)(1-p)
letting xn be the obvious (expected
xn = xn−1 + p + (xn + 1)(1 − p)
so we get a recursion relation
since
x1 = 1/p,
time for n consecutive heads):
xn = (1/p)(xn−1 + 1)
we can find all the others.
This then seems to be the recursion relation for
xn =
P
(1/pi ),
where the sum is between i=1 and i=n
Задача 19.16. i offer to play a card game with you using a normal deck of 52 cards. the rules of the game
are that we will turn over two cards at a time. if the cards are both black, they go into my pile. if they are
both red, they go into your pile. if there is one red and one black, they go into the discard pile. we repeat
the two card flipping until we’ve gone through all 52 cards. whoever has more cards in their pile at the
end wins. i win if there is a tie. if you win, i pay you a dollar. how much would you pay to play this game?
Solution: tie is inevitable
Задача 19.17. A and B are to play a game. A third player, N constantly throws 2 dices. Player A wins if
N rolls ’12’. Player B wins if there are 2 consecutive 7 rolled by N.
The question reads: ’What is the probability that A wins’.
Задача 19.18. You randomly pick three numbers on (0;1). Let’s call them a,b,c. What is the probability
that there is a triangle which edges of lenghts a,b,c?
Related questions are:
a,b are two random numbers picked from (0,1) – non-correlated. What are the expected values of :
1. Min (a,b) (answer: 1/3)
2. Min(a+b,1) (answer: 5/6)
3. Min(a+b,1) - Abs(a-b) (answer: 1/2) ... This gives answer to the question below.
In this case, I think the probability of being able to form an n-simplex is 1-1/n! This is because we are
given n+1 "areas"for the "faces which gives us n+1 inequalities. Each inequality cuts out an n-simplex
from the unit cube of volume 1/(n+1)!. These n-simplices are nonintersecting, and there are n+1 of them,
so the the volume we have left is 1 - (n+1)/(n+1)! = 1 - 1/n!. Note that in 2 dimensions, this gives us the
correct answer, 1/2.
Задача 19.19. WC 20XX final game Brazil-Germany 8:6
Brazil scored the first goal. After that, the score follows a random sequence until it reaches 8:6. What is
the probability that Brazil has been at least 1 goal ahead for the whole duration of the game?
we can also use the reflective principle to do the counting.
let N be the difference of games brazil won and german won, the status of the game can be denoted as a
pair (t, N). given brazil won the first game, the number of total possible paths is C(13,6). the game starts
at (1,1), and ends at (14,2). we need to count all paths from (1,1) to (14,2) without touching N(t)=0.
consider the complement and use N=0 as the mirror, all paths ever reach N=0 can be viewed as starting
from (1,-1) and ending at (14,2), i.e., the number of such paths is C(13,5). so the probability is
1 - C(13,5) / C(13,7) = 1/4
I don’t know if this is legit but could you do it this way.
ordinarily, if you just know the results of two candidates the probability that the winner was always ahead
is (w-l)/(w+l) where w = number votes for the winner, and l= number votes for the loser.
So in this case we have (8-6)/(8+6) = 2/14
however we are given that brazil won the first game.
103
so if we divide 2/14 by the conditonal probability of brazil winning the first game we get (2/14)/(8/14)
= 1/4
What is the probability that the lottery draw will contain two or more consecutive integers?
Задача 19.20. Some components of a two-component system fail after receiving a shock. Shocks of three
types arrive independently and in accordance with Poisson processes. Shocks of the first type arrive at a
Poisson rate of ?1 and cause the first component to fail. Those of the second type arrive at a Poisson rate
?2 and cause the 2nd component to fail. The third type of shock arrives at a Poisson rate ?3 and causes
both components to fail. Let X1 and X2 denote the survival times for the two components.
Show that X1 and X2 both have exponential distributions?
One can attack this successfully from a high level, with only a few key observations:
1) For any dt > 0 and any t, the conditional probability that, for example, X1 > t + dt given that X1 >
t, is a function of dt alone, i.e., independent of t.
This should be clear without writing anything down.
2) Distributions satisfying this criterion can be shown to be exponential.
Задача 19.21. You’re throwing a fair dice and each time you add the outcome to the total (you start from
0). What is the probability that the path of this process will visit a number N? What happens when
n → ∞?
A sequence p(n) is defined using recurrent formula:
p(n) = 1/k*sum(over j=1, j=k)(p[n-j]) (1)
The values of p[1]...p[k] represent the initial conditions.
The sequence converges to some finite value at n->+inf for any initial conditions (easy).
k=6, p[1]=p[2]=...=p[5]=0, p[6]=1 is a special case for the probability to get cumulative sum "n"in a
process of throwing a dice.
We shall prove a general formula that
p(n) -> 2/(k+1) at n->inf
if p[1]=...p[k-1]=0, p[k]=1
Proof:
——
Let’s denote
Sm
to be the sequence (1) with initial conditions:
p[1]=...p[m-1]=0, p[m]=1, p[m+1]=...p[k]=0
In that notation, we are proving that
First, notice that
Sk − > 2/(k + 1)
S1 − > 1/k ∗ Sk
Now suppose we have proven that
Sm − > m/k ∗ Sk .
Consider a new sequence
S( m + 1) − Sm
(i.e.
every element is a difference of the corresponding elements). That new sequence has the following initial
conditions:
0,0,...-1,1,..0
p[1]=...p[m-1]=0, p[m]=-1, p[m+1]=1, p[m+2]=...p[k]=0
(S( m + 1) − Sm )− > 1/k ∗ Sk ,
S( m + 1)− > (m + 1)/k ∗ Sk and therefore
It is easy to see that
i.e.
104
Sm − > m/k ∗ Sk
for any m = 1,...k
Now consider a sequence
S1 + ... + Sk ,
i.e.
the one having initial conditions
p[1]=...p[k]=1
That sequence is stable and equal to 1 at every point.
At the same time, that one also converges
to
(1/k + 2/k + ...k/k) ∗ Sk = (k + 1)/2 ∗ Sk
We obtain:
(k + 1)/2 ∗ Sk − > 1
i.e.
Sk − > 2/(k + 1)
Q.E.D.
A special case k=6: p(n) -> 2/7 = 1/3.5
————————————
We have a call option on coin flips - the payoff of the "security"is the number of heads that comes up after
a number of flips. The strike price of the option is 2.
Need to value the option for 4 coin flips. (Interest rate is zero.)
Also, find the delta of this option.
You need one more thing to solve this problem, the price of bets on individual coin flips. If we assume
that we can bet $1 at even money, so we win $1 if a flip is heads and pay $1 if the flip is tails, then we
have the following values:
4 - 0 $2 3 - 1 $1 2 - 2 $0 1 - 3 $0 0 - 4 $0
3 - 0 $1.50 2 - 1 $0.50 1 - 2 $0 0 - 3 $0
2 - 0 $1 1 - 1 $0.25 0 - 2 $0
1 - 0 $0.625 0 - 1 $0.125
0 - 0 $0.375
The delta is $0.25, that’s how much you bet on the first flip to replicate the option
Yes we need to bet $0.25 on the first flip, but the delta of the option is 0.5. After the first flip, the
underlying will be worth $2.5 or $1.5, whereas the call will be worth $0.625 or $0.125. So Delta=0.5.
Your answer is better than mine. You have computed the delta with respect to the underlying, which is
the sum four coin flips. I was computing it with respect to a security that pays $1 for heads and -$1 for
tails. So you would buy half a coin flip, paying $0.25 to get $0.50 for a head and $0.00 for a tail. I would
buy one-quarter of a $1 coin flip bet, getting $0.25 for a head and paying $0.25 for a tail. It all comes out
to the same thing, but it shows you always have to be careful what your delta is computed with respect
to.
—————————————Markov property - стоит взглянуть (считается интеграл с броуновским движением)
Pool Puzzle - соотношение скоростей плавающего и ловящего в круглом бассейне
An expectation problem - складываем равномерные величины, до тех пор пока сумма не станет
больше 1
Series gamble - возможно неплохая задача
shrimp by the pound - возможно неплохая задача
7 color hats puzzle - решение задачи для одновременного раздавания шляп
Harmonic tossing - про сходимости по вероятности - хорошая задача
105
Life is full of error and round off.
Для стохана - поиграться с какой-нибудь функцией типа реализации броуновского движения (попробовать посчитать от нее обычный интеграл), построить график
Там есть пробабилити:
http://www.mathematik.uni-bielefeld.de/ sillke/
И досмотреть пункты 4, 6, 19 (8 возможно досмотреть) на cut-the-knot
http://www.mathpages.com/home/iprobabi.htm
Several old brainteaser (or math) questions – good to work for fun - стоит посмотреть
123 theorem and its extensions
aops:
88391
97893, 88977 - максимизация ожидаемого сохраненного броска
154385 - expected number of rounds
152421 - про экспоненциальное распределение с страховые случаи
38284 - первый шаг
http://home.att.net/ numericana/answer/weighing.htm
Общее решение задачи про взвешивания
Prisoner’s Dilemma by William Poundstone
про "случайные"с т.зр. человека последовательности:
http : //www.wilmott.com/messageview.cf m?catid = 26&threadid = 17247
В одной пачке 56 эмэндэмсин (50 грамм). Эмэндэмсины бывают 5 цветов (не считая синего).
Only math nerds would call
2500
finite (Leonid Levin)
topic "simple fun"(про игру в половину от среднего и эмпирические данные)
Включить в вопросы computation-intensive tasks
(это похоже на компьютерные экпсерименты, но не совсем: два пути - использование компьютера
для симуляции или для решения системы уравнений)
Какова вероятность наличия (хотя бы раз) 10 последовательных орлов при 2007 подбрасываниях
правильной монетки?
Задача 19.22.
>You have a black box with N balls in it - each of a different color. >Suppose you take turns as follows
- randomly pick a ball in each hand, >and paint the left hand ball the same color as the right hand ball.
>You replace both balls in the box before the next turn. > >How many turns do you expect before all
balls are the same color?
I worked this out as follows. I think this might be more or less along the lines of Robert Israel’s analysis,
which I didn’t actually follow, sad to say.
We just analyze one color, looking at the cases where it "wins". After any draw and replacement ( a turn,)
the urn is in a state i with i red balls. Of course, red (say) "wins"if i = N, and "loses"if i=0. We note that
the probablity of going from i -> i+1 ( i>0>N ) is always equal to the probability of i -> i-1, and this
value is i*(N-i)/(N-1)/N.
106
So, we can treat this as a random walk starting from 1 with absorbtion at i=0 and i=N. However,the
expected number of turns per step depends on the state i and is given by N*(N-1)/i/(N-i)/2 . The 2 is
because of the 2 equally probable transitions.
Now we just have to evaluate the average number of visits to each state 0<i<N, given that red wins. Since
each visit must terminate with a step, we multiply the visits by the number of turns per step for each
state to get the expected number of turns taken in each state, and sum over states.
I get that the probability of winning starting from state i is just i/N, and I get for the average number of
visits to a state, starting from i=1, ( win or lose ) 2*(N-i)/N.
Then the average number of "winning visits"to state i per trial is :
i/N * 2*(N-i)/N
and the average number of turns accounted for by these visits is:
i/N * 2*(N-i)/N * N*(N-1)/i/(N-i)/2 = (N-1)/N
but the expected number of wins is 1/N times the number of trials, so the expected number of turns per
winning trial in each state i is just N-1, and the expected number of turns in a win for red, or for any
other color, is just the number of nonterminal states times the expected number of turns in each state, or
(N − 1)2 .
Lew Mammel, Jr.
2
Yes, it is (N − 1) . Let Si be the event that the balls eventually end up all coloured i, and Ii the indicator
of this event (1 if it occurs and 0 if not). Let T be the number of steps until all balls are the same colour.
Let Ai be the initial number of balls of colour i (1 in the problem as given, but let it vary). Let V(k) =
E(I1 T | A1=k) (as far as the random variable I1 T is concerned, it doesn’t matter what Ai is for i <> 1).
We have V(0) = V(N) = 0. Note that E(I1 | A1=k) = k/N. By a "first-step analysis you can show that
P
2
V (k) = kV (1) − (N − 1) k−1
j=1 j/(N − k + j) for 1 <= k <= N For k = N this means V (1) = (N − 1) /N .
PN
2
But what we want is E(T |A1 = A2 = ... = AN = 1) =
i=1 E(IiT |Ai = 1) = N V (1) = (N − 1) .
Also interesting: if N is even and you start out with 2 balls of each of N/2 colours, the expected number
2
of steps is (N − 1) − N/2.
– Robert Israel
Задача 19.23. If W(t) is a standard Wiener process, the first passage time T(x) is defined as:
T(x) = inft: W(t) = x
|x|Exp[−x2 /(2t)]/Sqrt[2P it3 ]
understanding why E[T (x)] is Infinity for
T(x) has density function
I have a problem
all x > 0.
Solution:
If
E[T (x)]
were finite then according to Wald identities E[W(T(x))] were equal to 0, but it is not as
W(T(x)) = x.
Now let’s say that you don’t know the stochastic process followed by the stock price, but you are given
the price at time 0 of a call with expiration at time T as a function of the strike X (that is, given any
input strike from 0 to infinity, you can output the price of the call). How would you use this information
to determine the risk-neutral distribution of the stock price at time T?
Let f(S, T) be the terminal risk-neutral pdf for the stock - the 2nd deriv of the call price with respect to
strike is the discounted value of the risk-neutral pdf:
C”(K, T) = exp(-rT) f(K, T)
This is the well-known Breeden-Litzenberger result
I post here a usefull equation, for a GS interview for example.
Suppose you have a function f, defined on NxN, that verifies:
(1) f(n,k) = f(n-1,k) + f(n, k-1)
(2) together with some relevant initial condition.
–> Solve (1),(2), that is express f with some known functionals.
Hint: a. The easiest way to prove that is to know the answer (or have an intuition) as f is uniquely
determined by induction.
Solution: this equation for the number of path on a grid,
n
Cn+m
107
Задача 19.24. IBM:
Consider a loop of string of unit length. Suppose we cut the string independently and at random in n
places. This will divide the loop into n pieces.
This month’s puzzle asks
1. What is the expected (average) size of the smallest piece?
2. What is the expected (average) size of the largest piece?
Solution:
Identify the loop of string with real numbers from the unit interval in the obvious way. We may assume
without loss of generality that one of the cuts is at 0. Fix n. Let x be the expected size of the smallest
piece. Let f(t) be the probability that the smallest piece has size at least t. Note it is easy to see that
x equals the integral from 0 to 1 of f(t). We claim f(t)=(1-n*t)**(n-1) for t < 1/n, f(t)=0 otherwise.
Let t < 1/n. Then we assert configurations of n points on a unit loop of string (with one point at 0)
such that cutting at those points produces pieces of length at least t correspond to configurations of n
points on a loop of length (1-n*t) with one point at 0 and otherwise unrestricted. This follows because
the configurations map into each other by deleting (or adding) length t of string after each point. The
claim follows because the density of the configurations on the shorter loop is (1-n*t)**(n-1). We can now
compute x by evaluating the integral of (1-n*t)**(n-1) with respect to t from 0 to 1/n. Let s=(1-n*t).
Then the integral becomes (1/n)*(integral from 0 to 1 of s**(n-1) with respect to s) or 1/(n*n). So the
answer to the first part is 1/(n*n).
The answer to the second part can be derived by applying the above idea recursively and inductively.
Consider a set of n points on an unit loop (with one point at 0). Let x be the size of the smallest piece
if the loop is cut at those points. Consider deleting a piece of length x after each point. This will merge
two of the points leaving a set of (n-1) points on a loop of length (1-n*x) (still with one point at 0).
Since the expected size of x is 1/(n*n) the expected size of the smaller loop is (1-1/n). Let y be the size
of the smallest piece when the smaller loop is cut at the remaining points. Clearly the expected size of
y=(1-1/n)*(1/(n-1)*(n-1))=1/(n*(n-1)). Now the size of the second smallest piece in the original loop is
x+y which has expected size (1/n)*((1/n)+(1/(n-1)). Continuing in this way we find that the expected
size of the kth smallest piece is (1/n)*((1/n)+(1/(n-1))+ ... +(1/(n-k+1))). (Note the sum of the expected
sizes of all the pieces is 1 as expected.) So the expected size of the largest piece (ie the nth smallest piece)
is (1/n)(1+1/2+...+1/n). It is well known that the sum (1+1/2+...+1/n) behaves like ln(n) for large n so
asymptotically the largest piece has size ln(n)/n.
Задача 19.25. The Volume of a Simplex
The length of the unit interval is 1. The area of the triangle bounded by the x and y axes, and x+y ?
1, is 1/2. The volume bounded by the xy, xz, and yz planes, and x+y+z ? 1, is 1/6. Can we generalize this?
The volume of a hypersimplex is 1/n!. Proceed by induction on n. Assume an n dimensional simplex,
where all n variables are greater than 0, and their sum is less than 1. Let x be the variable of integration
in a nested integral. At the floor, when x = 0, we find a simplex in n-1 dimensions. Its volume is 1/(n-1)!.
As we move along the x axis, the sum of the remaining variables is restricted to 1-x, rather than 1. This
is a scaled version of the n-1 simplex. When all the variables are scaled by a factor of k, the volume is
multiplied by kn-1. Thus the volume of the simplex on the floor is multiplied by (1-x)n-1. The integrand
is therefore (1-x)n-1/(n-1)!. We can replace 1-x by x; that just reflects the shape through a mirror. Thus
the integral is xn/n!. Evaluate at 0 and 1 to get 1/n!.
The generalized octahedron in n dimensions consists of n variables such that the sum of their absolute
values never exceeds 1. This is actually a bunch of simplexes placed around the origin. In fact we need 2n
simplexes to make an octahedron. In 3 dimensions we place 8 simplexes around the origin, one for each
octant. Each simplex presents one face of the octahedron. The volume of the generalized octahedron is
2n/n!. This approaches 0 as n approaches infinity.
108
I just got a copy of Paul & Dominic’s Guide to Getting a Quant Job, which I hope everyone reads. I say
not not for your benefit in getting a job, but for my benefit in saving time and annoyance when looking
to fill a job. I’m posting here because it recommends this forum for practicing for brain teaser interview
questions. That’s an excellent idea. But if that’s what you’re here to do, you should also know how your
answer will be scored. So here’s my take on it, I invite other opinions and comments.
I ask three types of brain teaswer type questions.
(1) Really easy ones. This is the equivalent to moving the mouse around at random to see if the computer
is frozen. An example might be, "Suppose I flip a fair coin 10 times and pay you $1 if the first flip is heads,
$2 if the second flip is heads, up to $10 if the 10th flip is heads. What is your expected payout?"I assume
that anyone applying for a quant job knows how to sum the numbers from 1 to 10 (or has memorized the
answer) and can divide by 2. If not, I don’t want them anyway. I score the answers as follows:
10 - Listens carefully as the problem is posed, thinks for a minute, then answers $27.50 without
embellishment.
8 - Same as above, except asks for unnecessary clarification, appears suspicious of a trap, needs to use
pen and paper or feels it necessary to explain the reasoning.
6 - Same as 8 or 10, but gets a wrong answer between $20 and $35, or a clearly unreasonable wrong
answer but says it’s unreasonable.
4 - Same as 8 or 10, but gets a clearly unreasonable wrong answer and doesn’t mention it.
2 - Cannot understand the question, or pretends to misunderstand it to avoid giving a numerical answer.
0 - Throws out random phrases and numbers and watches my face hoping for some kind of hint, like
Clever Hans, the horse that could do math.
The point here is that good quants are confident of their ability to recognize and solve simple problems.
Most applicants are not good quants. You can do well on exams without this skill, because problems are
often set at an expected moderate level of difficulty, if it’s too easy or too hard you probably misunderstood
it. Real work is not like that. If you’re too scared to give a simple answer to a simple question, you’re not
going to work out.
(2) Hard ones, like many of the ones you will find here. Here the scoring is:
10 - Listens carefully as the problem is posed, thinks for a minute, then says anything intelligent that
indicates they understand why the problem is hard, and shows some confidence at being able to solve it.
Right or wrong doesn’t matter.
8 - Answers immediately and correctly (I assume in this case they’ve heard it before).
6 - Says, "I’ve heard it before,"and answers correctly
4 - Says, "I’ve heard it before,"and answers incorrectly, or in a way that
shows they don’t understand the answer, or can’t remember the answer
2 - Cannot understand the question, or pretends to misunderstand it to avoid giving an answer.
0 - Throws out random phrases and numbers and watches my face hoping for some kind of hint, like
Clever Hans, the horse that could do math.
There are people who expect to get the right answer, and are happy to be judged on their actual level of
intelligence. There are others who know in their hearts they will never get the right answer, and/or hope
to mislead you into thinking they are smarter than they are. Only the first group will get considered for
the job. I judge almost entirely on demeanor. Good people are eager to get these questions, bad people
dread them. Good people are proud when they get it right and honest but unembarassed when they get
it wrong. Bad people make excuses and complaints either way.
(3) Open-ended ones, like "Give me your personal subjective 50% confidence interval for the Gross
Domestic Product of Finland in USD."
109
10 - Without prompting comes up with a reasonable approach like, "I know it’s a small country, but it’s
not so tiny I’ve never heard of it, so I’ll guess 5 million population. It’s not known as particularly poor
or rich for Northern Europe, so I’ll guess $10,000 per capita income. That gives me a point estimate of
$50 billion. National income and GDP are close enough to each other for this approximation. For a 50%
confidence level I’d go 2 to 20 million population and $5,000 to $20,000 per capita income, that gives me
$10 billion to $400 billion. Considering everything, I think that’s too big a range, I’ll go with $25 billion to
$150 billion. If I had to bet, I’d be equally happy to bet the true number is inside as outside that range."
8 - Same as above but needs prompting and either uses some clearly unreasonable numbers or makes
numerous math errors.
6 - Thinks for a while and comes up with a reasonable range, but can’t discuss it intelligently.
4 - Cannot understand what a 50% confidence interval is.
2 - Keeps asking for more information, or explains how they would look up the answer.
0 - Throws out random phrases and numbers and watches my face hoping for some kind of hint, like
Clever Hans, the horse that could do math.
Here I’m looking for ability to understand and follow instructions, some small amount of practical sense
and knowledge of the world and ability to use quantitative reasoning. I retired this question when a
candidate outsmarted me. I’d used it for about two years and gotten scores from 0 to 10. Then someone
said "My interval is all dollar amounts that round to odd integer amounts."He turned a (3) question into
a (2) one, and got the job.
Задача 19.26. There are n points 1, 2, 3, ... n arranging in order on a circle. If the i th point is n, then the
i+1 th point is 1.
We choose a pair of adjacent points at random with equal probability of 1/(n-1). We continue to choose
pairs from the points on the circle at random. If one of the points of the pair has been chosen before,
we disregard this pair. The process is repeated until no new pair can be chosen and only isolated points
remain.
What is the mean number of isolated points?
Solution: may be a bad answer?
Задача 19.27. Two questions on Brownian Motion:
1) A standard Brownian motion z starting at positon a>0. There is an absorbing boundary at 0 - i.e.
after z first hits zero at time t0, z stays at zero for t>t0. What is the probability that z is at zero at any
arbitrary time T?
2) Two independent standard Brownian motions z1 and z2. z1starts at position a>0, z2 starts at position
0. z1 and z2 are "mutually absorbing i.e. when z1=z2 for the first time at t0, they both stay at z1(t0) for
t>t0. What is the probability that z1=z2 at any arbitrary time T?
Solution (may be wrong):
Let Ta = mint > 0 : Wt = a. Using simple properties of
2P (WT > a). That’s all you need to solve both problems.
1. 2N (−a/sqrt(T ))
2. 2N (−a/sqrt(2T ))
B.M. you can easily show that
P (Ta < T ) =
Задача 19.28. A bar in a town has 25 seats in a row. The folks in this town are antisocial, so they only
take a seat of which the adjacent seats are empty, or leave.
If you are the bartender, how can you seat your first guest to get more people (expected) at your bar?
Solution: (maybe computational)
Has anyone tried to calculate this maximum expectation? I see that seat num 3 and num 23 gives you the
highest expectation, however, the value I am getting is 11.38
Here is how I solve it.
Let E(n) be the expected number of antisocial people you can seat in n seats. For example,
110
E(0) = 0
E(1) = 1
E(2) = 1
E(3) = 1/3 * (1 + E(1)) + 1/3 * (1 + E(0)) + 1/3 * (1 + E(1)) = 1 + 2/3 * E(1)
E(4) = 1/4 * (1 + E(2)) + 1/4 * (1 + E(1)) + 1/4 * (1 + E(1)) + 1/4 * (1 + E(2)) = 1 + 2/4 * (E(1)
+ E(2))
E(5) = 1 + 2/5 * (E(1) + E(2) + E(3))
and E(n) = 1 + 2/n * (E(1) + E(2) + ... + E(n-2)) n = 2, 3, ...
The last expression can also be written as
E(n) = E(n-2) + 2/n * (1 + E(n-3)) n = 3, 4,
Now, let A(m) be the expected number of antisocial people you can seat in 25 seats provided that the
first person takes the mth seat. Then,
A(1) = 1 + E(23)
A(2) = 1 + E(22)
A(3) = E(1) + 1 + E(21)
A(4) = E(2) + 1 + E(20)
and so on
I am getting A(3) = 11.38
It is interesting that A(m) is oscillating.
Задача 19.29. Найти
P
(−1)k /Cnk
вопрос: где бы ее использовать?
answer:
n-odd
=> 0,
n
- even =>
2−
2
n+2
solution:
P
n!Sn = (−1)kP
· k!(n − k)!
(n + 1)!Sn+1 = (−1)k · k!(n + 1 − k)! = 0
or:
P
(n + 1)! = (−1)k · k!(n + 1 − k)!
(n + 2)n!Sn − (n + 1)! = ... = (n + 1)!
Можно решать через арифметику (задача про
f (a, b)
заменить биномиальный коэффициент на
интеграл)
Задача 19.30. Joric and social egalitarianism
Source: Romanian TST 5 2007, Problem 2
The world-renowned Marxist theorist Joric is obsessed with both mathematics and social egalitarianism.
Therefore, for any decimal representation of a positive integer
n,
he tries to partition its digits into two
groups, such that the difference between the sums of the digits in each group be as small as possible.
Joric calls this difference the defect of the number n. Determine the average value of the defect (over all
Pn
k=1 δ(k)
positive integers), that is, if we denote by δ(n) the defect of n, compute limn→∞
.
n
Solution:
δ(n) is always less than 10. If you partition the digits in two groups with
δ(n) and δ(n) ≥ 10 then move one of the digits from the group with bigger sum to the
1
other group. Hence δ(n) is bounded by 10. I claim that the average is . It can be seen that this number
2
1
is bigger than or equal to , because if the sum of the digits of n is odd then δ(n) ≥ 1, and the probability
2
It is easy to observe that
difference of sums
1
. One can also see that the probability of a number to have at least
2
10 digits of 1, is 1 (Really easy to show). Now the average of δ(n) over all numbers will be equal to the
of having an odd digital sum is
average over the special numbers I mentioned (The ones having at least ten digits of 1) For each of these
numbers like n, just remove the first ten digits of 1 to obtain
m.
Then we know that
δ(m) < 10.
Now
111
using the extra 1 digits we have at hand we can distribute them among the two groups we have for
to decrease the difference between sums to either 0 or 1. If the sum of digits of
m
δ(m)
is odd we can reduce
δ(m)
to 1 and in the other case to 0. One can again easily observe that the probability of m having an
1
. So for half of the numbers (with respect to probability) we have δ(n) = 0 and for
2
another half (disjoint from the last half ) we have δ(n) ≤ 1. So the average we are trying to find is less
1
and hence my claim is proved.
than or equal to
2
odd digital sum is
Задача 19.31. (wilmott) "вероятности"для натуральных чисел
n
Probability that 2 begins with 603245?
Prime factor problem
Задача 19.32. Оптимальный момент остановки для случая:
платеж = доле орлов
Sn /n
problem
—–
Wald had a number of important results, including the famous theorem that the expected value of the sum
of a random number of random variates is equal to the product of the expected value of a single variate
times the expected number in the sum; as long as the stopping rule is independent of the value of the sum
Like a lot of theorems, Wald’s Theorem is valuable when it doesn’t apply. That is, people often assume it
is true, it’s a handy trick for solving certain kinds of problems. Wald gave rigorous conditions under which
it is true. When you come across an application, it’s a good idea to check the three conditions.
Here is a simple example. In the mindless children’s card game "War"two players split a deck of card
between them. At each turn, both players reveal their top card, the player with the higher card takes both
and puts them at the bottom of her deck. If the cards are the same there is a "war,"meaning each player
deals three cards face down, then turns up the next card. The player with the higher card takes all ten
cards. In case of a tie there is another war, with the winner getting 18 cards. And so on until one player
runs out of cards.
Question: what is the expected number of concealed Aces (the high card) that will change hands in the
first play of the game? This is an important parameter for analyzing the game.
You could figure out all the possible combinations, but that would take a while. Wald’s theorem tells us
−2
we can compute the expected number of wars on the first play, 16 ∗ (17
+ 17−3 + 17−4 ), and multiply
by the expected number of concealed Aces from the losing player per war, 3/13. The answer is 0.0144. By
the way, you cannot have more than three wars. In the unlikely event (1/83,521) that the players tie four
times in a row, both of them lose the game and no cards are exchanged.
The three conditions of Wald’s theorem are:
(1) The number of trials is a non-negative integer with finite expectation.
(2) The outcome of the trials are i.i.d. with finite expectation.
(3) The outcome of the trial is independent of whether or not it is included in the total.
In the War example, condition 2 is violated. The probability that the first two cards match is 3/51 =
1/17. If they do, the probability that the next two compared cards match is 2/50 * 1/49 + 48/50 * 3/49
= 146/2,450 = 0.0596 instead of 0.0588.
—Version
#1
(tboafo): A die is rolled once. If the outcome is 1, 2, or 3, one stops; otherwise (i.e., if it is 4,
5, or 6) one rolls the die a second time. What is the total expected value? [A: 5.25]
Version
#2
(Wilbur): A die keeps being rolled until the outcome is 1, 2, or 3. What is the total expected
value? [A: 7]
Now, on to version
#3:
112
One has an arbitrarily large number of dice at our disposal (this is just a conceptual convenience; the
problem can easily be formulated with one single die). The first die is rolled. If the outcome is 1, 2, or 3,
one stops; otherwise, if it is 4, 5, or 6, a corresponding number of dice are rolled. This procedure continues
for every rolled dice whose outcome is 4, 5, or 6. Let n denote the n-th round of rolls. What is the total
expected value at the end of the n-th round of rolls?
Let’s clarify with a couple of examples:
Example I:
1st round of rolls (only one die is rolled): 3 => game ends.
Example II:
1st round of rolls (only one die is rolled): 4 => 4 dice will be rolled next 2nd round of rolls: 3, 5, 2, 6 =>
1st and 3rd dice end their lives, 2nd and 4th dice will lead to 5 and 6 dice to be rolled next, respectively
3rd round of rolls: 1, 2, 4, 4, 3; 5, 3, 6, 1, 6, 2 => ... and so forth ...
solution:
expected value for 1st round: 3.5
expected number of dice for 2nd round: (4+5+6)/6 = 2.5
expected value for 2nd round: 2.5 * 3.5
...
(
expected number of dice for n-th round: 2.5 n − 1)
(
expected value for n-th round: 2.5 n − 1) ∗ 3.5
n
total expected value at n-th round = (2.5 − 1)/(1.5)
∗ 3.5 = 7/3 ∗ (5/2n − 1)
—
Задача 19.33. In a large city the phone book comes in 4 volumes.
In a phone booth those for volumes are stacked one on top of the other.
Every t minutes someone takes out one volume from the stack, looks up a number and puts it back on
the top of the stack.
Suppose that the probability that any user should pick volume i is
Define the depth
di = 2
di
Pi .
(sum
of volume i as the distance from the top of the stack (di
Pi = 1).
= 1 if it is
on top of thestack,
if there is one volume on top of it and so on).
di .
a) Assume P1 = 0.4 P2 = 0.3 P3 = 0.2 P4 = 0.1
b) Find the expressions for di for general Pi
Find the long-term average depths
solut:
The key is that each pair of books is independent. Using the 0.4, 0.3, 0.2 and 0.1 probabilities: book 1
spends 3/7 of the time below book 2, 2/6 of the time below book 3 and 1/5 of the time below book 4. So
it has, on average, 202/210 books on top of it.
If the probabilities are p1, p2, . . ., pn, book i’s average depth will be the sum for j = 1 to n (not = i)
pj/(pi+pj).
Задача 19.34. If I start with nothing and I play this game whereby I throw a dice as many times as I
want. For each throw, if 1 appears I win $1, 2 appears I win $2 ....but if 6 appears I lose all my money.
So when is the optimal stopping time and what is the expected winning?
solution:
strategy - easy, Expectation - computational expensive (?)
Задача 19.35.
Let
X
and
Y
be independent uniform random variables between 0 and 1.
113
P (N, M ) := P (max(X, Y ) > M |min(x, y) < N )
P (N, M ) for M = 0 (trival case)
b) Find P (N, M ) for M = 1 (trival case)
c) Find P (N, M ) for N = M = 1/2
d) find N s.t. P (N, N ) = 1/2
e) find N and M = f (N ) s.t. P (N, M ) = 1/2
f ) find P (N, M )
Let
for any N,M in [0,1].
a) Find
Задача 19.36. In Bertrand’s paradox one is given a circle and asked: What is the probability P that a
chord chosen at random is longer than the side of an inscribed equilateral triangle.
The solution is highly dependent on how one interprets the phrase "chord chosen at random".
Solution 1: Assign a uniform probability distribution to the angles of intersection of the chord on the
circumference – then, clearly P = 1/3
Solution 2: Assign a uniform probability distribution to the center of the chord over the area of the circle
– then one can show that P = 1/4
Solution 3: Assign a uniform probability distribution to the linear distance between centers of the chord
and circle midpoint, then p = 1/2
Remarkably, in a 1973 paper The Well-Posed Problem E.T. Jaynes argued that this problem is well-posed
after all if one requires certain physical symmetries of the probability distribution: rotational invariance,
translational invariance and scale invariance. As Jaynes demonstrates, the 3rd solution is actually superior
on these physical grounds.
Задача 19.37. You stand by the bank of a straight river. You then walk 1 km straight in any direction
that keeps you dry, i.e. without crossing the river and stop at a point P.
a) What is the expected value of the distance to the river?
b) If at point P you walk 1 km in any direction, what is the probability that you will get back to the
river?
Comment: here random means uniform angle
sol:
I agree w/silverside on (a) 2/pi
for (b, given a random direction theta in [0, pi] that we walked to get to P, the probability of reaching the
river by walking in a random direction is Abs(pi - 2 theta ) / 2pi. Averaging from 0 to pi gives 1/4.
Задача 19.38. Выбираются три независимые равномерные на [0;1] величины. Найдите функцию
плотности средней величины, минимальной, максимальной.
Задача 19.39. Consider the ratio x/y of two positive reals x and y that are picked at random from [0;1].
What is the probability that the first non-zero digit in this ratio is a 4?
Задача 19.40. (досочинять) There is another example. The premier b-schools in India (IIMs) conduct
an admission test to select students. The test usually contains 200 multiple choice (four alternatives
per question) questions (1 mark each) with a 1/4 marks penalty for every incorrect answer. The test
duration is 2 hours. It is generally reagrded as a tough one and a score of more than 100 is considered decent.
Suppose a student randomly selects one of the alternatives. Regarding the test as a sequence of 200
independent trials we can figure out the probability of scoring more than 100 marks is of the order of
10( − 25). Fair enough. Suppose you have an "average"student, who gets 50 questions correct. He can
also narrow down the four alternatives of each question to two alternatives. Now he tosses a coin (or by
114
anyother random method ) to select the correct answer. To get a score of 100 or more, net of negative
marking, he needs to get atleast 70 of the remaining 150 questions correct. The probability, for this
case, turns out to be an astounding 81%! Even for a "below average"guy who gets 30 or less correct and
tosses a coin to get the rest of the questions, has a significant probability of scoring more than 100 (40-50%).
Now given score cards, what kind of conditional probability that a score of more than 100 came from a
below average guy (who tossed a coin for marking answers) can we expect? I guess it should be substantial.
I haven’t actually calculated it. But it should be substantial.
отсутствует доля средних и глупых
Задача 19.41. Assume two binary sequences of lenght N generated by fair coin flips (independent,
identically distributed with equal probability for H or T). What is the expected length of the longest
common contiguous subsequence that appears in both sequences?
E.g.:
S1: 000101111
S2: 101010101
longest common contiguous subsequence is: 0101
sol:
I ask that people who post problems make some effort to be precise. I assume "random"binary sequence
means each digit is 0 or 1 with probability 0.5, independent of all other digits. I assume subsequence
means contiguous subsequence as specified by the subsequent post.
A squence of length N has N - k + 1 subsequences of length k. The probability of two sequences of length
−k
k being identical, given the assumptions above, is 2
. So the expected number of identical subsequences
2
−k
of length k is (N − k + 1) ∗ 2
.
(N 2 ) ∗ (2−k ). Using that approximation, the expected number
2
(1−k)
or longer is (N ) ∗ 2
. This equals 1 if 2*ln(N)+(1 - k)*ln(2) =
For large N, this is approximately
of
identical subsequences of length k
0,
or k = 2*ln(N)/ln(2) + 1.
For large N and k near 2*ln(N)/ln(2) + 1, the probability of having an identical subsequence of length
k+1, given that you have have an identical subsequence of length k, is near 0.75. 1/(1 - 0.75) = 3.
This gives an approximation for the expected value of the longest subsequence to be 2*ln(N)/ln(2) + 4,
for large N.
For N = 3 there are 64 possible combinations. 2 of them, 000/111 and 111/000 have longest identical
subsequence equal to zero. 20 of them have longest identical subsequence equal to one. 34 of them have
longest identical subsequence equal to two. 8 of them, anything/the same thing, have longest identical
subsequence equal to three. The expected value is (2*0 + 20*1 + 34*2 + 8*3)/64 = 112/64 as Merlin81
said. That’s not close to the 7.17 asymptotic approximation for large N.
variation:
the first sequence is 00000...000
Задача 19.42.
Априори известно, что парная регрессия должна проходить через точку
(x0 , y0 ).
а) Выведите фор-
мулы МНК оценок; б) В предположениях теоремы Гаусса-Маркова найдите дисперсии и средние
оценок
Solution:
115
Вроде бы равносильно переносу начала координат и применению результата для регрессии без
свободного члена. Должна остаться несмещенность.
T:эконометрика
Задача 19.43.
Is there a probability distribution on
1
is ...
p
a) for all prime p?
b) for all
Z>0
such that the probability of selecting an integer divisible by
p
p?
Solution:
a) I believe the answer to the first question is yes. Let
pi
i-th prime. For convenience, set p0 = 1.
q0 = 1.) Assign the number qi a probability
be the
Let qi be the product of the first i primes. (In particular,
1
1
of
− pi+1
. Every other number is assigned a probability of zero. I believe this probability distribution
pi
satisfies your property.
b) I believe the answer to the second question is no. We can’t even make the condition work for all
squarefree numbers. Here is a sketch of the argument.
Suppose the condition did hold for all squarefree numbers. We can then easily show that the events
of being divisible by 2, 3, 5, etc., are mutually independent. In particular, the probability of being
nondivisible by 17, 19, 23, 29, 31, etc., is 0. (Here I am using the fact that the sum of inverse primes is
infinite.) It follows that the probability of being 1 through 16 is 0. By changing the 17 to an arbitrarily
large prime, we get that the probability of each integer is 0, which is a contradiction.
Here’s a remaining question. Can we make the condition hold for all primes and products of two distinct
primes?
source: aops, t=187407
Задача 19.44. A particle is bouncing randomly in a two-dimensional box. How far does it travel between
bounces, on average?
Suppose the particle is initially at some random position in the box and is traveling in a straight line in a
random direction and rebounds normally at the edges.
Solution:
r, the point has moved r · cos(θ)
horizontally and r · sin(θ) vertically, and thus has struck r · (sin(θ) + cos(θ)) + O(1) walls. Hence the
average distance between walls will be 1/(sin(θ) + cos(θ)). We now average this over all angles θ :
√
√
π/2
2/pi · intθ=0 (1/(sin(θ) + cos(θ)))dθ = 2 · 2 · ln(1 + 2)/π ≈ 0.79
Let
θ
be the angle of the point’s initial vector. After traveling a distance
source: probability puzzles, archive
==> probability/flips/once.in.run.p <==
What are the odds that a run of one H or T (i.e., THT or HTH) will occur in n flips of a fair coin?
==> probability/flips/once.in.run.s <==
References:
John P. Robinson, Transition Count and Syndrome are Uncorrelated, IEEE Transactions on Information
Theory, Jan 1988.
First we define a function or enumerator P(n,k) as the number of length "n"sequences that generate
"k"successes. For example,
P(4,1)= 4 (HHTH, HTHH, TTHT, and THTT are 4 possible length 4 sequences).
I derived two generating functions g(x) and h(x) in order to enumerate P(n,k), they are compactly
represented by the following matrix polynomial.
116
g(x)
h(x)
=
1 1
1 x
(n−3) 4
2 + 2x
The above is expressed as matrix generating function. It can be shown that
xk in the polynomial (g(x) + h(x)).
P (n, k)
is the coefficient of
the
n = 4 we get (g(x) + h(x)) from the matrix generating function as (10 + 4x + 2x2 ). Clearly,
(coefficient of x) is 4 and P (4, 2) = 2 ( There are two such sequences THTH, and HTHT).
For example, if
P(4,1)
We can show that
mean(k) = (n-2)/ 4 and
sd =
√
5n − 12/4
We need to generate "n"samples. This can be done by using sequences of length (n+2). Then our new
statistics would be
mean = n/4
sd =
√
5n − 2/4
Similar approach can be followed for higher dimensional cases.
==> probability/lights.p <==
Waldo and Basil are exactly m blocks west and n blocks north from Central Park, and always go
with the green light until they run out of options. Assuming that the probability of the light being
green is 1/2 in each direction, that if the light is green in one direction it is red in the other, and that
the lights are not synchronized, find the expected number of red lights that Waldo and Basil will encounter.
==> probability/lights.s <==
Let E(m,n) be this number, and let (x)C(y) = x!/(y! (x-y)!). A model for this problem is the following
nxm grid:
where each + represents a traffic light. We can consider each traffic light to be a direction pointer, with
an equal chance of pointing either east or south.
IMHO, the best way to approach this problem is to ask: what is the probability that edge-light (x,y) will
be the first red edge-light that the pedestrian encounters? This is easy to answer; since the only way to
reach (x,y) is by going south x times and east y times, in any order, we see that there are (x+y)C(x)
(
possible paths from (0,0) to (x,y). Since each of these has probability (1/2) x + y + 1) of occuring, we
(
see that the the probability we are looking for is (1/2) x + y + 1) ∗ (x + y)C(x). Multiplying this by the
expected number of red lights that will be encountered from that point, (n-k+1)/2, we see that
Pn−1
P
(
(
E(m, n) = m−1
k=0 (1/2) m + k + 1) ∗ (m + k)C(m) ∗
k=0 (1/2) n + k + 1) ∗ (n + k)C(n) ∗ (m − k + 1)/2 +
(n − k + 1)/2
Are we done? No! Putting on our Captain Clever Cap, we define
P
(1/2)k ∗ (m + k)C(m) ∗ k
f (m, n) = n−1
k=0
Pn−1
k
and g(m, n) =
k=0 (1/2) ∗ (m + k)C(m).
Now, we know that
P
f (m, n)/2 = nk=1 (1/2)k ∗ (m + k − 1)C(m) ∗ (k − 1)
and since f(m,n)/2 = f(m,n) - f(m,n)/2, we get that
P
k
f (m, n)/2 = n−1
k=0 (1/2) ∗ ((m + k)C(m) ∗ k − (m + k
n
− 1)C(m) ∗ (k − 1))
−(1/2) ∗ (m + n − 1)C(m) ∗ (n − 1)
P
(
= n−2
k=0 (1/2) k + 1) ∗ (m + k)C(m) ∗ (m + 1)
−(1/2)n ∗ (m + n − 1)C(m) ∗ (n − 1)
= (m + 1)/2 ∗ (g(m, n) − (1/2)( n − 1) ∗ (m + n − 1)C(m)) − (1/2)n ∗ (m + n − 1)C(m) ∗ (n − 1)
therefore
117
f (m, n) = (m + 1) ∗ g(m, n) − (n + m) ∗ (1/2)( n − 1) ∗ (m + n − 1)C(m) .
(
(
(
Now, E(m, n) = (n + 1) ∗ (1/2) m + 2) ∗ g(m, n) − (1/2) m + 2) ∗ f (m, n) + (m + 1) ∗ (1/2) n + 2) ∗
(
g(n, m) − (1/2) n + 2) ∗ f (n, m)
= (m+n)∗(1/2)( n+m+1)∗(m+n)C(m)+(m−n)∗(1/2)( n+2)∗g(n, m)+(n−m)∗(1/2)( m+2)∗g(m, n).
Setting m=n in this formula, we see that
E(n, n) = n ∗ (1/2)( 2n) ∗ (2n)C(n),
and applying Stirling’s theorem we get the beautiful asymptotic formula
E(n, n) sqrt(n/pi).
==> probability/random.walk.p <== Waldo has lost his car keys! He’s not using a very efficient search; in
fact, he’s doing a random walk. He starts at 0, and moves 1 unit to the left or right, with equal probability.
On the next step, he moves 2 units to the left or right, again with equal probability. For subsequent turns
he follows the pattern 1, 2, 1, etc.
His keys, in truth, were right under his nose at point 0. Assuming that he’ll spot them the next time he
sees them, what is the probability that poor Waldo will eventually return to 0?
==> probability/random.walk.s <== I can show the probability that Waldo returns to 0 is 1. Waldo’s
wanderings map to an integer grid in the plane as follows. Let
(Xt , Yt )
be the cumulative sums of the
length 1 and length 2 steps respectively taken by Waldo through time t. By looking only at even t, we
get the ordinary random walk in the plane, which returns to the origin (0,0) with probability 1. In fact,
landing at (2n, n) for any n will land Waldo on top of his keys too. There’s no need to look at odd t.
Similar considerations apply for step sizes of arbitrary (fixed) size.
==> probability/transitivity.p <== Can you number dice so that die A beats die B beats die C beats
die A? What is the largest probability p with which each event can occur?
==> probability/transitivity.s <== Yes. The actual values on the dice faces don’t matter, only their
ordering. WLOG we may assume that no two faces of the same or different dice are equal. We can assume
"generalised dice where the faces need not be equally probable. These can be approximated by dice with
equi-probable faces by having enough faces and marking some of them the same.
Take the case of three dice, called A, B, and C. Picture the different values on the faces of the A die.
Suppose there are three:
A A A
The values on the B die must lie in between those of the A die:
B A B A B A B
With three different A values, we need only four different B values.
Similarly, the C values must lie in between these:
C B C A C B C A C B C A C B C
Assume we want A to beat B, B to beat C, and C to beat A. Then the above scheme for the ordering of
values can be simplified to:
B C A B C A B C A B C
since for example, the first C in the previous arrangement can be moved to the second with the effect
that the probability that B beats C is increased, and the probabilities that C beats A or A beats B are
unchanged. Similarly for the other omitted faces.
In general we obtain for n dice A...Z the arrangement
B ... Z A B ... Z ...... A B ... Z
where there are k complete cycles of B..ZA followed by B...Z. k must be at least 1.
CONJECTURE: The optimum can be obtained for k=1.
So the arrangement of face values is B ... Z A B ... Z. For three dice it is BCABC. Thus one die has just
one face, all the other dice have two (with in general different probabilities).
CONJECTURE: At the optimum, the probabilities that each die beats the next can be equal.
Now put probabilities into the BCABC arrangement:
B C A B C x y 1 x’ y’
Clearly x+x’ = y+y’ = 1.
Prob. that A beats B = x’ B beats C = x + x’y’ C beats A = y
118
Therefore x’ = y = x + x’y’
Solving for these gives x = y’ = 1-y, x’ = y = (-1 + sqrt(5))/2 = prob. of each die beating the next =
0.618...
For four dice one obtains the probabilities:
B C D A B C D x y z 1 x’ y’ z’
A beats B: x’ B beats C: x + x’y’ C beats D: y + y’z’ D beats A: z
CONJECTURE: for any number of dice, at the optimum, the sequence of probabilities abc...z1a’b’c...z’ is
palindromic.
We thus have the equalities:
x+x’ = 1 y+y’ = 1 z+z’ = 1 x’ = z = x + x’y’ = x + x’y’ y = y’ (hence both = 1/2)
Solving this gives x = 1/3, z = 2/3 = prob. of each die beating the next. Since all the numbers are rational,
the limit is attainable with finitely many equiprobable faces. E.g. A has one face, marked 0. C has two
faces, marked 2 and -2. B has three faces, marked 3, -1, -1. D has three faces, marked 1, 1, -3. Or all four
dice can be given six faces, marked with numbers in the range 0 to 6.
Finding the solution for 5, 6, or n dice is left as an exercise.
Source: Richard Kennaway, jrksys.uea.ac.uk
Martin Gardner (of course!) wrote about notransitive dice, see the Oct ’74 issue of Scientific American, or
his book "Wheels, Life and Other Mathematical Amusements ISBN 0-7167-1588-0 or ISBN 0-7167-1589-9
(paperback).
In the book, Gardner cites Bradley Efron of Stanford U. as stating that the maximum number for three dice
is approx .618, requiring dice with more than six sides. He also mentions that .75 is the limit approached
as the number of dice increases. The book shows three sets of 6-sided dice, where each set has 2/3 as the
advantage probability.
20. Выборочное среднее и общая интуиция
Задача 20.1. Среднее и медиана.
Имеется пять чисел:
x, 4, 5, 7, 9.
При каком значении
x
медиана будет равна среднему?
Задача 20.2.
Измерен рост 100 человек. Средний рост оказался равным 160 см. Медиана оказалась равной 155
см. Машин рост в 163 см был ошибочно внесен как 173 см. Как изменятся медиана и среднее после
исправления ошибки?
Задача 20.3.
This is a famous WWII story about the statistician Abraham Wald. He was asked by the Air Force
to determine where to reinforce the armor on bombers. If you put armor everywhere, the plane is too
heavy to take off. But the Air Force maintained detailed records of the location of every hit on every
plane returning from missions in Germany. Wald looked at the tabulations and performed a simple
transformation before using the distribution to place the armor. What transformation?
Wald inverted the distribution. The places where there were no holes were not places the anti-aircraft
guns always missed, they were places where a hit was fatal. The places with lots of holes were places
where hits didn’t matter much.
Задача 20.4.
The New York Times’ weekly science supplement called «Science Times» on August 22, 1989 stated:
«... From June 4 through November 4, 1984, for instance, 132 such victims were admitted to the Animal
Medical Center.... Most of the cats landed on concrete. Most survived...
... [Veterinarians] recorded the distance of the fall for 129 of the 132 cats. The falls ranged from 2 to 32
stories... 17 of the cats were put to sleep by their owners, in most cases not because of life-threatening
injuries but because the owners said they could not afford medical treatment. Of the remaining 115, 8
died from shock and chest injuries...
... Even more surprising, the longer the fall, the greater the chance of survival. Only one of 22 cats that
119
plunged from above 7 stories died, and there was only one fracture among the 13 that fell more than
9 stories. The cat that fell 32 stories on concrete, Sabrina, suffered [only] a mild lung puncture and a
chipped tooth...»
Is it true, that there is positive relation between length of the fall and chance of survival?
Задача 20.5.
Возможно ли, что риск катастрофы в расчете на 1 час пути больше для самолета, чем для автомобиля, а в расчете на 1 километр пути - наоборот?
Задача 20.6.
Деканат утверждает, что если студента N перевести из группы А в группу В, то средний рейтинг
каждой группы возрастет. Возможно ли это?
Задача 20.7.
Из класса А в класс Б перевели группу человек, затем из класса Б в класс В перевели группу
человек. После этой операции рейтинг каждого класса возрос по сравнению с первоначальным.
Затем другие группы переводили в обратном направлении (из В в Б, потом из Б в А). При этом
рейтинг каждого класса снова вырос. Возможно ли это?
Задача 20.8.
Два лекарства испытывали на мужчинах и женщинах. Каждый человек принимал только одно
лекарство. Общий процент людей, почувствовавших улучшение, больше среди принимавших лекарство А. Процент мужчин, почувствовавших улучшение, больше среди принимавших лекарство В.
Процент женщин, почувствовавших улучшение, больше среди принимавших лекарство В. Возможно
ли это?
Задача 20.9.
Пусть
X ∈ L1
F (t)
F (c0 ).
и ее функция распределения
минимума при некотором
c0 .
a) Найдите
непрерывна. Величина
E|X − c|
достигает своего
b) Проинтерпретируйте
hint: удобно воспользоваться геометрической интерпретацией мат. ожидания
Задача 20.10.
Расположите по порядку: среднее, мода, медиана
Задача 20.11.
На курсах 3 группы по 10 человек, 2 группы по 20 человек и 1 группа по 40 человек.
а) Каков средний размер группы, для которой читает лекции наугад выбранный профессор?
б) Каков средний размер группы, в которой учится наугад выбранный студент?
в) На других курсах Вы опросили
n
человек и спросили у каждого размер группы, постройте
несмещенную оценку для среднего размера группы.
Задача 20.12. [Mosteller] Странное метро (шутка)
Мэрвин кончает работу в случайное время между 15 и 17 часами. Его мать и его невеста живут
в противоположных частях города. Мэрвин садится в первый подошедший к платформе поезд,
120
идущий в любом направлении, и обедает с той из дам, к которой приедет. Мать Мэрвина жалуется
на то, что он редко у нее бывает, но юноша утверждает, что его шансы обедать с ней и с невестой
равны. Мэрвин обедал с матерью дважды в течение 20 рабочих дней.
Объясните это явление.
Задача 20.13.
Два эскалатора находятся рядом так, что человек может перешагнуть с одного на другой без
потери и набора высоты. Можно ли сделать так, чтобы: коробка, стоящая на левом эскалаторе,
спускалась бы вниз; коробка, стоящая на правом эскалаторе в среднем спускалась бы вниз; человек,
переходящий с одного эскалатора на другой без изменения высоты, в среднем поднимался бы вверх?
Движение эскалаторов может быть не равномерным.
21. Несмещенность и прочие свойства оценок
Задача 21.1.
Пусть
X
равномерна на
[0; a].
Придумайте
Y = α + βX
так, чтобы
Y
была несмещенной оценкой
a.
Задача 21.2.
Xi - независимы и одинаково распределены. При каком значении
2X1 − 5X2 + βX3 будет несмещенной оценкой для E (Xi )?
β (X1 + X2 − 2X3 )2 будет несмещенной оценкой для V ar (Xi )?
Пусть
а)
б)
параметра
β
Задача 21.3.
Пусть
X1
X2
и
параметра
a
независимы и равномерны на
[0; a].
При каком
β
Y = β · min {X1 , X2 }
оценка
для
будет несмещенной?
Задача 21.4.
Пусть случайная величина
хочет оценить параметр
X
распределена равномерно на отрезке
θ = P (X < 3).
[0; a],
Рассмотрим следующую оценку
a > 3. Исследователь
1, X < 3
θ̂ =
.
0, X ≥ 3
где
а) Верно ли, что оценка θ̂ является несмещенной?
2 θ̂ − θ
б) Найдите E
.
Задача 21.5.
Пусть
a
X
равномерна на
[3a − 2; 3a + 7]. При каких α и β
оценка
Y = α + βX
неизвестного параметра
будет несмещенной?
Задача 21.6.
X имеет вид
xi
0
1
a
xi
0 1
2
а)
; б)
P (X = xi ) 1/4 1/4 2/4
P (X = xi ) 1/4 a (3/4 − a)
Постройте несмещенную оценку вида Y = α + βX для неизвестного
Закон распределения с.в.
параметра
a
Задача 21.7. [т]
Время горения лампочки – экспоненциальная с.в. с ожиданием равным
менно 20 лампочек. С.в.
а) Найдите
X
θ.
Вася включил одновре-
обозначает время самого первого перегорания.
E(X)
б) Как с помощью
X
построить несмещенную оценку для
θ?
Задача 21.8.
Xi ∼ iid,
какая из приведенных оценок для
наиболее эффективной среди несмещенных?
E (Xi )
является несмещенной? наиболее эффективной?
121
а)
X1
; б)
X1 + 3X2 − 2X3
; в)
(X1 + X2 ) /2 ; г) (X1 + X2 + X3 ) /3 ; д) (X1 + ... + X20 ) /21 ; е) X1 − 2X2
Задача 21.9. (3 золотых слитка - задачу можно клонировать!!!)
2
Весы имеют ошибку со средним ноль и дисперсией σ .
Имеется три золотых монеты и результаты следующих 7 взвешиваний:
Каждую монету взвешивали по отдельности, все три монеты вместе, монеты взвешивали парами (1
и 2; 1 и 3; 2 и 3).
Вася предлагает сложить .... вычесть.
Будет ли оценка веса первого слитка смещенной?
Будет ли она лучше чем усреднить семь взвешиваний по 1-му слитку
Задача 21.10. Есть два золотых слитка, разных по весу. Сначала взвесили первый слиток и получили
результат
X.
Затем взвесили второй слиток и получили результат
и получили результат
Y.
Затем взвесили оба слитка
Z.
Допустим, что ошибка каждого взвешивания - это случайная величина с
2
нулевым средним и дисперсией σ .
а) Придумайте наилучшую оценку веса первого слитка.
б) Сравните придуманную Вами оценку с оценкой, получаемой путем усреднения двух взвешиваний
первого слитка.
Solution:
a) Пусть истинные веса слитков равны
x, y
и
z.
Назовем оценку буквой x̂
x̂ = aX + bY + cZ
Несмещенность: E(x̂) = aE(X) + bE(Y ) + cE(Z) = ax + by + c(x + y) = x
a + c = 1, b + c = 0
x̂ = (1 − c)X + (−c)Y + cZ
Эффективность:
V ar(x̂) = ((1 − c)2 + c2 + c2 ) · σ 2 = (3c2 − 2c + 1)σ 2
Чтобы минимизировать дисперсию нужно выбрать c =
2
1
1
Т.е. x̂ = X − Y + Z
3
3
3
2 2
б) V ar(x̂) = σ
3
1 2
2
V ar X1 +X
= 2σ
2
Усреднение двух взвешиваний первого слитка лучше.
1/3
Задача 21.11. Задача по эконометрике?
Given a weighing device and two balls. Weigh the first ball using the weighing device, we get a value
X; weigh the second ball using the weighing device, we get a value Y; weigh the two balls together, we
get a value Z. Suppose that each time the error in the weights is an iid random variable, for example,
X = a + Ra , Y = b + Rb , Z = a + b + Rc ,
where
Ra , Rb , Rc
are iid. How do you get the best estimation of
the first ball’s the real weight (i.e., a). We don’t know the distribution of the R’s.
best - unbiased, min var
Задача 21.12.
Xi ∼ iid,
а)
X12 −
какая из приведенных оценок для V ar (Xi ) является несмещенной
Pn
Pn
2
2
(Xi −X̄ )
i=1 (Xi −X̄ )
1
X1 X2 ; б) i=1 n
; в)
; г)
(X1 − X2 )2 ; д) X1 − 2X2 ; е)
n−1
2
X1 X2
Задача 21.13.
Случайные величины
Xi
распределены равномерно на отрезке
θ = P (X1i <5) .
ватель хочет оценить параметр
а) Используя
X̄n
постройте несмещенную оценку
θ̂
б) Найдите дисперсию построенной оценки
в) Является ли построенная оценка состоятельной?
для
θ
[0; a],
известно, что
a > 10.
Исследо-
122
Задача 21.14.
X1 ,
Пусть
...
X6
независимы и равномерно распределены на
[0; k],
где
k ∈ [1; 2]
- неизвестный
параметр.
а) Постройте несмещенную оценку для
б) Постройте несмещенную оценку для
в) Постройте несмещенную оценку для
k
k
k
вида
вида
вида
k̂ = c · X̄
k̂ = c · min{X1 , ..., X6 }
k̂ = c · max{X1 , ..., X6 }
г) Какая из них является наиболее эффективной?
Задача 21.15.
Пусть
Xi
независимы, одинаково распределены с функцией плотности
По имеющейся выборке
X1 , X2 ,
...
p(t) =
2t/k 2 , t ∈ [0; k]
0, иначе
Xn :
а) Оцените медиану этого распределение методом максимального правдоподобия
б) Оцените медиану этого распределения методом моментов, используя
E(Xi )
Solution:
Медиана равна
m=
√k
2
max{X1 ,...,Xn }
√
2
Source: Suhov, 5.20 (i)
mM L =
Задача 21.16.
Let
Xi (
- iid
with pdf given by:
e−t/λ
,t ≥ 0
µ+λ
p(t) =
et/µ
,t < 0
µ+λ
You are given a sample of X1 , ...
a) Find ML estimators for
Xn
µ and λ
Hint: You may found useful the following notation:
S + - sum of all positive Xi
S − - sum of all negative Xi
b) Let
n = 1,
is the estimator of
λ
is unbiased?
Solution: √
S + + −S + S −
a) λ̂ =
√ n
−
+ −
µ̂ = −S + n−S S
b) only if µ = 0
Source: Suhov, 5.27 (ii)
Задача 21.17.
X 1 , X2 , X 3 ek−t , t ≥ k
p(t) =
0, t < k
Пусть
независимы, одинаково распределены с плотностью
Являются ли следующие оценки для
k
несмещенными?
Какая из оценок будет наиболее эффективной?
k̂ = X3 − 1
b) k̂ = min{X1 , X2 , X3 } −
X̄
c) k̂ =
−1
3
а)
1
3
Solution:
Все несмещенные, b - наиболее эффективная
Source: Suhov, 5.31
Задача 21.18.
Xi - iid, U [−b; b].
b̂ = c · (|X1 | + |X2 |).
Пусть
Имеется выборка из 2-х наблюдений. Вася строит оценку для
b
по формуле
123
a) При каком
б) При каком
c
c
оценка будет несмещенной?
M SE = E((b̂ − b)2 )?
Pn
формуле b̂ = c
i=1 |Xi |
оценка будет минимизировать средне-квадратичную ошибку,
в) Решите задачу, если имеется
n
наблюдений, и оценка строится по
Source: экзамен миэф, 2008, Пересецкий
Задача 21.19.
Пусть величины X1 , ..., Xn равномерны на
n
k̂1 = X̄ − 21 и k̂2 = max{Xi } − n+1
а) Являются ли оценки несмещенными?
[k; k + 1]
и независимы. Есть две оценки параметра
k:
б) У какой оценки ниже дисперсия (при большой выборке)?
в) Являются ли оценки состоятельными?
Answers:
a) обе несмещенные
1
n
б) V ar(k̂1 ) =
max uniform12n < n+2
see 14.36
−
2
n
n+1
= V ar(k̂2 )
в) да, обе
Задача 21.20.
Пусть
Xi
p(t) = ea−t при t > a, где a
a используется ân = min{X1 , X2 , ..., Xn }.
- независимы и имеют функцию плотности
параметр. В качестве оценки неизвестного
- неизвестный
а) Является ли предлагаемая оценка состоятельной?
б) Является ли предлагаемая оценка несмещенной?
Solution:
ân ≥ a.
P (|ân − a| > ε) = P (ân − a > ε) = P (ân > a + ε) = P (min{X1 , X2 , ..., Xn }> a + ε) =
n
R∞
n
= P (X1 > a + ε ∩ X2 > a + ε ∩ ...) = P (X1 > a + ε) · P (X2 > a + ε) · ... = a+ε ea−t dt = (e−ε ) = e−nε
limn→∞ e−nε = 0
б) нет, не является ни при каких n, хотя смещение с ростом n убывает
Заметим, что
Задача 21.21.
Пусть
Xi
[a − 1; a], где a
ân = max{X1 , X2 , ..., Xn }.
- независимы и распределены равномерно на
качестве оценки неизвестного
a
используется
- неизвестный параметр. В
а) Является ли предлагаемая оценка состоятельной?
б) Является ли предлагаемая оценка несмещенной?
Solution:
ân ≤ a.
P (|ân − a| > ε) = P (−(ân − a) > ε) = P (ân < a − ε) = P (max{X1 , X2 , ..., Xn } < a − ε) =
= P (X1 < a − ε ∩ X2 < a − ε ∩ ...) = P (X1 < a − ε) · P (X2 < a − ε) · ... = (1 − ε)n
limn→∞ (1 − ε)n = 0
б) нет, не является ни при каких n, хотя смещение с ростом n убывает
Заметим, что
22. MM, ML and BA
Задача 22.1. simple MM, ML и Bayesian
Допустим, что закон распределения
X
-1
0
Xn
имеет вид:
2
2θ − 0.2 1.2 − 3θ
X1 = 0, X2 = 2.
a) Найдите оценки θ̂M L и θ̂M M
b) Первоначально ничего о θ не было известно и поэтому предполагалось,
равномерно на [0.1; 0.4]. Как выглядит условное распределение θ ?
Prob
θ
Имеется выборка:
что
θ
распределена
124
Задача 22.2.
Xi распределены одинаково и независимо. С помощью ML оцените значение неизвестного параметра θ , если функция плотности Xi имеет вид:
2
−θ
θ(lnθ−1 y )
2y
θe 2y
θ−1
а) θy
при y ∈ [0; 1] ; б) 2 при y ∈ [0; θ] ; в) √
при y ∈ [0; +∞) ; г)
при y ∈ [1; e]
θ
y
3
Известно, что
2πy
e−|y|
при
д)
2(1−e−θ )
y ∈ [−θ; θ].
В пунктах а), б) и д) постройте также ММ оценку.
Задача 22.3.
Постройте MM и ML оценки параметра
λ
экспоненциального распределения.
µ
нормального распределения при известном
Задача 22.4.
Постройте MM и ML оценки параметра
σ2.
Задача 22.5.
Постройте MM и ML оценки параметра
σ2
нормального распределения при известном
µ.
Задача 22.6.
X
n независимы
1 , X2 ,..., X
(k + 1)xk , x ∈ [0; 1];
f (x) =
0,
x∈
/ [0; 1].
Найдите оценки параметра k :
Пусть
и их функции плотности имеет вид:
а) Методом максимального правдоподобия
б) Методом моментов
Задача 22.7.
X1 , X2 ,..., Xn независимы и равномерно распределены
Постройте θ̂ методом максимального правдоподобия
Постройте θ̂ методом моментов
Пусть
а)
б)
на отрезке
[0; θ], θ > 1
в) Как изменятся ответы на «a» и «б», если исследователь не знает значений самих
только количество
Xi
Xi ,
а знает
оказавшихся больше единицы?
Задача 22.8. Про зайцев
В темно-синем лесу, где трепещут осины, отловлено
100
зайцев. Каждому из них на левое ухо
завязали бант из красной ленточки и отпустили. Через неделю будет снова отловлено
Из них
N
(св.) окажутся с бантами. Найдите
E(N ),
если всего в лесу
z
100
зайцев.
зайцев.
а) Придумайте MM оценку общего числа зайцев.
б) Придумайте ML оценку общего числа зайцев.
Задача 22.9. Шутка
Вася утверждает, что оценки метода максимального правдоподобия являются состоятельными.
Является ли Васино утверждение
а) максимально правдоподобным;
б) состоятельным?
Задача 22.10.
Найдите MM и ML оценки параметра a, если
2
−a·t
плотности p(t) = a · t · e
при t > 0.
Xi
- независимы и одинаково распределены с функцией
Задача 22.11.
Пусть с.в.
Xi
независимы и имеют функцию плотности
p(t) = a2 e−a·|t| .
Найдите оценку параметра
a) методом максимального правдоподобия
б) методом моментов «приравняв» теоретическую
V ar(Xi )
и эмпирическую
σ̂ 2 ;
a
125
в) методом моментов «приравняв»
E(X 2 )
и соответствующий эмпирический момент.
Задача 22.12. [т?]
Xi
a и b.
Пусть с.в.
метров
независимы и имеют функцию плотности
p(t) = a2 e−a·|t−b| .
Найдите ML оценку пара-
Задача 22.13. [т]
Y1 = β + u1 и Y2 = 2 · β + u2 , где ненаблюдаемые ui независимо и одинаково
распределены, причем E(ui ) = 0 и V ar(ui ) = γ . Оказалось, что y1 = 0.9, а y2 = 2.3. Найдите оценки
методом максимального правдоподобия для β и γ , если:
а) ui - равномерно распределены;
б) ui - нормально распределены;
Пусть наблюдаются
Задача 22.14.
Y1 и Y2 независимы и распределены
E(Y2 ) = ea+b . Найдите ML оценки для a и b.
Ответ: â = ln(Y1 ), b̂ = ln(Y2 ) − ln(Y1 )
Пусть
по Пуассону. Известно также, что
E(Y1 ) = ea
и
Source: Suhov, Probability and statistics
Задача 22.15.
Пусть
Xi
независимы и одинаково распределены
По выборке
X1 ,
...,
Xn
постройте оценку для
N (α, 2α)
a:
а) Методом моментов
б) Методом максимального правдоподобия
Задача 22.16.
На отрезке
величину
[0; b]
X,
равномерно независимо друг от друга выбираются два числа. Пете сообщают
максимум из этих двух чисел. Васе сообщают величину
X,
минимум из этих двух
чисел.
а) Помогите Васе и Пете построить оценки неизвестного
b
методом моментов и методом максималь-
ного правдоподобия.
б) Какие из четырех оценок будут несмещенными?
в) Какая из четырех оценок будет наиболее эффективной? Наиболее эффективной среди несмещенных?
г) Решите эту задачу, если вместо двух чисел на отрезке выбираются
n
чисел.
Задача 22.17. Корректоры очепяток-2
p, Петя независимо от Васи замечает опечатку вероятностью
Известно, что Вася обнаружил N1 опечаток, Петя - N2 опечаток,
Вася замечает опечатку с вероятностью
q.
В книге имеется
причем
N12
n
опечаток.
опечаток были обнаружены и Петей, и Васей.
а) Предполагая, что параметры
p
и
q
известны, а
n
- неизвестен, построить оценку
n
методом
максимального правдоподобия.
б) Предполагая, что
p, q , n
- неизвестны, построить их оценки методом максимального правдоподо-
бия.
Solution:
korrektori ochepiatok
Задача 2.22 может быть полезна.
а) Находим вероятность получить заданные
N1 , N2 , N12
p, q и n (получаем f (p, q, n))
1)/f (p, q, n) ≥ 1
как функцию от
Максимизируем по n: значение n увеличиваем пока f (p, q, n +
N +N2 −N12
Получаем n̂ = 1
(если забыть про целочисленность)
p+q−pq
б) Оценки ML для p и q выглядят стандартно:
p̂ = Nn̂1 , q̂ = Nn̂2
Решая систему уравнений получаем:
N2
n̂ = NN112
126
p̂ =
q̂ =
N12
N2
N12
N1
23. Проверка гипотез о среднем
Задача 23.1.
До проведения рекламной компании в среднем 7 из 10 посетителей художественного магазина-салона
делали покупки. После рекламной компании из 200 посетителей покупки сделали 163. Можно ли
считать, что рекламная компания имела эффект на 5%-ом уровне значимости?
Задача 23.2.
Дневные расходы электроэнергии на предприятии составляли 1400 КВт со стандартным отклонением 50 КВт. За 50 дней прошедших после ремонта и наладки оборудования средние расходы за день
составили 1340 КВт. Можно ли считать, что ремонт способствовал экономии электроэнергии на 10%
уровне значимости?
Задача 23.3.
Монету подбросили 1000 раз, при этом 519 раз она выпала на орла. Проверьте гипотезу о том, что
монета «правильная» на уровне значимости 5%.
Задача 23.4.
Вася отвечает на 100 тестовых вопросов. В каждом вопросе один правильный вариант ответа из
пяти возможных. На 5%-ом уровне значимости проверьте гипотезу о том, что Вася ставит ответы
наугад, если он ответил правильно на 26 вопросов из теста.
Задача 23.5.
Некоторых студентов спросили, на какую оценку они рассчитывает по теории вероятностей, 30
человек надеются на 4 балла, 20 человек – на 6 баллов, 30 человек – на 8 баллов, 10 человек – на 10
баллов. Проверьте гипотезу о том, что медиана равна 7 баллам на уровне значимости 10%.
Задача 23.6.
Кубик подбросили 160 раз, из них 29 он выпал на шестерку. Проверьте гипотезу о том, что вероятность выпадения шестерки правильная на уровне значимости 10%.
Задача 23.7.
Двести домохозяек попробовали новый «Вовсе не обычный порошок», 110 из них получили более
удачный результат, чем раньше. На уровне значимости 5% проверьте гипотезу о том, что «Вовсе не
обычный порошок» по эффективности не отличается от старого средства (против альтернативной
гипотезы о большей эффективности).
Задача 23.8.
Величины
x1 , x2 ,
...,
xn
независимы и распределены
N (10, 16).
Вася знает дисперсию, но не знает
среднего. Поэтому он строит 60% доверительный интервал для истинного среднего значения. У него
получаются две границы - левая и правая.
Какова вероятность того, что: левая меньше 9, левая и правая забирают настоящее среднее?
Доделать, может включить неизвестную дисперсию?
Задача 23.9.
По предварительному опросу 10000 человек на выборах в Думу 462 человека будут голосовать за
партию «Обычная партия». На уровне значимости 0,05 проверьте гипотезу о том, что «Обычная
партия» преодолеет 5% барьер.
127
Задача 23.10.
Вася и Петя метают дротики по мишени. Каждый из них сделал по 100 попыток. Вася оказался
метче Пети в 59 попытках. На уровне значимости 5% проверьте гипотезу о том, что меткость Васи
и Пети одинаковая, против альтернативной гипотезы о том, что Вася метче Пети.
Задача 23.11.
По 820 посетителям супермаркета средние расходы на одного человека составили 340 рублей. Из
достоверных источников известно, что дисперсия равна 90000 руб. Постройте
95%
доверительные
интервалы для средних расходов одного посетителя (двусторонний и два односторонних).
Задача 23.12.
В прошлом году средняя длина ушей зайцев в темно-синем лесу была 20 см,
σ = 4.
В этом году у
случайно попавшихся 15 зайцев средняя длина оказалась 24 см. Предполагая нормальность распределения, проверьте гипотезу о том, что средняя длина ушей не изменилась (против альтернативной
гипотезы о росте длины).
Задача 23.13.
Стандартное отклонение количества иголок у ежа равно 130. По выборке из 12 ежей было получено
среднее количество иголок 5120. Допустим, что количество иголок на одном еже можно считать
нормально распределенным.
90%-ый доверительный интервал для среднего количества иголок.
5%-ом уровне значимости проверьте гипотезу о том, что среднее количество иголок равно 5000.
a) Постройте
б) На
Задача 23.14.
Средний бал по диплому студента - c.в.
абитуриентов этого года, составил
для
µ: (4.2424; 4.3576).
4.30.
N (µ; 0.04).
Средний бал, рассчитанный по выборке из 25
По данной выборке был построен доверительный интервал
Какой уровень доверия соответствует этому интервалу?
Задача 23.15.
Вася очень любит играть в преферанс. Предположим, что Васин выигрыш распределен нормально.
За последние 5 партий средний выигрыш составил 1560 рублей, при оценке стандартного отклонения
равной 670 рублям. Постройте 90%-ый доверительный интервал для математического ожидания
Васиного выигрыша.
Задача 23.16.
In 1882 Michelson performed experiments to measure the speed of light. 23 trials gave an average of
299756.2 km/sec with a standard deviation of 107.12. Find a
95%
confidence interval for the speed of
light. The correct answer is 299710.5 so there must have be some bias in his experiments.
Задача 23.17.
An English biologist named Weldon was interested in the ’pip effect’ in dice – the idea that the spots,
or ’pips’, which on some dice are produced by cutting small holes in the surface, make the sides with
more spots lighter and more likely to turn up. Weldon threw 12 dice 26306 times for a total of 315672
throws and observed that a 5 or 6 came up on 106602 throws. Find a
95%
confidence interval for the true
probability of getting 5 or 6 on a dice.
Задача 23.18.
On 384 out of 600 randomly selected farms, the operator was also the owner. Find a
95%
confidence
interval for the true proportion of owner operated farms.
Задача 23.19.
During a two week period (10 weekdays) a parking garage collected an average of $126 with a standard
deviation of $15. Find a 95% confidence interval for the mean revenue.
128
Problems are borrowed from www.math.cornell.edu/∼durrett/ep4a/ep4a.html
Задача 23.20.
In their last 100 chess games played against each other, Bill has won 46 and Monica has won 54. Using
this information and a 95% confidence level, what is the probability that Bill will win a «best of seven»
series with Monica? The first one to win 4 games is the winner and no more games are played.
Hints: First determine a 95% confidence interval for the probability that Bill will win a game. Then, using
the two extremes of this interval, determine the probability that Bill will win the series. This is a binomial
experiment. Bill could win the series in 4 games, 5 games, 6 games, or 7 games. Calculate the probability
of each and add them up. Do this for each of the two interval extremes.
Source: (?):
http://www.artofproblemsolving.com/Forum/viewtopic.php?highlight=probability+game&t=87203
Задача 23.21.
Имеются две монетки. Одна правильная, другая - выпадает орлом с вероятностью
значение
q
0 < q < 0.5,
известно. Монетки неотличимы по внешним признакам. Одну из них (неизвестно какую)
N
H0 :
подкинули
раз и сообщили Вам, сколько раз выпал орел. Опишите процедуру тестирования
гипотезы
«подбрасывалась правильная монетка» против
Ha :
«подбрасывалась неправильная
монетка».
а) Каким должно быть
процентов, если
N
чтобы вероятность ошибок первого и второго рода не превышала 10
q = 0.4
b) Ответьте вопрос «a» при произвольном
q
Задача 23.22.
Имеется две конкурирующие гипотезы:
H0 :
Ha :
Величина
Величина
X
X
распределена равномерно на отрезке
распределена равномерно на отрезке
[0; 100]
[50; 150]
Исследователь выбрал такой критерей:
Если
X < c,
то использовать
H0 ,
иначе использовать
Ha .
а) Что такое «ошибка первого рода», «ошибка второго рода», «мощность теста»?
б) Постройте графики зависимостей ошибок первого и второго рода от
c.
Задача 23.23.
Известно, что
Xi
iid
N (µ; 900)
выборке из 20 наблюдений. Критерий выглядит следующим
иначе выбрать
H0 : µ = 10 против HA : µ = 30 по
образом: если X̄ > c , то выбрать HA ,
. Исследователь проверяет гипотезу
H0 .
а) Рассчитайте вероятности ошибок первого и второго рода, мощность критерия для
б) Что произойдет с указанными вероятностями при росте количества наблюдений (c
в) Каким должно быть
г) Как зависят от
c
c,
чтобы вероятность ошибки второго рода равнялась
вероятности ошибок первого и второго рода (c
c = 25.
∈ (10; 30))?
0, 15?
∈ (10; 30))?
Задача 23.24.
Дама утверждает, что обладает особыми способностями и безошибочно отличает «бонакву» без газа
от «святого источника» без газа.
H0 :
дама не обладает особыми способностями,
Ha :
дама обладает
особыми способностями. При даме в 3 стаканчика из 8-ми налили «бонакву», а в 5 оставшихся «святой источник». При отгадывании стаканчики предлагаются даме в неизвестном ей порядке.
Критерий: принимается основная гипотеза, если дама ошиблась хотя бы один раз и альтернативная
иначе.
а) Рассчитайте вероятности ошибок первого и второго рода, мощность критерия.
б) Сколько из 8 стаканчиков надо наполнить «бонаквой» и сколько «святым источником», чтобы
вероятность ошибки первого рода была минимальной?
129
Коммент: некоторые студенты утверждают, что отличить «святой источник» от «бонаквы» - элементарно, а вот отличить «бонакву» от «акваминерале» - трудно.
Задача 23.25.
Школьник Вася аккуратно замерял время, которое ему требовалось, чтобы добраться от школы до
дома. По результатам 90 наблюдений, среднее выборочное оказалось равным 14 мин, а несмещенная
2
оценка дисперсии - 5 мин .
a) Постройте 90% доверительный интервал для среднего времени на дорогу
б) На уровне значимости 10% проверьте гипотезу о том, что среднее время равно 14,5 мин, против
альтернативной гипотезы о меньшем времени.
в) Чему равно точное
P -значение
при проверке гипотезы в п. «б»?
Ответы:
a)
[13.61; 14.39]
b) Отвергается (Zobserved
c)
= −2.12, Zcritical = −1.28)
Pvalue = 0.017
24. Остальные гипотезы
Задача 24.1.
При подбрасывании кубика грани выпали 234, 229, 240, 219, 236 и 231 раз соответственно. Проверьте
гипотезу о том, что кубик «правильный».
Задача 24.2.
Проверьте независимость дохода и пола по таблице:
< 500 500 − 1000 > 1000
М
112
266
34
Ж
140
152
11
Задача 24.3.
Вася Сидоров утверждает, что ходит в кино в два раза чаще, чем в спортзал, а в спортзал в два
раза чаще, чем в театр. За последние полгода он 10 раз был в театре, 17 раз - в спортзале и 39 раз
- в кино. Правдоподобно ли Васино утверждение?
Задача 24.4.
Проверьте независимость пола респондента и предпочитаемого им сока:
Апельсиновый
Томатный
Вишневый
М
69
40
23
Ж
74
62
34
Задача 24.5.
У 200 человек записали цвет глаз и волос. На уровне значимости 10% проверьте гипотезу о независимости этих признаков.
Цвет глаз/волос
Светлые
Темные
Итого
Зеленые
49
25
74
Другие
30
96
126
Итого
79
121
200
Задача 24.6.
Идея задачи на хи-квадрат.
Если предложить голосовать за 3 альтернативы...
Если предложить голосовать за 4 альтернативы...
Выполняется ли предпосылка независимости от посторонних альтернатив?
Задача 24.7.
Изучалось воздействие модератора на количество идей, сочиняемых группой людей. По выборке из
130
4-х групп с модератором, среднее количество идей оказалось равным 78, при стандартном отклонении 24.4, по выборке из 4-х групп без модератора, среднее количество идей оказалось равным 63.5,
при стандартном отклонении 20.2. Предположим нормальность распределения.
а) Проверьте гипотезу о равенстве дисперсий.
б) Предполагая равенство дисперсий проверьте гипотезу о равенстве средних.
Задача 24.8.
Маркетинговый отдел банка опросил 300 женщин и 400 мужчин. Оказалось, что реклама банка
вызывает положительные эмоции у 74% опрошенных женщин и 69% опрошенных мужчин.
а) Можно ли считать, что реклама банка одинаково нравится мужчинам и женщинам?
б) Постройте 90% доверительный интервал для разницы долей мужчин и женщин, одобряющих
рекламу банка.
в) Проверьте гипотезу о том, что рекламу одобряет 70% женщин (против гипотезы о том, что
рекламу одобряет более 70% женщин).
г) Предположим, что среди потребителей рекламы мужчин и женщин поровну. Постройте 95%
доверительный интервал для доли людей, которым нравится реклама банка.
Задача 24.9.
Исследователь сравнивал суровость климата (дисперсию температуры) в двух странах. Для этого
случайным образом были выбраны 37 наблюдений за среднедневной температурой в первой стране
и 46 наблюдений за среднедневной температурой во второй стране. Известно, что X I = 14 , X II = 11
2
= 3079.
σ̂I2 = 2341 и σ̂II
а) Постройте 95% доверительный интервал для разности математических ожиданий среднедневных
,
температур в двух странах.
б) Предполагая, что среднедневная температура распределена нормально, проверьте гипотезу об
одинаковой суровости климата. С помощью компьютера найдите точное Р-значение.
в) В предположениях о нормальности постройте 90%-ые доверительные интервалы для дисперсий
среднедневной температуры в двух странах. Почему один из интервалов оказался шире?
Задача 24.10.
Имеются две нормальные выборки:
{Xi } = {34, 28, 29, 41, 32} , {Yi } = {32, 30, 31, 25, 24, 29}.
а) Проверьте гипотезу о равенстве дисперсий. С помощью компьютера укажите точное P-значение
б) В предположении о равенстве дисперсий проверьте гипотезу о равенстве математических ожиданий. С помощью компьютера укажите точное P-значение.
Задача 24.11.
Вася Сидоров хвастался перед Аней Ивановой, что в среднем прыгает не меньше, чем на 2 метра.
Напомним его результаты: 1,83; 1,64; 2,27; 1,78; 1,89; 2,33; 1,61; 2,31. Предположим, что длины
прыжков можно считать нормальными
а) Постройте
80%-ый
доверительный интервал для дисперсии длины прыжка.
б) Дополнительно предположив, что
σ = 0, 3
проверьте гипотезу о том, что Вася действительно
прыгает на 2 метра (Аня, естественно, ему не верит, и утверждает, что он прыгает меньше, чем на
2 метра).
в) Постройте двусторонний
90%-ый
доверительный интервал для
µ,
если
σ = 0, 3.
Задача 24.12.
Контрольные камеры ДПС на МКАД, зафиксировали скорость движения 6-и автомобилей: 89, 83,
78, 96, 81, 79. В предположении нормальности скоростей:
90%-ый доверительный интервал для дисперсии скорости.
Постройте 90% доверительный интервал для средней скорости автомобилей.
Постройте 90% доверительный интервал для средней скорости автомобилей,
а) Постройте
б)
в)
настоящая дисперсия равна 50 (км/ч)
2
если известно, что
131
г) На
10%-ом
уровне значимости проверьте гипотезу о том, что средняя скорость равна 90 км/ч.
Задача 24.13.
На курсе два потока, на первом потоке учатся 40 человек, на втором потоке 50 человек. Средний
балл за контрольную на первом потоке равен 78 при (выборочном) стандартном отклонении в 7
баллов. На втором потоке средний балл равен 74 при (выборочном) стандартном отклонении в 8
баллов.
а) Постройте 90% доверительный интервал для разницы баллов между двумя потоками
б) На 10%-ом уровне значимости проверьте гипотезу о том, что результаты контрольной между
потоками не отличаются.
в) Рассчитайте точное P-значение (P-value) теста в пункте ’б’
Задача 24.14.
Предположим, что время жизни лампочки распределено нормально. По 10 лампочкам оценка
стандартного отклонения времени жизни оказалась равной 120 часам.
а) Найдите 80%-ый (двусторонний) доверительный интервал для истинного стандартного отклонения.
б) Допустим, что выборку увеличат до 20 лампочек. Какова вероятность того, что выборочная
оценка дисперсии будет отличаться от истинной дисперсии меньше, чем на 40%?
Задача 24.15.
Допустим, что логарифм дохода семьи имеет нормальное распределение. В городе А была проведена
2
случайная выборка 40 семей, показавшая выборочную дисперсию 20 (тыс.р.) . В городе Б по 30
2
семьям выборочная дисперсия оказалась равной 32 (тыс.р.) .
На уровне значимости 5% проверьте гипотезу о том, что дисперсия одинакова, против альтернативной гипотезы о том, что город А более однородный.
Задача 24.16.
Допустим, что логарифм дохода семьи имеет нормальное распределение. В городе А была проведена
2
случайная выборка 40 семей, показавшая выборочную дисперсию 20 (тыс.р.) . В городе Б по 30
2
семьям выборочная дисперсия оказалась равной 32 (тыс.р.) .
На уровне значимости 5% проверьте гипотезу о том, что дисперсия одинакова, против альтернативной гипотезы о том, что город А более однородный.
Solution:
F29,39 = 32
= 1.6
20
Fcritical = 1.74
Гипотеза о том, что дисперсия одинакова не отвергается.
25. Дисперсионный анализ
Задача 25.1.
Ниф-ниф ходит обедать в одно из трех близлежащих кафе.
Вот данные о его расходах (в рублях):
Кафе «Четыре поросенка»: 160, 100, 230, 200
Кафе «Суши для поросят»: 120, 100, 140, 150, 110, 160, 280
Кафе «Здоровый хряк»: 210, 160, 140, 140, 150
а) Проведите однофакторный дисперсионный анализ и сделайте выводы.
б) Постройте интервалы для разницы расходов между разными кафе
Задача 25.2.
Данные о продаже мороженого в киоске, сгруппированные по годам и сезонам:
Зима Весна Лето Осень
1980 90 120 420 200
132
1981 80 150 380 210
1982 90 100 410 180
1983 100 130 540 180
а) Проведите двухфакторный дисперсионный анализ и сделайте выводы.
б) Можно ли сделать вывод о том, что продажи в 83 году выросли по сравнению с 82?
26. Непараметрические тесты
Задача 26.1.
Из 10 опрошенных студентов часть предпочитала готовиться по синему учебнику, а часть - по
зеленому. В таблице представлены их итоговые баллы.
Синий
76
45
57
65
Зеленый
49
59
66
81
38
88
а) С помощью теста Манна-Уитни (Mann-Whitney) проверьте гипотезу о том, что выбор учебника
не меняет закона распределения оценки.
Разрешается использование нормальной аппроксимации
б) Возможно ли в этой задаче использовать (Wilcoxon Signed Rank Test)?
Задача 26.2.
Имеются результаты экзамена в двух группах. Группа 1: 45, 67, 87, 71, 34, 12, 54, 57; группа 2: 46,
66, 81, 72, 11, 47, 55, 51, 9, 99. На уровне значимости
5%
проверить гипотезу о том, что результаты
двух групп не отличаются.
Задача 26.3.
Имеются результаты нескольких студентов до и после апелляции (в скобках указан результат до
апелляции): 48(47), 54(52), 67(60), 56(60), 55(58), 55(60), 90(70), 71(81), 72(87), 69(60). На уровне
значимости
5%
проверьте гипотезу о том, что апелляция в среднем не сказывается на результатах.
Задача 26.4.
Имеются наблюдения за говорливостью 30 попугаев (слов/день): 34, 56, 32, 45, 34, 45, 67, 1, 34,
12, 123, ... , 37 (всего 13 наблюдений меньше 40). Проверить гипотезу о том, что медиана равна 40
(слов/день).
Задача 26.5.
Вашему вниманию представлены результаты прыжков в длину Васи Сидорова на двух соревнованиях. На первых среди болельщиц присутствовала Аня Иванова (его первая любовь): 1,83; 1,64;
2,27; 1,78; 1,89; 2,33; 1,61; 2,31. На вторых Аня среди болельщиц не присутствовала: 1,26; 1,41;
2,05; 1,07; 1,59; 1,96; 1,29; 1,52; 1,18; 1,47. С помощью теста (Mann-Whitney) проверьте гипотезу о
том, что присутствие Ани Ивановой положительно влияет на результаты Васи Сидорова. Уровень
значимости
α = 0.05.
Задача 26.6.
Некоторые результаты 2-х контрольных по теории вероятностей выглядят следующим образом
(указан результат за вторую контрольную и в скобках результат за первую): 43(55), 113(108), 97(53),
68(42), 94(67), 90.5(97), 35(91), 126(127), 102(78), 89(83). Можно ли считать (при
α = 0.05),
что
вторую контрольную написали лучше?
Задача 26.7.
Садовник осматривал по очереди розовые кусты вдоль ограды. Всего вдоль ограды растет 30
розовых кустов. Из них оказалось 20 здоровых и 10 больных.
Вот заметки садовника:
(+ - здоровый куст,
+ + + + + + + + + + + + + + + + + + ++
- больной куст)
а) С помощью теста серий проверьте гипотезу о независимости испытаний
133
б) Какой естественный смысл имеет эта гипотеза?
Подсказка: можно использовать нормальное распределение
27. Solutions
Vasya, Petya i figuristka
13.27
вроде бы
a1 =
σ22
и
σ12 +σ22
a2 = 1 − a1
simple third
1.53
Устно:
1
3
dlia ravnomernoi monetki
15.3
f (a, b) =
a!b!
(a+b+1)!
ravnomernaia monetka
17.2
a!b!
f (a, b) = (a+b+1)!
P
rob =if jensen
(k + 1, n − k)/f (k, n − k) =
moloko
k+1
n+2
13.32
1.
2.
E(P ) = 10. E( 10
) ≥ 1. Заплатили поровну, купила больше баба Катя.
P
E(P + 10) = 10E(1 + 10
). E( 10
) ≥ 1. E(P ) ≥ 10. Заплатила больше баба
P
P
Маша, купила больше
баба Катя.
chetnost chisla chernih
5.46
Число черных всегда нечетно. Единица.
new shaman
5.31
Answer: 24
Solution:
Решаем систему уравнений:
x∅ = 1 + 13 xa + 32 xother
xa = 1 + 12 xab + 21 xother
xab = 1 + 12 xabr + 12 xother
xabr = 1 + 12 0 + 12 xother
xother = 1 + 12 xa + 21 xother
Solution (Non state-space):
If the bug is both not on A and has deviated from the ABCA sequence, it will take him an expected two
moves to get back to A by a simple geometric sum.
Now suppose the bug is on A beginning a possible ABCA. He has a 1/2 chance of moving to C, and that
adds 3 expected moves: one to move to C, expected two to get back to A.
The alternative is that he moved to B with 1/2 chance. Now he has a 1/2 (1/4 cumulative) chance of
moving back to A, adding 2 moves (ABA). Otherwise, he’s still going.
Now he’s on C. There’s a 1/2 (1/8 cumulative) chance of moving to B and adding 5 moves (ABCB plus
the two moves to return to A). Else, he moves to A, gets squished, and finishes with 3 moves (ABCA).
So we have the following possibilities:
AC: +3 moves (1/2 = 4/8 chance)
ABA: +2 moves (1/4 = 2/8 chance)
ABCB: +5 moves (1/8 chance)
ABCA: +3 moves and finish (1/8 chance)
So the expected value is
3·4+2·2+5+3=
24
134
On-Off
5.32
Ответ:
1
2
wait for n
5.34
pn = 13 pn−1 + 23 pn−2 , p0 =
1
,
4
p1 =
1
16
healthy passenger
??
99
2
additional shoot
5.43
Solution: Во первых, заметим, что ожидаемое количество выстрелов, если у Вас осталось
имеет вид
n патронов
En = k · n.
Во-вторых, получим уравнение на
En :
En = 1 + En−1 + kE(X),
1
Находим k : k =
1−E(X)
- ожидаемый выигрыш патронов от одного выстрела.
100
1−0.45
Solution2: Интуитивно:
где
E(X)
Ответ задачи:
100 + 100 · 20 · 0.01 + 100(·20·)2 + ...
newspapers
5.55
Solution 1:
p2 = 12
n−1
сверху стопки лежит номер, меньший n, в этом случае можно считать, что n-ый
С вероятностью
n
номер вообще отсутствует в стопке.
1
С вероятностью
сверху стопки лежит n-ый номер, тогда обязательно происходит одно переклаn
дывание, после которого мысленно выкинув n-ый номер можно считать, что имеется случайно
упорядоченная стопка из (n −
pn = n−1
pn−1 + n1 (pn−1 + 1)
n
Pn 1
Итого: pn =
i=2 i ≈ n ln(n)
Solution 2:
Пусть
qi -
1)
выпуска.
вероятность того, что число
i
«уберут» с верха стопки.
q1 = 0
i «уберут» с верха
2,... i число i будет первым, т.е. 1i .
P
E(X) = E(X2 ) + ... + E(Xn ) = ni=2 1i ≈ n ln(n)
Вероятность того, что число
стопки равна вероятности того, что среди чисел
1,
tri chisla
2.18
3
C10
103
Каждый способ выбрать три разных числа соответствует благоприятной комбинации.
Solution:
korabli na planete
7.7
a)
3/8
в-а) Зафиксируем координату посадки первого корабля. Обозначим центральный угол между пер2π sin(α)
вым и вторым кораблем α. Функция плотности имеет вид p(α) =
.
4π
R π/2
Rπ
α+π
π−α
π+2
Итог:
p(α) 2π dα + π/2 p(α) 2π dα = 4π .
0
cut of apple
7.19
Solution
Во-первых. Вместо выбора
n
точек наугад будем выбирать
n
осей (n пар точек) наугад, а затем на
каждой оси выбирать наугад одну из двух точек. Каждой оси соответствует один экватор.
2
Во-вторых. Имеющиеся n экваторов разрезают яблоко на n − n + 2 части. Доказываем по индукции:
каждый новый экватор пересекает имеющиеся
пересечение дает одну новую часть.
k
экваторов
2k
раз (каждый по два раза), каждое
135
В-третьих. На
n
осях имеется
2n
разных комбинаций выбора конкретных n точек.
n2 − n + 2 имеющихся. На этой части выберем
В-четвертых. Рассмотрим произвольную часть из
произвольную точку. Проведем ось через эту точку. Разрежем яблоко по соответствующему экватору. Выберем ту половину, в которой лежит выбранная точка. В этой половине лежат
заданных пар. Любая точка из той же части приведет к отрезанию тех же
n2 −n+2
Итого: p =
2n
strategia udvoenia
1.30
1−
b) 0
a)
1
1024
shokoladnie
konfeti
4
1.19
20
Vasia i Petia na lektsii
1.24
P (оба придут) = 0.7 · 0.6 = 0.42
P (хотя бы один придет) = 1 − (1 − 0.7) · (1 − 0.6) = 0.88
sudba Don Juan
1.25
a)
b)
E(X) = 1, V ar(X) = 1
1 − 2!1 + 3!1 − ..., 1e
do 2-h v summe
1.22
X
1
2
24
11
Prob
36
36
49
E(X) = 36
3
1
36
E(36X − 17) = 32
371
V ar(X) = 1296
V ar(36X − 5) = 371
f dlia nepravilnoi moneti
5.37
f (p) =
1
2−p
legkomislennii chlen juri
1.46
p−
p2
2
< p,
т.е. жюри из одного человека лучше
iz 5 detalei 2 brakovannih
1.27
X
1
2
3
12
6
2
Prob
20
20
20
E(X) = 23 , V ar(X)
=
9
20
orli podriad
5.45
P (A|B) = pk−1 · 1 + (1 − pk−1 )P (A|B c )
P (A|B c ) = q r−1 · 0 + (1 − q r−1 )P (A|B)
P (A) = p · P (A|B) + q · P (A|B c )
tri shara iz korobki
1.66
E(B) = 3 · 94 = 43 , E(G) = 3 − E(B) =
E(B − G) = − 34 , E(B · G) = 2 · 56
8
3
Iska priglasil 3 druzei
1.70
P (N = 0) = 0.1 · 0.3 · 0.5
P (N = 3) = 0.9 · 0.7 · 0.5
P (N = 1) = 0.9 · 0.3 · 0.5 + 0.1 · 0.7 · 0.5 + 0.1 · 0.3 · 0.5
a)
n
точек.
n
точек из
n
136
E(N ) = 0.9 + 0.7 + 0.5, V ar(N ) = 0.9 · 0.1 + 0.7 · 0.3 + 0.5 · 0.5
b)
abrikos, bej i raznie viborki
4.32
a
a+b+c
na
b)
a+b+c
в) V ar(Ai )
a)
= pa (1 − pa ), Cov(Ai , Bi ) = −pa pb , ov(Ai , Bj ) = −pa pb N1−1
q
pa pb
N −n
−n
, Cov(A, B) = −npa pb
, Corr(A, B) = −
V ar(A) = npa (1 − pa ) N
N −1
N −1
(1−pa )(1−pb )
V ar(Ai ) = pa (1 − pa ), Cov(Ai , Bi ) = −pa pb , Cov(Ai , Bj ) = q
0
pb
V ar(A) = npa (1 − pa ), Cov(A, B) = −npa pb , Corr(A, B) = − (1−ppaa)(1−p
b)
г)
optimalnaia f plotnosti
11.17
а) равновесия в чистых нет.
б) Первый игрок должен быть безразличен между усилиями:
Если первый игрок выбирает уровень усилий
довательно:
R e1
p(t)dt − 2e21 = 0 − 2a2 =
0
Поскольку есть стратегия
1 − 2b2
e1 = 0,
e1 ,
e1 ∈ [a; b].
Т.е.
U (e1 ) = U
= U (b).
R (a)
e1
p(t)dt
. Сле0
то он выигрывает с вероятностью
приносящая полезность 0, любая играемая стратегия должна
приносить платеж не меньше 0.
√
a = 0 и b = 1/ 2√
. Взяв
p(t) = 4t на отрезке [0; 1/ 2].
Отсюда
производную по
e1
получаем:
sumashedshaia starushka
5.27
1/2,
для
n=2
очевидно, далее по индукции
sumashedshaia starushka 2
5.30
1/2.
Доказательство по индукции. Для
вероятностью
k=1
и
z=1
ответ очевиден. При других
α
вяжется всего два шарфа и c вероятностью
α
Искомая вероятность равна P =
· 1 + α2 · 0 + (1 − α) · 12
2
1−α
k
и
z
с некоторой
снижается размерность задачи.
Simpson’s paradox
1.47
a) спортсмена Б; б) спортсмена А; в) Зависимость от третьей альтернативы.
dve shkatulki
1.31
a) нет, условное распределение суммы можно определить, только зная безусловное.
б) при заданном безусловном распределении Васе следует сменить свой выбор вне зависимости от
того, что он увидел в первой шкатулке. Вторая открытая лучше первой открытой. Это возможно
из-за того, что безусловная ожидаемая сумма равна бесконечности для обеих шкатулок.
simple optimization
3.19
1
5 + 12 27
2
= 4.25
n-gon
7.17
n+1
Answer:
4n−2
center
inside
1
7.18
8
razorenie
4.7
Solution:
Пусть
Xn
- благосостояние Васи после
n-го
хода, тогда
E(Xn ) = 100. E(Xf inal ) = 250p + 0(1 − p).
kopilki
5.51
f (k, n) = nk f (k − 1, n − 1) +
n−k
f (k, n − 1).
n
k
n−k
Ключ из первой разбитой копилки с вероятностью
не дает ничего нового, а с вероятностью
n
n
б)
137
«разбивает» еще одну копилку.
k
Решение f (k, n) = .
n
В силу простоты ответа хочется какого-нибудь более красивого решения. Welcome!
k(n+1)
в)
k+1
k(n+1)
. Красивое док-во - welcome!
г)
(k+1)n
ritsari-bliznetsi
1.18
1
1
P = 17 + 14
+ 28
1
b) P = n
· 2 · (1 − 0.5n )
2 −1
a)
vtoroi v finale
??
P =
4
7
po rublu za 6
1.68
E(X) = 3 · 16 + 1 + (−2) ·
5 3
6
sin
14.4
pY (t) =
√2
при
π 1−t2
t ∈ [0; 1]
dva ferzia
1.101
14
1 1
a)
+ 64
4(7 · 7 + 5 · 9 + 3 · 11 + 1 · 13)
63
63
Шахматная доска делится на четыре квадратных зоны с одинаковым числом клеток, покрываемых
ферзем.
b) 0, длина нескольких линий по отношению к площади
vtoroie podkidivanie
1.48
2
a) По сути эксперимента: Binomial(n,p )
b)
c)
q
p
1+p
0, 0.5,
чез=черт его знает
sleeping beauty
1.95
«Сегодня понедельник» - это НЕ событие. Вероятность не определена. Это функция от времени.
Вероятность того, что монетка выпала орлом равна
0.5.
Поэтому ей все равно, как отвечать, если
наказанием является превращение в тыкву, и нужно отвечать «решка», если наградой является
молодильное яблоко. Предполагается, что красавица максимизирует ожидаемое количество молодильных яблок.
maksimum iz kartochek
1.94
1
, одна из k +
k+1
rekordnaia
volna
1.93
1
карточек должна быть наибольшей
a) Какая-то из первых
k
величин будет наибольшей. В силу iid, получаем, что
P (Ek ) = 1/k
b) Да. Например, если известно, что 10-ое наблюдение было рекордом, это ничего не говорит о
рекордах в первых 9-ти наблюдениях.
1
с)
n(n−1)
d)
∞
pop i balda
11.16
pi f (p̂i ) + (1 − pi )f (1 − p̂i ) был в точке p̂i = pi необходимым условием будет:
pf (p) = (1 − p)f (1 − p).
Если левую часть обозначить q(p), то получаем уравнение q(p) = q(1 − p). Берем любую функцию,
симметричную относительно 1/2 (останется потом только проверить, что p̂i = pi - это максимум, а
не минимум). Например, подойдет q(x) = 1, тогда получаем f (x) = ln(x), или q(x) = 2x(1 − x), тогда
Чтобы максимум
0
0
138
получаем
f (x) = −x2 + 2x
Euler’s formula
10.1
a)
P ({X
делится на
p}) = p−s
b) Левая и правая части - это вероятность того, что число не делится ни на одно простое. Такое
число одно - единица.
1
в) P =
ξ(2s)
X
г) Если X делится на k , то условное распределение
совпадает с безусловным распределением X :
k
X
−s
P ( k = n|X = 0 mod k) = n /ξ(S) = P (X = n)
д) [better is welcome?]
X
Y
Аналогично, можно заметить, что в случае, когда X и Y делятся на k , то
и
будут (условно)
k
k
независимы:
P ( Xk = x ∩ Yk = y|X = 0 mod k ∩ Y = 0 mod k) = P ( Xk = x|X = 0 mod k)P ( Yk = y|Y = 0 mod k)
P (p не является общим делителем X и YQ) = (1 − p−2s )
1
P (X и Y не имеют общих делителей) = p is prime (1 − p−2s ) = ξ(2s)
.
−2s
Вероятность того, что X и Y делятся на k равна k
.
k−2s
Искомая вероятность равна
ξ(2s)
tsvetnaia sfera
10.3
Да. Впишем куб наугад.
Следовательно,
P ({все
Ai = {i-ая
вершина белая}.
вершины красные})
≥ 0.2,
P (Ai ) = 0.1
и
P (∪Ai ) ≤
P
P (Ai ) = 0.8.
т.е. искомый куб существует.
izvlekaem do odnotsvetnih
4.30
a) Доизвлекаем все шары. Искомая вероятность равна вероятности того, что последним будет
a
извлечен абрикос. А эта вероятность равна
a+b
b) Пусть с.в. A - число шаров-абрикосов в конце, а B - число шаров беж в конце. Т.е. если в конце
a
b
извлекается три абрикосовых шара, то A = 3, а B = 0. Искомая величина это E(A)+E(B) =
+ a+1
b+1
в) Поменяем порядок. Каково ожидаемое количество шаров до первого абрикосового, если известно,
b−1
что первый шар - беж. 1 +
a+1
Parrondo’s game
1.73
a) убывать
б) убывать
в) возрастать
5 ranshe 7
5.18
Применяя метод первого шага получаем уравнение,
p=
4
36
·1+
6
36
·0+
26
36
· p.
raznie pravila
11.19(C.L. Anderson)
При любом варианте правил, Маша будет ходить белыми не больше 10 раз, а Саша - не больше 9
раз. Победитель гарантированно определяется за 19 партий. Если победитель определился раньше,
то дополним турнир недостающими фиктивными партиями (чтобы Саша ходил белыми ровно 9 раз,
а Маша - ровно 10 раз). В турнире из 19 партий победителем при любом варианте правил оказывается тот, кто выиграл больше партий. Следовательно, Машины шансы не зависят от выбираемого
варианта правил.
optimalnoe raskladivanie
11.1
Один белый отдельно, остальные 99 шаров вместе.
vilkodir
2.9
Представим себе мифический объект «вилкодыр». Он может превращаться либо в вилку, либо в
139
дырку. Вилкодыров у нас
n − k.
Из них нужно выбрать
k
вилок. Ответ:
k
Cn−k
б)
n
k
Cn−k
n−k
lampochki v riad
2.10
k
3 · Cn+1−d
,
k
где
- число серий,
d
- суммарная длина серий. Домножение на 3 взялось из трех
вариантов (2-3-3, 3-2-3, 3-3-2). Мифический объект - серия из горящих лампочек плюс негорящая
справа.
mosti
3.39
1/2. Если рассмотреть «проплывы» под мостами (по научному, двойственный граф), то он выглядит
точно также. Вероятность того, что есть проплыв под разрушенными мостами равна вероятности
добраться с одного берега на другой. А в сумме они равны единице.
chepchiki
4.28
г)
1
2!
1−
+
1
3!
−
1
4!
+ ...
mechenii shar
4.29
a) Персиковые шары кроме меченого на вероятность не влияют. Следовательно,
b)
P = 1/(m + 1)
E = k/(m + 1)
ogranichennie patroni
5.42
E(X) =
1−q n
1−q
dve tochki na otrezke
7.6 б)
1/4
simple bayesian updating
17.1
pQ (t|
монетка выпала орлом
Находим
P (Q ≤ t|
) = c · t · pQ (t)
монетка выпала орлом
),
причем на знаменатель не обращаем внимания, т.к. он
- константа. Затем берем производную. Или интуитивно.
Monty Hell problem
1.96
a) 1 b) 0
id001
2.4
4
C25
id002
2.5 5!
id003
а)
б)
3
3!
C25
2.6
4
5
C
12 C8
id004
2.7
2 3!3!
6!
podpravka i udavi
4.41
a) 0 б)
Cov(Xi , Xj ) = − N1−1 σ 2
в)
V ar(X̄n ) =
mishka ishet sir
5.10
m=
1
3
+ 31 (2 + m) + 13 (3 + m), m = 6
id005
1.1
P (хотя бы одна шестерка) = 1 −
2
P (ни одной шестерки) = 56
id006
1.2
P (ровно
P (ровно
= C51 ( 21 )5
2 1 5
«орла») = C5 ( )
2
один «орел»)
два
52
6
σ 2 N −n
n N −1
140
P (ни
одного «орла»)
= C50 ( 21 )5
id007
1.3
2 1 2 5 4
P
(ровно две
шестерки) = C6 ( ) ( )
6
6
simmetria
razbitia
otrezka
1.4
Да. Возьмем окружность. Наугад отметим три точки. Одну будем трактовать как разрезающую
окружность на отрезок. Две других, как разрезающие отрезок на три части.
id008
1.5
C 10
id009
P = C46
10 1.6
50
P
({b}) = 0.2, P ({a}) = 0.6, P ({c}) = 0.5
id010
c
c
1.7 P (A ∩ B ) = 1 − 0.3 − 0.4
id011
1.8 P (A ∩ B) ∈ [0.1; 0.3]
id012
1
1.9 P =
3
id013
1.10
P (результат второго броска будет строго
5
P (в сумме будет 6) = 36
4
P (в сумме будет 9) = 36
9
P (Максимум равен 5) = 36
7
P (Минимум равен 3) = 36
P
(Разница будет равна 1 или 0) = 16
36
id014
больше, чем результат первого)
1.11 Равны
id015
1.12
P (ровно
P (ровно
P
(ровно
id016
4
5
3
= 2 21
1 14
партии) = 2C3
2
2 15
партий) = 2C4
2
3 партии)
1.13
365·364·363·...·356
365n
минимальная компания в 23 человека
5 iz 36
1
1.14 P = 5
C36
id018
1.15
9 C 1 id019
C45
5
1.16
10
C50
id020
1
P = 4 (1 − f rac18) + 12 ( 81 + 3 18 ) + 14 18 = 21 1.17
4
4 15
4 16
( 12 + C
P
= 16raznih
5 2 + C6 2 )
deti
polov
1.20
1 (n−1)
(ojidanie
)
≤ot0.15
2
bernulli
1.21 E(X) = p
s chego vse nachinalos
1.26
700 : 100
korreliatsia dlia kubika
4.1
Cov(X1 , X1 + ... + X6 ) = 0,
Corr(X
− 15
1 , Xpo
6) =
karandashi
korobkam
4.2
10 · (1 −
9 7
)
10
mabinogion easy
5.35
bad solution:
b100 = 1
b0 = 0
n
bn = 100
bn+1 +
100−n
bn−1
100
т.к.
X1 + ... + X6 = const
=
15
36
141
4n = bn − bn−1 :
b100 = 41 + 42 + ... + 4100 = 1
4n+1 = 100−n
4n
n
После замены
Решая, находим:
n−1 −99
4n =P
C99
2
k −99
bSpagetti
n =
0≤k≤(n−1) C99 2
5.47
1
получаем 1 узел и гарантированно снижаем число
2n−1P
n
1
1
спагетти на единицу. Следовательно, E(X) =
i=1 2n−1 , примерно, 2 ln n
1
б) вероятность того, что заданная спагетти завяжется сама с собой равна
, значит среднее равно
2n−1
n
2n−1
Сделав первый узел мы с вероятностью
gadanie v pole
1.60
8/15
ugadivanie ravnomernih
11.22
Решение (Winkler)
Выбирая свой ответ наугад, Саша может гарантировать себе вероятность выигрыша 50%. Переда1
далее, Маша лишает Сашу релевантной для
вая Саше ту величину, значение которой легло от
2
отгадывания информации.
kart do 1 tuza
4.34
Решение (предложила Саша Серова)
Сначала сделаем колоду из четырех тузов, лежащих стопкой в случайном порядке. Затем карты
будем по одной класть на случайное место в формируемой колоде. Всего 5 мест. По индукции
1
видно, что вероятность для каждой карты попасть на место впереди первого туза равна . Если X
5
1
- количество карт, попавших раньше первого туза, то X = X1 + ... + X48 и E(X) = 48 ·
= 48
.
5
5
Решение (идея Mosteller, 50 задач?)
Добавим в колоду ложного, пятого, туза. Разложим колоду случайным образом по окружности.
Пятый туз разделит окружность на обычную колоду (именно за этим он и нужен). До разрезания
окружности пять тузов разбивали оставшиеся 48 карт на пять равнораспределенных частей.
ravnomernaia summa
16.33
Нет, на квадрате появляется противоречие
dn001
dn013
18.13
dn024
18.24
dn035
dn002 dn003
dn004 dn005
dn006
dn007
dn008 dn009
dn010
dn011
dn012
dn014
dn015
dn016
dn017
dn018
dn019
dn020
dn021
dn022
dn023
да 18.14 нет 18.15 да 18.16 нет 18.17 да 18.18 нет 18.19 да 18.20 нет 18.21 да 18.22 нет 18.23 нет
dn025
dn026
dn027
dn028
dn029
dn030
dn031
dn032
dn033
dn034
да 18.25 да 18.26 да 18.27 нет 18.28 нет 18.29 нет 18.30 да 18.31 нет 18.32 нет 18.33 да 18.34 нет
dn036
dn037
dn038
dn039
dn040
dn041
18.1 да 18.2 да 18.3 нет 18.4 да 18.5 нет 18.6 нет 18.7 нет 18.8 да 18.9 нет 18.10 да 18.11 да 18.12 да
18.35 да 18.36 нет 18.37 нет 18.38 да 18.39 нет 18.40 нет 18.41 нет
28. Функции MS Excel
СЛЧИС(), RAND() = генерирует с.в. равномерно распределенную на [0; 1]
P
Xi
СРЗНАЧ(Набор чисел) = X̄ =
n
P
2
(Xi −X̄)
ДИСП(Набор чисел) =
n−1
СТЬЮДРАСП(x,n,1) = P (Tn > x), где Tn - с.в. имеющая t распределение c
СТЬЮДРАСП(x,n,2) =
P (|Tn | > x) = 2 · P (Tn > x),
где
Tn
степенями свободы
СТЬЮДРАСПОБР(p,n) = обратная к СТЬЮДРАСП(x,n,2)
2
НОРМРАСП(x,µ,σ ,1) = P (N ≤ x), где N ∼ N (µ, σ )
НОРМСТРАСП(x) =
P (N ≤ x),
где
N ∼ N (0, 1)
НОРМРАСПОБР(p,µ,σ ) = обратная к НОРМРАСП(x,µ,σ ,1)
НОРМСТОБР(p) = обратная к НОРМСТРАСП(x)
2
ХИ2РАСП(x,n) = P (C > x), где C ∼ χn
n
степенями свободы
- с.в. имеющая
t
распределение c
n
142
ХИ2ОБР(p,n) = обратная к ХИ2РАСП(x,n)
P (F > x), где F ∼ Fn1 ,n2
n1 , n2 ) = обратная к FРАСП(x,n1 ,n2 )
FРАСП(x,n1 ,n2 ) =
FРАСПОБР(p,
29. Обозначения
X ∼ U [a; b]
- случайная величина
X
распределена равномерно на отрезке
[a; b],
буква U - от слова
uniform
X ∼ N (µ, σ 2 )
X ∼ N (µ, Σ) Σ
- ковариационная матрица
iid - independent and identically distributed, независимы и одинаково распределенные
30. Все, что вы хотели узнать, но боялись спросить
Ω
= множество всех исходов
событие = набор исходов
свойства вероятности:
P (Ω) = 1, P (∅) = 0, 0 ≤ P (C) ≤ 1
Если A и B несовместны (не могут произойти одновременно, A ∩ B = ∅ ), то P (A ∪ B) = P (A) + P (B).
Существуют такое событие
Число способов выбрать
k
D,
что
P (D) = 0,
предметов из
n
P 6= ∅.
но
( из
n
по
k ),
если не важен порядок:
Cnk = (
n
)=
k
n!
.
k!(n−k)!
Maple: binomial(n,k); n!;
A, если
P (B) > 0
Условная вероятность наступления события
условная вероятность определена при
A и B
P (A|B) = P (A)
события
Ac = Ω\A
называются независимыми, если
- дополнение к событию
формула полной вероятности
известно, что
B
наступило =
P (A ∩ B) = P (A) · P (B)
P (A|B) =
или (для
P (A∩B)
P (B)
P (B) > 0)
A
P (A) = P (A|B1 ) · P (B1 ) + ... + P (A|Bn ) · P (Bn ),
если
Bn
если
P (a ≤ X ≤ b) =
- не пересека-
ются, и в сумме исчерпывают все возможные варианты.
Функция распределения
Функция
Rb
a
p(t)dt.
p(t)
F (a) = P (X ≤ a).
называется функцией плотности для случайной величины
Функция плотности - это не вероятность. Это - почти :) вероятность.
Для случайных величин с функцией плотности
Для всех
X,
P (X = t) = 0.
P (−∞ < X < +∞) = 1.
p (x|y) =
R X|Y
a
F (a) = −∞ p(t)dt.
Условная функция плотности
Для непрерывных с.в.
P
E(X)
R = xi · P (X = xi ),
E(X) = x · p(x) · dx.
Для дискретных величин
Для непрерывных -
pX,Y (x,y)
pY (y)
143
Условное ожидание:
Для
P
xi · P (X = xiP
|A).
удобства E(f (X)) =
f (xi ) · P (X = xi ).
E(X|A) =
E(aX + Y ) = aE(X) + E(Y ).
V ar(X) = E((X − E(X))2 ).
Ковариация Cov(X, Y ) = E((X − E(X))(Y − E(Y ))).
p
Cтандартное отклонение σX =
V ar(X)
Cov(X,Y )
Корреляция cor(X, Y ) =
σX ·σY
Дисперсия
С.в. независимы, если никакая информация о
X
не позволяет сделать никаких выводов о
Y.
E(XY ) = E(X)E(Y ). Для дискретных независимых случайных величин
P (X = x ∩ Y = y) = P (X = x)P (Y = y), для непрерывных независимых pX,Y (x, y) = pX (x)pY (y).
X +...+Xn
Выборочное среднее X̄ = 1
,
n
P
(Xi −X̄)2
2
Несмещенная оценка дисперсии σ̂ =
n−1
Для независимых с.в.
Центральная предельная теорема:
Если
Xi
- iid,
0 < V ar(Xi ) < ∞,
то
Sn −E(Sn ) distribution
√
−−−−−−→ N (0; 1).
V ar(Sn )
n→∞
Пуассоновское приближение:
Если
Ln
- биномиальные с.в. с параметрами
Зоопарк:
−λt
p(t) = λe , Экспоненциальное, E(X) =
2
1
), Нормальное
p(t) = √2πσ
exp(− (t−µ)
2σ 2
1
,
λ
(n, pn )
и
n→∞
npn −→ λ,
V ar(X) =
то
P (Ln = k) → e−λ λk k!
1
λ2
t
P (X = t) = e−λ λt! , Пуассон, E(X) = λ, V ar(X) = λ
P (X = t) = CNt pt (1 − p)N −t , Биномиальное
p(~xT ) = c exp(− 21 (x − µ)T Ω−1 (x − µ)), Многомерное
X ∼ N (~µ; Ω), Ω
- ковариационная матрица,
c=
нормальное
1
n
1
(2π) 2 det 2 (Ω)
λ:
t - с.в. имеющая распределение Пуассона с ожиданием
События представляют собой Пуассоновский поток с параметром
Количество событий, происходящих за время
(λ · t).
Время между двумя событиями в потоке распределено экспоненциально с ожиданием
Уровень значимости = «порог редкости». Если происходит редкое событие, то
H0
отвергается.
Метод максимального правдоподобия: Maximum likelihood, ML.
max
Q
p(xi , θ)
θ
Полезен переход к логарифму.
Метод моментов: Method of moments, MM.
Если
E(X̄) = f (θ),
Оценка
θ̂
то находим
θ̂
из уравнения
неизвестного параметра
несмещенной, если
θ
1
.
λ
X̄ = f (θ̂).
называется:
E(θ̂) = θ
∀ε > 0 lim P (|θ − θ̂n | > ε) = 0.
состоятельной, если для
эффективной среди некоторого набора оценок, если у нее минимальная дисперсия.
144
MSE, mean squared error,
If
Xi
- iid,
P (Xi −npi )2
npi
Xi
- iid
N (µ, σ 2 ),
∼ χ2r−1 ;
N (µ, σ 2 ),
then
M SE(θ̂) = E((θ̂ − θ)2 )
(n−1)·σ̂ 2
σ2
P (Xi,j −np̂i,j )2
np̂i,j
Xn −µ
then q 2
σ̂
n
=
P
(Xi −X)2
σ2
∼ χ2(r−1)(c−1) .
∼ tn−1 .
χ(n−1) .
If
X ∼ N (0; 1)
and
K ∼ χ2n
then
Y = √XK
n
is called
tn .
If
Download